You are on page 1of 103

MUC LUC

• •

1 Cac phep dot hinh phang 1

§ 1 Dai cirong v~ cac phep bien hinh va cac phep doi hinh phang 1

1 Khai niem v~ phep bien hinh 1

2 D~nh nghia phep bien hmh . . . . . . . . . . . . . 2

3 Thi du v~ phep bien hinh 2

4 Bai tap v~ phep bien hinh trong tap hop di~m T . 4

5 Cac phan tir bat bien trong mot phep bien hmh. 5

6 Phep bien hinh dao ngiroc . . . . . . . 6

7 Tfch cac phep bien hinh . . . . . . . . . . . . 6

8 Dai cirong v~ cac phep doi hinh phang . . . . 7

9 Bai tap v~ dai cirong cac phep doi hinh phang 9

§ 2 Sir xac dinh va dang chinh tac cua mot phep doi hinh phang 10

1 V ~ sl! xac dinh mot phep doi hinh phang . . . . . . . 10

2 Quan he giira cac phep tinh tien va cac phep quay phang

voi cac phep dO'i xirng - true trong mat phang . . . . . .. 12

3 Dang chinh tac cua mot phep doi hinh phang 16

4 Phan Ioai cac phep doi hinh phang (Bai d9C them b6 sung

van cac §1 va §2). . . . . . . . . . . . . . . . . . . . . .. 17

5 Bai tap ve sl! xac dinh va dang chinh tac cia phep doi hinh

phang . . . . . . . . . . . . . . . . . . . . . . . . . . . .. 24 § 3 Van dung phep doi hinh van viec giai mot sO' bai toan hinh h9C

phang, Thi du minh hoa va bai tap 26

1 Ung dung phep tinh tien va viec khao sat tinh chat cua

hinh va dung hmh. . . . . . . . . . . . . . . . . . . . . .. 26

2 Ung dung phep dO'i xirng tam van kho sat tinh chat cua

hinh va dung hmh . 28

3 Nhimg dieu can Iuu y khi van dung phung phap bien hinh

van viec giai toan hinh h9C 29

4 Ung dung phep doi xtrng - true van viec giai toan hinh h9C phang . . . . . . . . . . . . . . . . . . . . . . . . . . . .. 31

5 Ung dung cac phep quay va doi hinh (n6i chung) van viec

giai toan ..... 34

6 Cau hoi va Bai tap . 37

11

2 Cac phep dong dang phang 41

§ 4 Sir xac dinh va dang chinh tac cua mot phep dong dang phang 41

1 Dai cirong v~ cac phep dong dang phang 41

2 Phep vi nr . . . . . . . . . . . . . . . . . . . . . . . . 43

3 Sir xac dinh mot phep dong dang phang . . . . . . . . 48

4 Diem bat dong va dang chinh tac cua mot phep dong dang

phang . . . . . . . . . . . . . . . . . . . . . . . . . . . .. 50

5 Bai tap ve xac dinh va dang chinh tac cua mot phep dong

dang phang . . . . . . . . . . . . . . . . . . . . . . . . .. 53

§ 5 Van dung phep dong dang van viec giai mot s6 bai toan hinh h9C phang - Thf du minh hoa va Bai tap . . . . . . . . . . . . . . . .. 55

1 Ung dung phep vi nr van giai toan hinh h9C . . . . . . .. 55

2 Ung dung phep dong dang thuan (vi nr quay) van viec giai

toan hinh h9C . . . . . . . . . . . . . . . . . . . . . . . .. 60

3 Ung dung phep dong dang nghich (vi nr - d6i xirng) van

viec giai toan hinh h9C . . . . . . . . . . . . . . . . . . .. 65

4 Bai tap van dung phep dong dang van viec giai toan hinh

h9C phang . . . . . . . . . . . . . . . . . . . . . . . . . .. 67

3 Tra Uri va hmmg dan giai bai tap § 1

§ 2

§ 3

§ 4

§ 5

71 71 73 75 80 81

4 Bai tap bi) sung va On tap tu giai 87

§ 1 Be) nic ve g6c dinh huang trong mat phang . . . . . . . . . . 87

§ 2 Dai cirong ve cac phep bien hinh va cac phep doi hinh phang 89

§ 3 Sir xac dinh va dang chinh tac cua mot phep doi hinh phang 89

§ 4 Van dung phep doi hinh van viec giai mot s6 bai toan hinh h9C

phang . . . . . . . . . . . . . . . . . . . . . . . . . . . . . . . .. 91

§ 5 Sir xac dinh va dang chinh tac cua mot phep dong dang phang .. 95 § 6 Van dung phep dong dang van viec giai mot s6 bai toan hinh h9C

phang . . . . . . . . . . . . . . . . . . . . . . . . . . . . . . . .. 97

Tai lieu tham khao 100

Chuong 1

Cac phep dOl hmh phang

§ 1 Dai cuong ve cac phep bien hinh va cac phep dOi hinh phang

1 Khai niern ve phep bien hinh

Cho hai tap hop diem 7 va 7' ta goi la mot song anh t11 7 vao 7', moi phep tuong ling f rna voi moi diem M cua 7 deu diroc gan voi mot diem M' duy nhat cua 7', ky hieu Ia M' = f(M).

Nhu vay, cho mot song anh f : 7 f---7 7' vao 7' Ia cho mot quy tac d~, vci bat ky mot diem M E 7 bao giO' ta cung c6 mot diem f (M) hoan toan xac dinh cua 7' sao cho

(i) Neu M va N la hai diem phan biet cua 7 thi f(M) va f(N) la hai diem phan biet cua 7'; M =I N thi f(M) =I f(N).

(ii) V6i'llM' E 7' thi bao giO' cung c6 mot diem M E 7 sao cho f(M) = M'.

Diem M' = f (M) diroc goi la anh, hay diem tuong ling hoac hinh bien d6i cua diem M qya anh xa f. Ngiroc Iai, diem M diroc goi Ia tao anh cua diem M' = f(M) qua anh xa f.

Neu M' = f(M) thi ta con noi rang anh xa f (6 day la mot song anh roi) bien diem M cua 7 thanh diem M' cua 7'.

Khi hai tap hop diem 7 va 7' la dong nhat, ciing c6 nghia la trung nhau, ky hieu 7 = 7', ta n6i rang f Ia mot phep bien hinh trong 7 (hay t11 7 vao chinh n6). Nhu vay, ta co th~ dinh nghia mot phep bien hinh tren dirong thang, trong mat phang hay trong khong gian tuy theo 7 Ia tap cac di~m cua mot dirong thang ,6. nao do trong mat phang, hay 7 la tap hop tat d. cac diem cua mot mat phang p hay 7 la tap hop tat d. cac diem cua khong gian /C. Tham chi 7 c6 the la tap

1

2

hop tat ca cac di~m cua mot hinh H nao do Ia mot bo phan (tap con) cua dirong thing ~, hay mot bo phan cua mot mat phang P hay mot bo phan cua khong gian; ky hieu H c ~, H c P hay H E /C. Ta e6 dinh nghia sau dau

2 Dinh nghia phep bien hinh

Mot song anh f : ~ ----+ ~' hoac P ----+ P t11 tap cac diem cua dirong thing ~ hay cua mat phang P len chinh n6 diroc goi la mot phep bien hinh tren dirong thing ~ hay cua mat phang P.

Nhu vay chang han eho mot phep bien hinh cua mat phang f : P f---7 P Ia mot quy tac d~ vci moi di~m cua P ta tim diroc mot di~m M' = f (M) hoan toan xac dinh, thoa man hai dieu kien sau day

(i) Neu M va N deu thuoc P, M =I N thi f(M) va f(N) deu thuoc P, f(M) =I f(N)

(ii) Voi \1M' E P thi ton tai duy nhat diem M E P sao eho f(M) = M'.

Neu H la mot hinh nao do cua P thi ta e6 the xac dinh diroc tap hop diem H' = f(H) = {f(M), M E H}; f(H) la mot hinh phang diroc goi la anh hay hinh bien d6i hoac hinh nrong ling cua hinh H qua phep bien hinh f; ngiroc lai hinh H diroc goi Ia tao anh (hay hinh nguyen cua hinh f(H) qua phep bien hinh f (Hinh 1)

Hinh 1

Clui thich, Phep bien hinh dinh nghia nhu tren con diroc goi mot each chinh xac han Ia phep bien hinh diem (vi n6 bien d6i diem thanh diem). Hai phep bien hinh di~m f va f' Ia nrong dirong neu vci moi di~m M cua T deu co cung mot anh trong T: \1M E T suy ra M' = f(M) = f'(M), ta viet f' = f.

3 Thi du ve phep bien hinh

Thi du 1. Trong mat phang eho mot dirong thing ~ va mot di~m 0 co dinh tren ~. Voi m6i diem M =I 0 tren ~ ta lay diem M' d6i xirng voi M qua 0; dieu do cling e6 nghia la M' diroc hoan toan xac dinh boi ding tlnrc vee 1(1

---+ ---+

OM'=-OM

(1)

3

Neu M trung 0 thi ta lay M' = o. The thi r5 rang Ia anh xa f : M f---7 M' tir ,6. ----+ ,6. xac dinh nhu tren la mot song anh tir dirong thing ,6. van chinh n6, hay thea dinh nghia thi f la mot phep bien hinh cua dirong thing ,6.. D6 chinh la phep et6i xtrng tam 0 cua dirong thing ,6., ta ky hieu la 1>0 hay 'D( 0). Vay 'D( 0) : M f---7 M' tir ,6. ----+ ,6. va T = ,6..

Chti thich, Trong tnrong hop nay ta cung c6 1>(0) : M f---7 M. Neu T = P va M' = f (M) diroc xac dinh boi (1) thi f : M f---7 M' tir P ----+ P Ia mot phep et6i xtrng tam 0 cua mat phang P.

Thi d1).2. G9i Ao, Eo va Co liln Iuot la trung diem cac canh EC, CA va AB cua mot tam giac ABC cho tnroc trong mat phang. V ai moi diem M cua mat phang ta lay liln Iuot cac eti~m A', E', va C' et6i xirng vci M thea thtr nr qua Ao, Eo, va Co. Chirng t6 rang

1/ Cac dirong thing AA', EE' va CC' dong quy 6' mot diem M' nao et6;

2/ Anh xa f : M f---7 M' tir P van chinh n6 Ia mot phep bien hinh cua mat phang.

Hinh2

That vay, tir gia thiet d~ dang suy ra diem M la etinh thir tu chung cua ba hinh binh hanh EA'CM,CB'AM, va AC'EM. Tir et6 suy ra ECE'C',CAC'A' va AE A' E' deu Ia nhtmg hinh binh hanh va moi hinh nay nhan hai trong ba doan thing AA', EE' va CC' lam cac dtrong cheo. (Hinh 2).

Do et6 ba dirong thing AA', EE' va CC' doi mot cat nhau 6 cung mot diem; et6 la trung diem chung M' cua ca ba doan thing et6. Nhu vay voi moi diem M cua mat phang P eta chi ra mot quy tac f xac dinh hoan toan diem f (M) = M'

4

nhir da eho a tren va do do f Ia mot dan. anh,

Dao Iai, voi m6i diem M' cua mat phang ta H1Y cac diem A', B' ,C' thea thir tir d6i xtrng voi A, B, C qua M'. The' thi r5 rang M' Ia tam chung cua ba hinh binh hanh BC B' C', C AC' A' va ABA' B'. Bay gio ta H1Y diem M d6i xirng voi A' qua Ao; n6i khac di, M Ia dinh tlur nr cua hinh binh hanh B A' C M. Tir do suy ra M Ia dinh thtr tir chung cua ca ba hinh binh hanh BA'CM, CB'AM va AC' BM. B6i vay, M d6i xirng voi B' qua Bo, dong thai cung d6i xirng voi C' qua Co. N6i khac di la, AoA', BoB' va CoC' dong quy 0 diem M.

Nhu vay Ia, voi moi diem M' bat ky cua mat phang P ta da chi ra diroc each xac dinh diem M va diem M nay Ia duy nhat sao eho f (M) = M'. Va do do, f Ia mot toari anh,

Ket ltuin, Anh xa f : M f---7 M' tir P ----+ P vira Ia dan anh, vira Ia toan anh nen thea dinh nghia, f la mot phep bien hinh cua mat phang.

Clui thich, Neu goi G Ia trong tam cua tam giac ABC , ta con cluing minh diroc rang dirong thing M M' dia qua G va cap diem M, M' thoa man he thirc vecto

---+ 1 ---+

GM'=--GM 2

(2)

H~ thirc (2) cung noi len rang quy tac f : M f---7 M' tir P ----+ P chi ra trong thi du 2 nrong dirong voi quy tac (anh xa) I'. trong do voi m6i diem M bat ky cua P ta xac dinh diroc duy nhat diem M' E P thea (2). Ta se tro lai thi du 2 nay 6 phan sau, § 4.

4 Bai tap ve phep bien hinh trong tap hop di~m T

1.1 Trong mat phang eho mot dirong tron (0) va hai tiep tuyen a va b cua n6.

M9t tiep tuyen t thay d6i cua (0) c:it a va b thea tlur nr 6 M va N. Hoi anh xa f : M f---7 N tir a den b e6 phai Ia mot song anh khong? Xet hai duong tnrong hop a song song vci b va a cat b .

1.2 Trong mat phang eho hai dirong thing phan biet a va b. G9i A va B thea tlur nr Ia cac hinh chieu (1) tren a va b cua mot diem M bat ky cua mot mat phang; M' Ia di~m d6i xirng vci M qua trung di~m I cua doan thing AB. Chirng minh rang anh xa f : M f---7 M' tir P ----+ P Ia mot phep bien hinh cua mat phang khi va chi khi a va b khong vuong g6e voi nhau.

1.3 Trong mat phang eho hai dirong thing a va ,6. cat nhau (nhirng khong vuong g6e) a 0 va m9t di~m P e6 dinh khac 0 sao eho 0 Ia hinh chieu cua P

(l)Tir day tro di, neu khong co clni thich gl them, ta hieu hinh chieu Ia hinh chieu vuong goc.

5

tren a. V 6i moi di~m M tren ~, ta dung dirong tron (0 P M) cat lai a a C, roi dirong tron tam C di qua M cat lai ~ 6 N

a) Chirng minh rang anh xa f : M f---7 N tir ~ vao chinh no HI. mot phep bien hinh cua dirong thing ~.

---+

b) Hay cluing minh M N khong d6i voi moi M tren ~. Tir do co the goi

ten diroc phep bien hinh f cua ~.

1.4 Gia sir M HI. mot diem bat ky nam trong mat phang P cua mot tam giac ABC da cho. Dung cac duong thing Ax, By va Cz nrong img doi xirng voi vac dirong thing AM, B M, va C M liln Iuot qua phan giac cua cac goc A, B va C cua tam giac ABC.

a) Neu M nam trong tam giac ABC, cluing minh rang Ax, By, Cz dong quy 6 mot diem M' cling nam trong tam giac ABC.

b) Chirng minh rang anh xa f : M f---7 M' tir P ----+ P khong phai Ia mot phep bien hinh cua mat phang.

c) Cluing minh rang quy tfch nhtmg diem M trong mat phang sao cho Ax II By II CZ la dirong tron ngoai tiep tam giac ABC

d) TIm tap hop diem 7 (7 c P) d~ anh xa f : M ----+ M' tir 7 ----+ 7 la mot phep bien hinh cua tap hop di~m 7, mot bo phan cua mat phang P.

5 Cac phan nr bat bien trong mot phep bien hinh,

M9t di~m M nao do cua tap di~m 7 rna trung vci anh f (M) cua no trong mot phep bien hinh diem f cua 7 goi la mot diem bat dong hay diem kep doi voi phep bien hinh f trong 7: M E 7 rna f (M) = M, M la mot diem bat dong cua f trong 7. Phep bien hinh cua mat phang rna moi diem cua mat phang deu Ia diem bat dong goi Ia phep bien hinh dong nhat hay goi teit Ia phep dong nhat, ky hieu la Id hay e, f = Id khi va chi khi M = f(M), '11M E P.

MQi hinh H (cua P) rna trung vci anh (hinh bien d6i) H' = f(H) cua no trong phep bien hinh f diroc goi la mot hinh bat bien doi voi phep bien hinh f cua P.

Neu mot hinh H bat bien doi voi f trong P rna moi diem cua no deu bat dong thi H diroc goi Ia mot hinh co dinh doi vci f.

Ching han, trong phep doi xirng 'D( 0) thi tam doi xirng 0 Ia mot di~m bat dong duy nhat va moi dirong thing qua 0 deu la bat bien. Trong phep doi xirng 'D (~) true ~ thi true doi xtrng ~ Ia hinh co dinh con moi dirong thing vuong

6

g6c val Ll deu la bat bien.

Trong phep tinh tien 'Y(1i) thea vecto 1i khac vecto khong thi khong c6 diem bat dong nao, nhung moi dirong thing c6 phirong cua 1i (nrc song song voi 1i) deu la bat bien.

6 Phep bien hinh dao ngiroc

D~ thay rang moi song anh f t11 7 ----+ 7' deu xac dinh mot song anh f' khac t11 7' ----+ 7, goi Ia anh xa ngiroc cua f va ky hieu Ia f-1 : f' = f-l. Khi 7' = 7 thi f-1 goi Ia phep bien hinh dao ngiroc cua f trong 7.

D~ thay rang, phep tinh tien 7(1i) thea vecto 1i khac vecto khong c6 phep bien hinh dao ngiroc cling Ia mot phep tinh tien, nhung thea vecto -1i

7-1 (1i) = 7( -1i)

con phep doi xirng tam 'D ( 0) hay phep doi xirng true 'D (Ll) thi phep dao ngiroc trung voi chinh n6: 'D-1(0) = 'D(O), 'D-1(Ll) = 'D(Ll)

MOt phep bien hinh f cua P diroc goi Ia phep bien hinh doi hop neu moi diem ME P trung val anh cua di~m nrong irng M' = f(M) cua n6: '11M E P: M' = f(M) thi M = f(M').

Nhu vay, f la phep bien hinh doi hop khi va chi khi f = f-1 . Phep doi xirng tam va phep doi xtrng tam va phep doi xtrng true Ia nhtmg thi du ve phep bien hinh doi hop trong mat phang.

7 Tich cac phep bien hinh

Gia sir I, va h la hal phep bien hinh cua mat phang I, : P ----+ P va h : P ----+ P; M la mot diem bat ky cua P. The thi, '11M E P, 3! M, E P va M' E P sao cho M; = ft(M) va M' = h(M1) va do do, anh xa f : M f---7 M' t11 P ----+ P cling la mot song anh, nghia la f tnrc tiep bien M thanh M' cling la mot phep bien hinh cua P.

Nhu vay, t11 hal phep bien hinh I, va h cua P ta da xac dinh mot phep bien hinh f cling cua P goi la tfch cua hai phep bien hinh I, va h, ky hieu la h 0 ft. Vay ta c6

M' = f(M) = h[Jl(M)], '11M khi va chi khi M' = h 0 ft(M)

va do do f = h 0 ft. Cling vay, tfch h 0 h 0 I, = h 0 (h 0 fd la tfch cua cac phep bien hinh (h 0 fd va h thea thir tir do; tfch !4 0 h 0 h 0 I, la tfch cua cac

7

phep bien hinh (h 0 12 0 fd va 14 thea thtr nr do, v.v ...

Can Iuu y rang phep bien hinh f = 12 0 I, Ia ket quMt cua viec thuc hien lien tiep hai phep bien hmh: phep bien hinh thir nhat la I, va phep bien hinh thir hai Ia 12, con g = I, 012 cling Ia mot phep bien hinh nhung diroc thuc hien thea tlur nr ngiroc lai. N6i chung, tfch f = 12 0 h va tfch g = h 012 Ia hai phep bien hinh khac nhau.

Tfch cac phep bien hinh c6 nhtmg tinh chat sau day

1/ Ket hop, nghia Ia h 0 (12 0 fd = (h 0 h) 0 I, = h 0 12 0 h. Nhu vay, bao giO' cling c6 th~ thay hai hoac nhieu phep bien hinh lien tiep bai tfch cua cluing, hoac ngiroc Iai, c6 the thay mot phep bien hinh nao do boi mot tfch tuong dirong.

2/ N6i chung, tfch cac phep bien hinh khong giao hoan; tfch hai phep bien hinh I, va 12 goi Ia giao hoan neu I, 0 12 = 12 0 h·

3/ Trong tap hop cac phep bien hinh cua di~rn p, phep dong nhat I d Ia phan til trung hoa trong cac phep toan tfch

Vf: Idof=fold=f

4/ Tich hai phep bien hinh dao ngiroc nhau la phep dong nhat

f-1 0 f = f 0 r' = Id, (Vf)

Tir do suy fa f 0 f = P = I d khi va chi khi f = f-1 khi va chi khi f Ia phep bien hinh d6i hop. Ngoai fa, phep bien hinh dao ngiroc cua tfch 12 0 I, la tfch t;' 0 t:'. vay

That vay ta c6

(12 0 fd 0 (11-1 0 f2-1) = 12 0 (11 0 fl1) 0 t:' = 12 old 0 i:'

= 120 (Idof2-1)

= 12 0 t:'

= Id

8 Dai cirong ve cac phep dOi hinh phfiog

a) Djnh nghia ph.ep doi hinh

8

Mot phep bien hinh f : P f--7 P diroc goi Ia mot phep doi hinh cua mat phang, ky hieu la D neu bat ctr hai diem M, N nao cua P va cac anh M' = f(M) va N' = f(N) cua cluing ta deu co M' N' = M N. Noi mot each ngan gon, phep doi hinh cua mat phang, hay goi van tat la phep doi hinh phang, la phep bien hinh bao toan khoang each giira bat err hai diem nao cua mat phang. V ay Ia

f E {V} cuaP khi va chi khi f(M)f(N) = M N, (V M, N E P)

{V} Ia ky hieu tap cac phep doi hinh cua mat phang.

Chti thich, Chfnh VI phep doi hinh bao toan khoang each giira bat err hai di~m nao nen ngiroi ta goi no la phep bien hinh dang cu hay goi van tat la phep aling qt.

Til dinh nghia cua phep doi hinh ta suy fa cac he qua.

a. Phep bien hinh dong nhat I d la mot phep doi hinh

b. Phep bien hinh dao ngiroc cua mot phep doi hinh ciing Ia mot phep doi hmh.

c. Tfch cua hai hay nhieu phep doi hinh la mot phep doi hmh.

d. Phep tinh tien, phep d6i xirng tam, phep d6i xirng true Ia nhtrng phep doi hmh.

b) Cac tinh chfit cua ph ep doi hinh

Dinh ly 1. Phep doi hinh bao toan su thang hang cua ha di~m va thtr nr cua cluing tren dirong thang chua ba diem va thir nr cua cluing tren dirong thang chua ba diem do. Cu the la

Phep doi hinh bien ba diem thang hang A, B, C thanh ba diem A', B' , C' thang hang ciing thea tlur nr do. Phep cluing minh xem nhu Ia bai tap cho ban doc.

H~ qua 1. Phep doi hinh bien mot dirong thang thanh mot dirong thang, bien mot tia thanh mot tia, bien mot doan thang thanh mot doan thang nao do.

H~ qua 2. Phep doi hinh bien mot tam giac thanh mot tam giac bang no, bien mot goc thanh mot goc bang no, bien mot dirong tron thanh mot dirong tron bang no, trong do tam bien thanh tam.

Dinh ly 2. Mot phep doi hinh phang co ha di~m bat dong khong thang hang la phep bien hinh dong nhat.

Chang minh. Gia sir f : P ---+ P Ia mot phep doi hinh phang co ba diem bat dong khong thang hang (Hinh 3.): A = A' = f(A), B = H' = f(B) va

9

C = C' = f( C). The thi, thea tfnh chat cua phep doi hmh, bat err mot dl~m nao tren cac dirong thing (BC), (C A) hoac (AB) deu la diem bat dong (hay cluing minh dieu do). Tir do d~ dang suy ra moi diem M cua mat phang (ABC) deu la diem bat dong, va do do f = I d

Hinh 3

H~ qua 3. MQt phep doi hinh phang V =I I d thi hoac khong c6 diem bat dong nao hoac c6 mot diem bat dong duy nhat, hoac c6 mot dirong thing rna moi dl~m cua n6 deu Ia dl~m bat dong, tire Ia c6 mot dirong thing co dinh.

9 Bai tap ve dai cuong cac phep dOi hinh phang

1.5 Cluing minh rang phep doi hinh bao toan su thing hang cua bat ky ba diem nao, do do, bien dirong thing thanh dirong thing (Dinh 1y 1).

1.6 Chirng minh rang phep doi hinh bao toan quan he song song cua hai dtrong thing va khoang each giira cluing, nghia Ia, neu a II b, a' = V(a), b' = V(b) thi a' II b' va d(a', b') = d(a, b), trong do d(x, y) chi khoang each giira hai dirong thing song song x va y.

1.7 Hay sir dung phirong phap cluing minh phan cluing, cluing minh tfnh chat neu trong Dinh 1y 2 cua phep doi hinh phang.

1.8 Chirng minh chi tiet han H~ qua 2 cua Dinh 1y 1 ve tinh chat cua V, urc la Phep doi hinh bien mot tam giac thanh mot tam giac bang n6, trong do trong tam G, tnrc tam H, cac tam I va 0 cua cac duong tron noi tiep, ngoai tiep cua tam giac tao anh ABC thea thtr nr bien thanh cac tam nrong irng G' , H' ,1' va 0' cua tam giac anh A' B' C'.

1.9 Chirng minh rang trong mot phep tinh tien, anh d' cua mot dirong thing d thi, hoac song song voi d hoac trung d. Tir do suy ra tap hop tat ca cac duong thing bat bien trong mot phep tinh tien.

10

---+ --+

1.10 Cluing minh rang phep d6i xirng tam 1)(0) bien mot vecto M N = a thanh

---+ ---+---+

mot vecto d6i M N voi n6, urc Ia M N = - M' N' = -C;. Tir do suy fa rang

phep d6i xtrng - tam bao toan phirong cua moi dirong thang; dong thai tim diroc tap hop tat ca nhtmg duong thang bat bien qua phep d6i xirng - tam.

1.11 Chirng minh rang phep d6i xirng - true 1)(6.) bien mot dirong thang a thanh dtrong thang a' d6i xirng voi a qua true 6.. Tir do tim diroc nhtmg phirong dtrong thang bat bien trong mot phep d6i xtrng - true.

1.12 Hay tim nhtmg phep doi hinh 1) bien mot tam giac deu ABC thanh chinh n6.

1.13 TIm nhtmg phep doi hinh 1) bien mot hinh vuong ABC D thanh chinh n6. 1.14 TIm nhtrng phep doi hinh 1) bien mot hinh binh hanh ABCD thanh chinh n6.

1.15 TIm nhtmg phep doi hinh 1) bien mot hinh chir nhat ABCD thanh chinh n6. 1.16 Chirng minh rang tfch cua hai phep d6i xirng true c6 cac true vuong g6c a diem 0 la phep d6i xirng tam D(O). Nguoc Iai, mot phep d6i xirng tam 'D( 0) c6 the phan tfch bang vo s6 each thanh tfch cua hai phep d6i xtrng true 'D(6.1) va 'D(6.2) mien Ia hai true 6.1 va 6.2 vuong g6c voi nhau 6 o.

§ 2 Sir X3C dinh va dang chfnh tac cua mot phep diri hinh phang

Khi cluing ta n6i cho mot phep doi hinh 1) rna tong quat han Ia cho mot phep bien hinh f cua P (mat phang) nrc la chi fa day du cac yeu t6 d~ xac dinh hoan toan phep doi hinh (hay phep bien hinh) do cua P. Dieu do c6 nghia la

V6i moi diem M bat ky cua P ta phai chi fa each (quy tac) dung, ciing la each xac dinh diroc diem nrong img (anh) M' cua n6 qua phep doi hinh (hay phep bien hinh) nay.

V~ phep doi hmh, ta da biet rang mot phep doi hinh bien mot tam giac ABC thanh mot tam giac A' B' C' bang n6, trong do cac canh tuong irng bang nhau, cac g6c tuong irng bang nhau. Menh d~ sau day khang dinh dieu ngiroc lai

1 Ve S1;I' xac dinh mot phep dOi hinh phfiog

Djnh nghia 3. Cho ABC va A' B' C' la hai tam giac bang nhau cho tnroc trong mat phang P voi B' C' = BC, C' A' = C A, A' B' = AB. Bao gia ta ciing c6 mot va chi mot phep doi hinh 1) : P ----+ P bien A thanh A', B thanh B' va C

11

thanh G'. Dong thai, phep doi hinh V nay c6 th~ phan tfch thanh tfch cua khong qua ba phep dO'i xirng true.

Chang minh. Tnroc het, d~ trinh bay diroc gon gang ta quy troc ky hieu nhu sau: t[M N] Ia trung true cua doan thing M N; 'D~i hoac 'D(~i) chi phep dO'i xirng true c6 true Ia ~i' trong d6 i Ia chi sO' 1,2,3.

Hinh 4

Ton tai. Neu A va A' la hai diem phan biet, ta goi ~l = t[AA']. Qua phep 'D(~l) tam giac ABG bien thanh tam giac A' BIGI bang n6 nen DA' BIGI = DA'B'G', do d6 A'BI = A'B', (Hinh 4.)

Neu B, =I- B', ta goi ~2 = t[BIB'] thi thea chimg minh tren, A' E ~2.

Boi vay, qua phep 'D(~2) tam giac A'BIGI bien thanh tam giac A'B'G2 bag n6, nen DA' B' G2 = DA' B' G' va do d6 A' G2 = A' G', B' G2 = B' G'. Den day, neu G2 =I- G', ta goi ~3 = t[G2, G'] thi A' va H' deu thuoc ~3. Va cuoi cling phep dO'i xirng true 'D(~3) bien tam giac A' B'G thanh tam giac A' B'G'. Nhu vay Ia, thuc hien lien tiep ba phep dO'i xirng true 'D(~i) voi i = 1,2,3 thi tfch f = 'D(~3) 0 'D(~2) 0 'D(~l) la mot phep doi hinh V bien tam giac ABG thanh tam giac A'B'G' trong d6 A' = V(A), B' = V(B) va G' = V(G).

Trong qua trinh cluing minh, cluing ta da ba 1iln sir dung den tir \neu" (cling c6 nghia la \gia sir rang") va moi 1iln nhu the deu xuat hien mot phep doi xtrng true 'D(~i) thea tlur nr i = 1,2,3 Voi nhan xet nay, ta thay rang neu A' = A thi khong phai thuc hien 'D(~l). Neu B, = B' thi khong phai su dung den 'D(~2) va neu G2 = G' thi cling khong phai sir dung den 'D(~3).

N6i t6m Iai, moi 1iln c6 mot cap diem trong ba cap diem chi ra a tren rna trung nhau thi so' phep doi xtrng true can phai thuc hien giam di mot.

Duy nluit. Gia su rang c6 hai phep doi hinh VI : P ----+ P va V2 : P ----+ P

12

cling bien tam giac ABC thanh tam giac A' B'C'. The thi phep bien hinh (doi hinh) tfch V;;I 0 VI cling la mot phep doi hinh V bien A thanh A, B thanh B va C thanh C. Nhu vay la phep doi hinh V = V;;I 0 VI co ba diem bat dong khong thing hang la A, B va C. Theo Dinh 1y 2, phep doi hinh V nay phai la phep dong nhat (V;;I 0 VI = I d) va do do

(1)

Mat khac, nho tinh chat ket hop cua tfch cac phep bien hinh ta lai co

(2)

Doi chieu (1) va (2) ta suy fa V2 = VI. Va Dinh 1y 3 da: diroc chimg minh.

H~ qua 1. Tich cua n. (1 :s; n. Ia mot so nguyen duong bat ky) phep doi xirng true trong mat phang bao gia cling phan tfch diroc thanh tfch cua khong qua ba phep doi xirng true.

That vay, VI phep doi xtrng true Ia mot phep doi hinh nen tfch cua ti tuy y nhtrng phep doi xirng - true, cling Ia mot phep doi hinh V nao do. Boi vay thea Dinh 1y 3, ta co dieu can cluing minh.

H~ qua 2. M6i phep doi hinh V neu khong phai Ia phep dong nhat thi, hoac la mot phep doi xirng - true hoac la (tuong dirong voi) tfch cua ba phep doi xirng - true.

2 Quan he gifra cac phep tinh tien va cac phep quay phfing vrri cac phep doi xirng - true trong mat phang

Chung ta da: bier phep doi xirng - true, phep tinh tien va phep quay xung quanh mot diem (bao gom trong do d. phep doi xirng - tam) deu la nhtmg phep doi hmh. Nhung Dinh 1y 3 (H¢ qua 2) b tren lai cho biet bat ky phep doi hinh phang V nao, khac I d deu hoac ban than la mot phep doi xirng-truc, hoac tuong dirong voi tfch cua hai hay ba phep doi xirng-truc. Vi the, cluing ta can phai lam fa moi quan he giira hai 10~i tfch cac phep doi xirng true nay vci cac phep tinh tien va cac phep quay. Ngoai fa, Dinh 1y 3 va cac H¢ quaa 1, H¢ quaa 2 cua no con co y nghia het sue dac biet va quan trong b ch6 chi fa rang: Cac phep doi xirng-truc dong vai tro nen tang trong viec tao thanh tat d. cac phep doi hinh phang. Cling chinh VI vay ngiroi ta con noi rang phep doi xirng - true phang Ia tap hop phan tir sinh {V} cac phep doi hinh phang.

Tinh chat sau day cua phep tinh tien phang lam fa mci quan he giira phep tinh tien phang va cac phep doi xirng-truc trong mat phang.

13

Dinh ly 4. Tich cua hai phep doi xirng-truc (trong mat phang) co true song song song HI. mot phep tinh tien phang theo vecto vuong goc voi hai true va gap doi vecto cua phep tinh tien bien true thir nhat .6.1 thanh true thir hai .6.2 (Hinh 5.)

Hinh5

Dinh ly 4'. (Dinh ly dao cua Dinh ly 4). Dao Iai, moi phep tinh tien phang

--+

theo mot vecto ~ (i=- 0) deu co the the phan tfch diroc bang vo so each thanh tfch

cua hai phep doi xirng - true co true song song va vuong goc vci vecto tinh tien ~, trong do true thir hai .6.2 diroc suy ra t11 true thir nhat .6.1 boi phep tinh tien theo vecto ~ ~. Chirng minh ca hai Dinh ly 4' va 4 xem nhu la bai tap.

Moi quan he giira phep quay xung quanh mot diem, trong do co phep doi xtrng - tam va cac phep doi xirng true, ta co dinh ly sau:

Dinh ly 5. Tich cua hai phep doi xirng true trong mat phang qua hai true cat nhau diem 0 la phep quay xung quanh tam 0 rna goc quay rp gap doi goc quay () cua phep quay tam 0 bien true thir nhat .6.1 thanh true thir hai .6.2

Hinh 6

'D(.6.2) 0 'D(.6.1) = Q(O, rp = 2(} mod 2Jr) (.6.1 n.6.2 = 0, (.6.1, .6.2) = O (mod Jr))

Djnh ly 5'. (Dinh ly dao cua Dinh ly 5). Dao Iai, moi phep quay phang Q ( 0, rp) deu co the phan tfch diroc bang so vo each thanh tfch cua hai phep doi

14

xirng true qua hai dirong thing cat nhau a tam quay 0, mien Ia true doi xirng thtr hai ~2 diroc suy fa t11 true thir nhat ~1 boi phep quay tam 0 g6c () = ip /2 bang nih g6c quay rp cua phep quay diroc xet.

v ~ moi quan he giira phep doi xirng tam (phep quay dac biet, g6c 7r) va cac phep doi xirng - true trong mat phang, ta c6 dinh ly sau:

Dinh ly 6. Tich cua hai phep doi xirng - true trong mat phang qua hai dirong thing vuong g6c la mot phep doi xirng tam rna tam doi xirng 0 la giao diem cua hai dtrong thing d6. Han nfra, tfch nay giao hoan diroc (Hinh 7)

Dao Iai, moi phep doi xirng - tam trong mat phang deu c6 the xem la hai tfch cua hai phep doi xirng - true qua hai duong thing vuong g6c voi nhau a tam doi xirng.

~1 =I ~2' ~1 n ~2 = 0; 'D(~2) 0 'D(~1) = 'D(~1) 0 'D(~2) = 'D( 0) Dieu nay tuong duong voi ~1 vuong g6c voi ~2 tai diem o.

Hinh 7

T6ng hop ca ba Dinh ly 4,5 va 6 (khi xet phep doi hinh V =I I d Ia tfch cua hai phep doi xirng true trong mat phang ta di den ket Iuan phat bien trong dinh ly sau:

Dinh ly 7. MQi phep doi hinh (=I I d) tuong dirong voi tfch cua hai phep doi xirng true thi, hoac Ia mot phep tinh tien, hoac Ia mot phep quay xung quanh mot di~m hay dac biet la phep doi xirng tam tuy thea hai true doi xirng song song vci nhau hoac cat nhau, hay dac biet cat nhau thea mot g6c vuong.

Bay gia ta xet tnrong hop phep doi hinh la tfch cua ba phep doi xirng true.

Tnroc het ta d~ cap den tnrong hop dac biet a d6 hai true doi xtrng song song va true tlur ba vuong g6c voi hai true song song d6. Nhu vay, trong tnrong hop nay phep doi hinh Ia tfch cua mot phep tinh tien va mot phep doi xirng - true c6 true cung phirong vci vecto tinh tien thea thtr nr d6 hay thea thtr nr ngiroc lai (Hinh 8). D~ thay fang tfch nay giao hoan duoc. Ta c6 dinh nghia sau day

15

Dinh. nghia phep dOL xung - truat, Tich giao hoan cua mot phep d6i xirng - true 'D(~) va mot phep tinh tien 'YC~) thea phirong cua true d6i xirng ~ diroc goi Ia mot phep doi xtrng - tnrot cua map phang, va ~ diroc goi Ia true doi xtrng - tnrot. Ky hieu: 'D(~, ti)

-+ -+ -+

'D(~, v) = T(v) o'D(~) = 'D(~) oT(v)

Dieu nay nrong dirong voi ~ II ti. Truong hop ba true d6i xirng song song voi nhau, hoac dong quy 6 mot diem ta c6 ket qua sau.

Dinh If 8. Tich cua ba phep d6i xirng - true c6 cac true song song voi nhau hoac dong quy 6 mot diem la mot phep d6i xirng - true (c6 true song song voi ba true d:iu hoac di qua di~rn chung cua ba true d6) Hay cluing rninh dieu do, xern la bai tap bat buoc.

Hinh 8

T6rn Iai, ngoai hai tnrong hop phep doi hinh V la phep d6i xirng- true hay phep d6i xtrng - tnrot rna n6i 6 tren, cluing ta chi con phai xet tnrong hop tong quat, 6 do ba true d6i xirng khong song song voi nhau, cling khong dong quy, nrc la

a) Hoac doi mot ceit nhau tao thanh mot tam giac

b) Hoac mot true nao do cat hai true con lai song song

Tuy nhien, chi viec sir dung cac Dinh ly 5 va 5', cluing ta d~ dang dira diroc tnrong hop b) v~ tnrong hop a), hoac ngiroc lai (xern Hinh 9a, 9b). Dac biet, c6 th~ dira ca hai tnrong hop nay v~ tnrong hop mot trong hai true thtr hai hoac thir ba vuong g6c voi true thir nhat ~1 (true nay van diroc gill nguyen tir d:iu den gio) d~ tao thanh mot tam giac AD H vuong g6c 6 H (Hinh ge). Nhu vay cluing ta da thay cap true ban d:iu ~2' ~3 hoac ~;, ~~, bang cap true d2, d3 trong do d3 .L ~1 va ~2 n ~3 = ~; n ~~ = d2 n d3 = A mien la cac g6c 6 A la (~2' ~3) = (~;, ~~) = (d2, d3) = a + 2k7r, (k E Z) khong d6i. Ta goi qua trinh bien d6i nrong duong nay Ia qua trinh nrong dirong hoa phep doi hinh V = 'D(~3) 0 'D(~2) 0 'D(~1) n6i tren (xern Hinh 9a, b, c).

16

Cuoi cung, ta thay cap true H A, H D vuong g6c vci nhau a hinh H hai cap true d~ = ,6. va d1 cling vuong g6c voi nhau 6 H trong d6 H E d1 II d2 va ,6. = (H K) vuong g6c voi d1 6 H va d2 6 K (Hinh 9d). Den day ta da dira diroc tfch D = '])(,6.3) 0 '])(,6.2) 0 '])(,6.1) cua ba phep dO'i xtrng - true 6 dang tong quat (D khong phai la phep dO'i xirng true) v~ tnrong hop dac biet da xet den 6 tren. D6 la tnrong hop cua phep dO'i xirng - tnrot a do hai true dO'i xirng song song con true thir ba thi vuong g6c voi hai true nay (Hinh 9d suy ra Hinh 8).

Hinh 9

3 Dang ehinh tac ella mot phep dOi hinh phang

Sau khi thuc hien viec bien d6i nrong dirong tfch cua ba phep dO'i xirng true ket hop vci cac Dinh ly 7 va 8 ta thu diroc ket qua sau day noi len mci quan he giira mot phep doi hinh bat ky voi phep dO'i xirng - true trong mat phang va tfch cua cluing goi la dang chinh tac cua mot phep doi hinh phang.

Dinh If 9. (V ~ dang chinh tac cua mot phep doi hinh phang) M9t phep doi hinh phang neu khong phai Ia phep dong nhat thi, hoac Ia mot phep dO'i ximg-truc, hoac Ia mot phep quay hay mot phep tinh tien, hoac Ia mot phep dO'i xirng-tnrot.

Chti thich, Cac phep quay va tinh tien n6i trong Dinh ly 9 c6 the goi la nhtmg

--+

phep quay hay tinh tien thuc sir, tire la g6c quay rp =I- 0, vecto tinh tien ~ =I- O.

Con phep quay g6c ip = 0 hay phep tinh tien thea ~ =I- -0 deu la khong thuc su

--+

va Ia phep dong nhat: Q(P,O) = I d, 'J( 0 = I d.

Co the xem phep dO'i xirng-truc la tnrong hop dac biet cua phep dO'i xirng -

--+ --+

tnrot 6 d6 vecto tnrot (vecto tinh tien) bang 0 nrc 1 a: ']) ( ,6., 0) = D ( ,6.).

Phep quay (thuc su) co mot di~m bat dong duy nhat Ia tam quay; phep tinh tien (thuc su) khong co di~m bat dong nao; phep dO'i xirng - tnrot cling khong co diem bat dong nao. Tuy nhien, phep dO'i xirng-truc c6 mot dirong thing cO' dinh rna moi diem tren d6 deu la diem bat dong, d6 la true doi xiing.

17

4 Ph an loai cac phep doi hinh phfing (Bai doc them b6 sung van cac §1 va §2)

1. Sl! can thiet phai dira vao khai niem duong thang dinh huong mat phang dinh hmmg va goc dinh hmmg trong mat phang dinh hmmg

Nhu cluing ta da biet, d6i tuong hinh hoc d:iu tien de cap den van de huang cua mot doan thing d6 Ia khai niem vecto. Sau d6 ngiroi ta da dua van khai niem true, tire Ia da dinh huang mot dirong thing ,6. nao d6 bang each dua van d6 mot di~m 0, goi la di~m g6e cua mot vecto dan vi 1. Tia Ox cling huang vci 1 diroc

---+

goi Ia tia dirong va do d6, xac dinh diroc huang dirong cua true ,6.. Tia Ox' Ia

tia d6i cua tia Ox, ngiroc huang vci 1 diroc goi Ia tia am va xac dinh huang am

---+

(hay huang nghich) cua true ,6.. Tiep d6 ngiroi ta dua van khai niem dQ dai dai

s6 cua mot vecto tren mot true d6 Ia mot s6 (s6 dai s6) rna nhan vci vecto dan vi

---+

1 cua true thi eho ta mot vecto bang vecto d6 (AB = AB. tt, AB goi la dQ dai

---+ ---+

dai s6 cua AB tren true ,6., trong d6 AB cling Ia ky hieu dQ dai thing thirong

cua doan thing AB).

D6i tuong hinh hoc thir hai rna cluing ta da de cap den 6 cac muc 2 va 3 cling e6 lien quan den van de hurmg, d6 Ia hai phep doi hinh phang Ia tfch cua hai phep d6i xirng true, phep tinh tien va phep quay xung quanh mot diem trong mat phang. Phep tinh tien g:in lien vci khai niem vecto tinh tien Ia mot d6i nrong hinh hoc (doan thing) e6 huang. Con phep quay phang xung quanh mot diem (goi la tam quay) thi gan voi khai niem g6e Iuong giac goi Ia g6e quay; d6 Ia khai niem g6e dinh huang cua hai vecto e6 diem g6e chung la tam quay. Trong tnrong hop nay, ngiroi ta bat buoc phai sir dung den khai niem g6e dinh huang cua hai vecto rna dQ 16'n dai s6 cua n6 (g6e quay) diroc xac dinh sai khac mot boi cua 3600 hay 2n tuy thea dan vi g6e la dQ hay radian. R5 rang rang neu sir dung each thong thirong thi mot diem M bat ky (khong trung voi tam quay 0) trong mat phang se irng hai diem M{ va M~ phan biet cling thoa man cac dieu kien:

OM~ = OM~ = OM, va LMOM~ = LMOM~ = a > 0, eho tnroc (0 < a =l2n)

Ngoai g6e dinh huang cua hai vecto diroc sir dung den trong phep quay ngiroi ta cling can den va sir dung g6e dinh huang cua hai dirong thing. Ching han, neu

---+ ---+

phep quay Q ( 0, a) tam 0 g6e a (mod 2n) bien vecto A B thanh vecto A' B', thi

---+ ---+ ---+ ---+

(AB, A' B') = a mod 2n. Neu xem AB va A' B', la hai true thi g6e dinh huang

---+ ---+

giira hai true AB va A' B' cling bang g6e giira hai vecto d6, va xac dinh sai khac

---+ ---+

2kn. Nhirng phep quay Q(O,a) da bien AB thanh A'B' thi cling bien dtrong

thing (AB) thanh dirong thing (A' B') sao eho

---+ ---+

(AB, A' B') = a + 2kn, va ((AB), (A' B')) = a + k-n , k E Z

18

trong hie g6c thong thirong cua hai dirong thing AB va A' B', thea dinh nghia, thi bang 0: hoac 7r - 0: neu 0 < 0: < 7r /2 hoac 7r /2 < 0: < it .

Le tir nhien, d~ thuan tien cho viec nghien crru tinh chat cua cac hinh hinh h9C trong mat phang, ngiroi ta cling con dira van khai niem mat phang dinh huang. Chung ta khong c6 dieu kien xay dung chat che va that chinh xac toan h9C cac khai niem hinh h9C lien quan den van d~ huang nhir mat phang dinh huang, g6c dinh huang cua hai vecto hay cua hai true, g6c dinh huang cua hai dirong thing, tam giac dinh huang, v.v ... trong mat phang dinh huang. Chung ta chi lam r5 nhtrng khai niem mot each tnrc quan hinh h9C, thong qua viec quan sat cu the mot each tnrc giac su chuyen dong cua mot diem tren mot dirong tron, hay tren mot tam giac, mot da giac dan (cling tire Ia mot dtrong gap klnic khep kin khong c6 diem nr cat) hay tren mot dirong cong kin.

M9t diem M chuyen dong tren mot duong tron (v) c6 the di chuyen thea hai huang khac nhau. M9t each tnrc quan, cluing ta chap nhan Slf viec quan sat nay tren moi duong tron (v) cua mat phang p, ton tai hai huang di. Ta c6 dinh nghia sau day.

Djnh nghia 1. Dinh huang dirong tron (v) Ia chon mot trong hai huang di tren (v).

Khi da cho hai dirong tron dinh huang (v) va (Vi) tren mat phang P, cluing ta c6 kha nang nhan biet rang (Vi) diroc dinh huang thea \cling mot huang" voi ( v) hay thea huang ngiroc lai.

Hinh 10

M9t mra dirong thing (cling goi Ia mot tia) Oy c6 goc trung vci tam 0 cua mot dirong tron (v) quay xung quan diem goc 0 cua n6 thea hai huang khac nhau nhu huang chuyen dong cua diem M, 6 d6 tia goc 0 z cat (v) tren dirong tron (v) (Hinh 10).

Dinh nghia 2. Dinh huang mat phang P Ia chon cling mot huang di tren tat cit cac dirong tron cua mat phang, huang nay goi Ia huang thuan. Thirong thi

19

nguoi ta quy tree huang thuan cua phep quay trong mat phang Ia huang nrong irng voi huang ngiroc chieu quay cua kim dong ho (Hinh 10). Huang ngiroc lai goi la huang nghich,

Huang thuan con diroc goi Ia huang dirong hay huang Iuong giac, huang nghich con diroc goi Ia huang am, va ky hieu dirong hay am boi dau + hay -.

Nhu vay, mat phang P goi Ia da dUQ'C dinh huang. Chien di thuan tren moi dtrong cong kin khong nr cat va dac biet tren moi da giac dan hoac moi dirong tron cua mat phang irng voi chieu thuan trong mat phang.

Dinh nghia 3. Ta goi Ia duong tron Iuong giac, mot duong tron cong ban kinh bang dan vi dai (dtrong trong dan vi) va dUQ'C dinh ly thea huang thuan.

Dinh nghia 4. Neu chieu di tren cac canh AB, BC va C A cua mot tam giac ABC trong mat phang dinh huang P nrong irng voi chieu thuan cua mat phang P thi ta noi tam giac ABC co huang duong, hay diroc dinh huang duong (Hinh 11a). Trai lai neu chieu di tren tam giac do irng voi chieu nghich cua P thi tam giac ABC co huang am, hay diroc dinh huang am (Hinh 11b). Nhu vay, hai tam giac co cung ba dinh A, B va C thi tam giac ABC va BAC Ia hai tam giac trong mat phang dinh huang P co huang ngiroc nhau. Ngoai ra cling d~ thay rang trong P thi huang cua mot tam giac dinh huang ABC trung voi huang cua cac goc dinh huang ABC trung voi huang cua cac goc dinh huang (AB, AC), (BC, BA) va (CA,CB)

Til cac nhan xet tren ta cling thay rang huang cua mot tam giac khong thay d6i khi ta hoan vi vong quanh cac dinh cua no (phu hop voi Dinh nghia 4 a tren). Cling vi Ie do rna ngiroi ta ira dung dinh nghia sau

Dinh nghia 4'. MQ bo ba diem sap thir tir {A, B, C} cua mat phang dinh huang P diroc goi Ia mot tam giac dinh huang (hay co huang) cua mat phang do va ky hieu Ia ,6,ABC nhir thong thirong (nhu clni thfch den each viet) thea thtr nr cac dinh.

Hinh 11

Khi ,6,ABC da diroc dinh huang thi dien tfch cua no mot each nr nhien tro

20

thanh dien tfch dai so va ta co dinh nghia sau

Djnh nghia 5. Ta goi la dien tfch dai so cua tam giac dinh huang ABC (trong mat phang dinh huang P), mot so dai so bang dien tfch cua ,6,ABC (khong dinh hurmg) v~ gia tri tuyet ctoi, nhung lay dau + hay - tuy thea tam giac ABC co huang thuan hay huang nghich va ky hieu Ia s(ABC) hay van tit Ia ABC. V<:ty Ia s(ABC) hay ABC = ±s(ABC).

2. Mc)t so van de b6 sung lien quan den nhung doi tuong hinh hoc co huong va so do dai so cua cluing

Nhu 6 muc 2, cac Dinh ly 5 va 5, cta: chi ra rang tfch cua hai phep ctoi xtrng true (trong mat phang) co true cat nhau Ia mot phep quay phang xung quanh giao diem o cua hai true do. B6i vay, ta ciing co the lay noi dung Dinh ly 5 lam dinh nghia cua phep quay nhir mot so tac gia cta: lam. Tuy nhien, neu cta: dira vao khai niem mat phang dinh huang goc dinh huang cua hai tia cung goc hoac goc giira hai vecto thi ta co the dira ra dinh nghia phep quay mot each tnrc quan hinh hoc han nhu sau.

Dinli nghia phep quay pluing, Trong mat phang P gia sir cta: diroc dinh huang cho mot diem 0 co dinh va mot goc dinh huang (goc Iuong giac) p, xac dinh sai khac 2k1r (k E Z). Ta goi la phep quay tam 0 vci goc quay cp, trong mat phang P, ky hieu la Q(O, cp) hay Qo,cp, mot phep bien hinh diem 0 thanh chinh no va bien m6i diem M =I- 0 thuoc P thanh diem M', cung thuoc P, xac dinh boi cac he tlnrc (Hinh 12)

---+ ---+

(OM, OM') = ip; va OM' = OM

Khi diem M vach nen mot hinh H thi diem M', tuong irng cua no vach len hinh nrong img (hinh bien ct6i hay anh) cua H trong phep quay do, ky hieu H' = Qo,cp(H).

Hinh 12

Theo dinh nghia tren, neu cp va cp' la hai goc dinh huang rna so do dai so cua cluing han kern nhau mot boi so cua 27r thi Q(O, cp) va Q(O, cp') Ia mot. B6i vay,

21

nguoi ta thirong chon rp sao cho -7r :s; rp :s; 7r hay doi khi 0 :s; rp :s; 27r).

Cling theo Dinh nghia tren thi Q ( 0, rp = 0) la phep dong nhat, va Q ( 0 , 7r ) hoac Q(O, -7r) Ia phep d6i xtrng '])(0) quan tam O.

Nhu cluing ta da biet giira ba diem A, B, C bat ky tren mot true ,6. ta c6 mot

---+ ---+ ---+

he tlnrc giira cac dQ dai dai s6 cua ba vecto AB, BC va C A tren true do nhu sau

goi la he tlnrc Salo (ChasIes)

AB + BC = CA hay Ia AB + BC + CA = 0, (VA, B, C E 6)

D6i voi s6 do (dQ 100) dai s6 cua cac g6c dinh lnrong giira hai phirong trinh dinh lnrong (bao gom cac g6c giira hai tia, giira hai vecto, giira hai true) va g6c dinh htrong giira hai duong thing trong mat phang dinh hurmg, ta cling c6 nhtmg he thirc nrong tu.

--+

H~ thiic Said (Chasles) v€ goc dinh. huang). V 6i bat ctr ba vecto (J;, b, ct nao,

hoac ba dirong thing a, b, c nao trong mat phang, ta deu c6 nhtmg he tlnrc sau

--+ --+ --+ --+ --+ --+

(a,b)+(b,c)=(a,c)+2k7r, kEZ

--+--+ --+--+

( a, b ) = - ( b , a) (mod 27r)

(a, b) + (b, c) = (a, c) + k7r (a, b) = -(b, a) (mod 7r)

D6i voi dien tfch dai so, ta cling c6 he tlnrc tuong tir (hay tir kiem nghiem)

H~ t.htrc Salo v~ di~n tieh dai s6 cua tam giac djnh hurrng. V 6i moi diem M thuoc map phang dinh htrong cua tam giac dinh huring ABC cho tnroc, ta c6 he thirc sau day ve dien tfch dai s6 cua tam giac.

s(M BC) + s(MCA) + s(M AB) = s(ABC), V M E mp(ABC)

hay ta

MBC + MCA+MAB = MBC +AMC +ABM = ABC, VM E mp(ABC)

Clui thich. Khi sir dung g6c dinh huring giira hai vecto va g6c dinh htrong giira hai dirong thing trong mat phang, can hru y dac biet den m6i quan he giira g6c 6 tam va g6c n6i tiep cling chan mot cung tren dirong tron. Trong mot dirong tron (v), mot g6c 6 tam va mot g6c n6i tiep dinh lnrong diroc goi la lien ket khi cluing chan cling mot cung dinh luning. Neu d~ y rang s6 do dai s6 cua cung

~ ---+ ---+

dinh lnrong AB cua dirong tron (0) bang s6 do dai s6 cua g6c 6 tam (OA, OA)

thi khi so sanh cac dai s6 cua g6c 6 tam va g6c noi tiep dinh lnrong lien ket, ta

22

co ket qua sau (d~ nghi nr cluing minh).

Djnh ly 10. Khi ba diem A, B, M cling thuoc mot dtrong tron tam ° thi ta co he tlnrc

1 ---+ ---+

(MA,MB) = 2(OA,OB) +ks:

Dinh ly tren co the diroc phat bieu lai nhu sau: MQt goc dinh huang (cua hai dtrong thing) noi tiep mot dirong tron thi bang mot mra goc a tam dinh huang (cua hai vecto) lien ket.

---+ ---+

Nhir vay neu (MA,MB) = a (mod 7r) thi (OA,OB) = AB = 20:, (mod 27r).

Hinh 13

Sau day Ia mot so he qua suy ra t11 dinh ly co han (Dinh ly 10) tren day can thiet cho viec irng dung vao giai toan (toan cluing minh cling nhu quy tfch)

Dinh ly 11. Muon cho bon diem A, B, C, D (khong thing hang) cling thuoc mot dirong tron, dieu kien can va du Ia

(CA, CB) = (DA, DB) = rp =I 0; (mod 7r)

Chuy. Neu (CA, CB) = (DA, DB) = 0 thi bon di~m A, B, C, D thing hang

Dinh ly 12.

a) {MI(MA,MB) = a (mod 7r)} Ia mot dirong tron di qua A va B, goi Ia dirong tron chua goc dinh huang cua cac dirong thing di qua A va B

---+ ---+

b) {MI (M A, M B) = a (mod 2)7r la mot cung tron Ao:B co hai d:iu nut la A

va B, goi la cung chua goc dinh huang 0: cua hai tia di qua A va B.

c) Co sa cua su phan Ioai cac phep doi hinh phang.

23

Dinh ly 13. Phep d6i xirng true trong mat phang bien mot tam giac thanh mot tam giac bang n6 nhung ngiroc huang voi n6.

Chang minh. Ta chi can cluing minh tinh chat sau day Ia du: phep d6i xirng - true bien mot g6c dinh huang (gifra hai vecto, hoac giira hai duong thing) trong mat phang dinh huang thanh g6c d6i voi n6 (sai khac 2k7r, hoac Ia k7r k E Z)

-----+ -----+ -----+-----+

Xet g6c (AB, AC) va g6c (A' B', A' C') anh cua n6 trong phep d6i xirng true

-----+ --+ -----+ --+ -----+ -----+ -----+ -----+ --+

'D(~). Dat AB = b, AC = c, A' B' = b' AC' = c' , va e Ia vecto dan vi chi

phirong cua true ~.

D~ thay rang

--+--+ --+--+

(e, b') = -( e, b),

(i)

Lai thea he tlnrc Salo ta c6

--+ --+ --+ --+ --+ --+ --+ --+ --+ --+

(b',c') = (e',c') - C~,b') va - (b, c) = (e, c) - (e, b) (ii)

Hinh 14

--+ --+ --+

Do do, tir cac ding thirc dang (i) va (ii) ta dUQ'C (b',c') = -(b, c'), nrc Ia

-----+ -----+ -----+ -----+

(A' B', A' C') = - (AB, AC). Tir day ta diroc dieu phai cluing minh.

-----+ -----+

Cu6i cung VI huang cua tam giac ABC trung vci huang cua g6c (AB, AC)

-----+ -----+

cling nhu huang cua tam giac A' B' C' trung voi huang cua g6c (A' B', A' C') ta

suy ra tam giac A' B' C' = 'D 6. (ABC) ngiroc huang voi tam giac ABC, va do do hai tam giac la phan bang nhau, tire la DA' B' C' : DABC

H~ qua. Phep d6i xirng - true Ia mot phep doi hmh, lam dao ngiroc huang cua hmh. Den day, cluing ta da c6 co sa d~ phan Ioai cac phep doi hinh (ding

24

cir) phang.

Djnh nghia 1. Cac phep doi hinh phang diroc chia lam hai loai, Ioai mot va Ioai hai tuy thea n6 bao toan huang hay dilO ngiroc huang hmh. Phep doi hinh Ioai mot ciing diroc goi la phep doi hinh thuan, hay gan gon la phep doi hinh. Phep doi hinh Ioai hai ciing goi la phep doi hinh nghich, hay phep phan chieu hoac phep doi hmh.

Djnh nghia 2. Hai hinh H va H'la anh cua nhau trong mot phep doi hinh thuan diroc goi Ia hai hinh bang nhau thuan, hay ngan gon Ia hai hinh bang nhau; ky hieu H' = H

Hai hinh H va H'la anh cua nhau trong mot phep doi hinh nghich diroc goi Ia hinh bang nhau nghich, hay phan bang nhau, ky hieuh H' : H.

d) Dang ehinh tl'ie cua ph ep doi hinh tb uan va ph ep doi hinh nghjch

Dinh ly 14. (V~ dang chinh tac cua cac phep doi hinh va phan doi hinh) M9t phep doi hinh thuan trong mat phang thi, hoac Ia phep bien hinh dong nhat, hoac Ia mot phep quay xung quanh mot diem, hoac Ia mot phep tinh tien.

M9t phep doi hinh nghich (phan doi hinh) trong mat phang thi, hoac la mot phep dO'i xirng - true, hoac Ia mot phep dO'i xirng - tnrot.

Chuy.

Phep quay thuc su (i=- I d) la phep doi hinh thuan c6 mot diem bat dong duy nhat la tam quay.

Phep dO'i xirng true Ia phep phan doi hinh c6 mot dirong thang cO' dinh gom toan nhtmg diem bat dong.

Phep tinh tien la phep doi hinh va phep doi xtrng - tnrot thuc slf la phep phan doi hinh khong c6 diem bat dong.

5 Bai tap ve s1;I' xac dinh va dang chinh tac cia phep doi hinh phfing

2.1 Chirng minh dinh Iy: Trong mat phang P cho hai doan thang bang nhau AB va A' B' the thi ton tai hai va chi phep dang cu i. : P ----+ P i = 1, 2 cua P bien A thanh A', B thanh B', trong do mot phep la doi hinh (thuan) con phep kia Ia doi hinh nghich (phan doi hinh)

25

2.2 Trong mat phang cho hai tam giac can bang nhau ABC va A' B' C' va can 6 A va A'. Tim cac phep doi hinh va phan doi hinh bien tam giac thir nhat thanh tam giac thir hai.

2.3 Cho hai tam giac deu, c6 canh bang nhau ABC va A' B' C', hay xac dinh tat ca cac phep doi hinh va phan doi hinh phang bien tam giac tlur nhat thanh tam giac thtr hai.

2.4 Tren mat phang cho hai tam giac ABC, A' B'C' nhau va cung huang nhirng khong c6 hai canh tuong irng nao song song voi nhau. Chirng minh rang c6 mot diem 0 duy nhat each deu ca ba cap diem A, A'; B, B'; va C, C'.

2.5 Trong mat phang cho hai tam giac phan bang nhau, DA' B'C' : DABC.

Chirng minh rang cac di~m chung Ao, B; va Co cua cac doan thing AA', BB', va CC' noi cac cap dinh nrong irng rna ba diem thing hang.

2.6 Chirng minh Dinh ly 8

2.7 Trong mat phang cho ba duong thing ~i' (i = 1,2,3). Cluing minh rang dieu kien can va du d~ ba dirong thing ~i' (i = 1,2,3) dong quy hoac song song la tfch f = 2)3 02)2 02)1 cling la phep doi xirng - true, trong do 'Di = 'D(~i)' i = 1,2,3

2.8 Cluing minh rang tfch cua hai phep quay Q1(01,lP1) va Q2(02,lP2) Ia mot phep quay hay phep tinh tien tuy thea tong lP1 + lP2 = ip cua mot cac g6c quay khong Ia boi cua 27T (radian) hay Ia boi cua 27T (cling tire Ia lP = 0 (mod 27T).

2.9 Chirng minh rang tfch cua mot phep quay va mot phep tinh lien (theo thir tir do hay thea tlur nr ngiroc Iai) Ia mot phep quay.

2.10 Cho ABC D va A' B' C' D' 1a hai til giac 10i cua mat phang P. Chirng minh rang neu A'B' = AB B'C' = BC C'A' = CA A'D' = AD thi

, , ,

B'D' = B D va C'D' = CD va hai til giac do bang nhau (thuan hay nghich:

A'B'C'D' = ABCD hoac A'B'C'D' : ABCD

2.11 Cho hanh hinh vuong ABC D va A' B' C' D' c6 canh bang nhau va c6 mot dinh A chung. Tim tat ca cac phep doi hinh va phan doi hinh bien hinh vuong thtr nhat thanh hinh vuong thtr hai.

2.12 Gia sir A1A2A3A4 la mot til giac (loi) noi tiep mot dirong tron (0) 09i Hi la tnrc tam cua tam giac AAkAZ voi {i, j, k, l} = {1, 2, 3,4,}. Chirng minh rang hai til giac A1A2A3A4 va H1H2H3H4 bang nhau thuan

Hudng dan. Cluing minh rang c6 mot phep doi xirng tam bien A thanh Hi (i = 1,2,3,4,) do do bien 1-l(A1A2A3A4) thanh 1-l'(H1H2H3H4).

26

§ 3 V *n dung phep dOi hinh vao viec giai mot s6 bai toan hinh hQC phang, Thi du minh hoa va hai t*p

1 Dog dung phep tinh tien va viec khao sat tinh chat ella hinh va dung hinh.

Thi du 1. Chirng minh tfnh chat sau day cua tnrc tam tam giac. Trong moi tam giac, khoang each tir moi dinh den tnrc tam gap doi khoang each tir tam dtrong tron ngoai tiep den canh dO'i dien.

Chang minh. Ta can cluing minh d(A, H) = 2d(0, BC) hay AH = 20Ao, trong do d Ia ky hieu khoang each va An Ia trung diem canh BC. Nhung ca hai doan thing AH va OAn deu vuong voi BC, han ntra lai cling huang. Boi vay, ta

-----+ -----+ -----+

nghi den sir dung vecto va bai toan tro' thanh cluing minh AH = 20 Ao = 00',

trong do 0' = 'D( 0). Tir do suy ra AH = 20Ao.

Hinh 15

That vay, VI di~m H' dO'i xirng vci tnrc tam H cua ,6,ABC qua canh BC thuoc van dirong tron ngoai tiep tam giac (Hinh 15) nen cac dirong tron (ABC) tam o va (H BC) tam 0' doi xtrng nhau qua BC thi bang nhau (c6 cling ban kfnh). Va do do, cluing lai con nrong irng vci nhau trong phep tinh tien 'J'(1i) theo vecto

-----+ -----+ -----+ -----+

1i = 00' = 20Ao. Vi T(1i) bien A thanh H nen ta c6 AH = 20Ao.

Thi du 2. Tam giac ABC noi tiep tron (0) cho tnroc c6 hai dinh A, B cO' dinh

a) Tim quy tfch tnrc tam H cua tam giac ABC.

-----+

b) Cluing minh rang voi moi vi trf cua C vecto C H Iuon diroc nhin tir A duoi

mot g6c dinh huang (cua cac dirong thing) khong d6i (Hinh 16)

Hudng ddn guii.

27

---+ ---+

a) Tnroc het thiet lap he thirc vecto C H = 20Co, trong d6 Co la trung diem

canh AB (xem Thi du 1) t11 do suy fa, {H} la dirong tron (0') diroc suy ra

---+

t11 (0) hai phep tinh tien 'YC~) thea -;z; = 20Co

---+

b) G9i B' la anh cua B trong phep tinh tien thea -;z; = 20Co, the thi B' = 'D(B)

la diem xuyen tam - d6i cua A tren dirong tron quy tfch (0') cua C. T11 ding

---+ ---+ ---+---+

tlnrc (O'B',O'H) = (OB,OC) (mod 2Jr) suy ra (AB',AH) = (AB,AC),

(mod Jr) va sau d6 ap dung he tlnrc SaID', ta diroc

1 ---+ ---+

(AC, AH) = (AB, AB') = "2(AO, AO'), khong d6i, (mod Jr)

Hinh 16

Thi du 3. Dung mot hinh thang bier d9 dai mot canh day, d9 dai hai dirong cheo va g6c (g6c nhon) giira cluing.

Hinh 17

Ph/in. tich, Gia sir hinh thang ABCD (AB II CD) da dung duoc, co canh day AB = a, hai dirong cheo AC = m, BD = ti va g6c (AC, BD) = rp :::; Jr /2. D~ lam xuat hien mot tam giac c6 mot g6e bang rp hoac Jr - rp va hai canh g6e

---+

d6 bang m va n, Ie nr nhien ta nghi den sir dung phep tinh tien thea vecto DC.

Va do do, bai toan diroc quy ve dung tam giac ACE c6 C A = m, C E = ti va LAC E = Jr - rp trong d6 E diroc suy fa t11 B trong phep tinh tien thea vecto

---+ ---+ ---+

---;: = DC (Hinh 17) nghia la BE = DC hay E la dinh thir tir cua hinh binh hanh

CDBE va rp :::; Jr /2 la g6e cua hai dirong cheo AC, BD. T11 d6 ta suy fa each

28

dung hinh thang can tim theo trinh nr sau. Tnroc het dung tam giac ACE nhir da n6i 6 tren. Sau do, tren tia [AE) dung diem B xac dinh boi AB = a. Cuoi

-----+ -----+

cling, dung diem D xac dinh boi CD = E B, la dinh thir tir cua hinh hanh C E B D.

Bi~n luijn. Bai toan c6 nghiem hinh khi va chi khi a < a', trong d6 a' = A R = Jm2 + n2 + 2mn cos cP va do d6 khi va chi khi 0 < cP < CPo = arccos a2-~:n+n2)

2 Dog dung phep doi xirng tam vao kho sat tinh chat ella hinh va dung hinh .

Thi du 4. Cluing minh rang neu mot tam giac c6 duong trung tuyen va duong phan giac phat xuat t11 cling mot dinh rna trung nhau thi tam giac d6 Ia can.

Hinh 18

That vay, gia sir tam giac c6 dirong trung tuyen AD dong thai la dirong phan giac (Hinh 18). Vi san c6 D la trung diem canh BC roi, hay B va C doi xtrng voi nhau qua diem D nen ta nghi den phep doi xtrng - tam 'D (D). Phep nay bien A thanh A' va do do, A' Ia dinh tlur nr cua hinh binh hanh BACA' tam D. B6i vay AC = BA' va LA; = LA2 = LA1; suy ra tam giac BAA' can a B va do d6 BA' = BA t11 d6 ta diroc AB = AC va tam giac ABC can 6 A.

Thi du 5. (Van dung giai toan dung hinh) Qua giao diem P cua hai dtrong tron cat nhau (01) va (02) da cho hay ke mot cat tuyen ~ sao cho n6 dinh ra tren cac dirong tron d6 hai day cung bang nhau.

Ph/in. tich, Gia su da dung diroc cat tuyen ~ di qua P cat (01) a A va (02) 6 B sao cho AP = P B, cling nrc la A va B doi xirng nhau qua P. Ta nghi den viec sir dung phep doi xtrng - tam P d~ tim ra tinh chat cua cat tuyen ~ can dung. Nhu vay la da c6 A = 'Dp(B) va BE (02) da cho. Suy ra diem A can xac dinh phi thuoc duong tron (02) doi xirng voi dirong tron (02) qua diem P. B6i vay, cat tuyen ~ can dung di qua P va giao di~m thtr hai A cua hai dirong tron (01) va (02). (Hinh 19). T11 d6 suy ra each dung ~ theo trinh nr sau. Tnroc het dung 0;, = 'D p ( O2), roi dung dirong tron tam 0; di qua P, n6 cat lai (01) 6 A, khac

29

vci P. Sau cung, dung giao di~m thtr hai B cua tia [B P) va dirong tron (02). D~ thay rang AP = P B va bai toan Iuon c6 nghiem duy nhar.

Hinh 19

3 Nhfrng dieD din hru y khi van dung phung phap bien hinh vao viec giai toan hinh hoc

a) Phuong phap bie'n hinh trong vi~c giai toan hinh hoc,

Nhimg bai toan hinh h9C trinh bay dtrrri dang nhtmg thf du minh hoa sap d:;tt vao hai muc 1 va 2 a tren d~ cap nrong doi day du cac dang toan hinh h9C, chi thieu dang tinh toan cac dai Iuong hinh h9C. Dang cluing minh tinh chat hinh h9C cua cac hinh diroc d~ cap den trong cac thi du 1, thi du 2 phan b) va thi dl;!4. Thi du 2 phan a) d~ cap den dang toan quy tfch (tim tap hop diem). Toan dung hinh d~ cap den trong cac thi du 3 va thi du 5. Tuy nhien dieu muon n6i a day Ia ca nam bai toan trinh bay tren day deu c6 mot dac di~m chung v~ phirong phap giai . Phep tinh tien da diroc sir dung d~ giai cac bai toan neu trong ba thi du dau va phep doi xtrng - tarn da diroc van dung vao hai thi du 4 va 5. Boi vay c6 the goi ten phirong phap giai cac bai toan do Ia phirong phap tinh tien va phirong phap doi xirng (cu the han la doi xirng - tarn) gan voi ten goi cua phep bien hinh diroc su dung den.

Nhimg bai toan nay deu c6 the giai diroc ve co ban chi can kien tlnrc hinh h9C thuoc cac lap trung h9C co sa, nhung da diroc cluing ta giai lai theo quan diem bien hmh. Nhu vay la trong viec khao sat tinh chat cua cac hinh h9C va n6i day du han Ia trong viec giai toan hinh h9C, ngoai phirong phap tong hop, phirong phap toa dQ va phirong phap vecto rna cluing ta da bier va da su dung, con c6 phirong phap bien hmh. D6 Ia phirong phap van dung tinh chat cua cac phep bien hinh diem thirong gap nhu doi hmh, dong dang v.v ... vao viec khao sat tinh chat hinh h9C cua cac hmh, tinh toan cac dai Iuong hinh h9C, tim tap hop diem (quy tfch) va vao viec giai ca toan dung hmh.

30

b) each nhan bie't lop cac bai toan hoc co kha nang giai dtigc bang phtidng phap bie'n hinh.

v ~ mat nguyen tac, bat ky bai toan hinh h9C nao cling c6 the giai bang phirong phap toa dQ (cling con goi la phirong phap dai so). Tuy nhien, nhieu bai toan hinh h9C giai bang phirong phap tong hop thong thirong lai di den ket qua nhanh ch6ng va gon gang cling nhir dep han nhieu. Cling vay, nhieu bai toan hinh h9C c6 th~ giai diroc nhanh ch6ng va gon gang neu biet sir dung phirong phap vecto.

Nhu cluing ta dii thay, thirong thi mot bai toan hinh h9C c6 the giai diroc bang nhieu each khac nhau, chi it Ia mot each, phirong phap tong hop hay phirong phap toa do. Tuy nhien drrng tnroc mot bai toan moi v~ hinh h9C ta nen xem xet can than d~ Iira chon phirong phap giai thfch hop sao cho dat ket qua nhanh, gon nhat.

D~ c6 the giai mot bai toan hinh h9C bang phirong phap bien hmh, tnroc het phi nhan ra diroc dau hieu cua lap cac bai toan c6 kha nang giai diroc bang phirong phap nay. Duong nhien, khong phai bai toan nao cling giai diroc bang phirong phap bien hmh. MQt diu hoi diroc d(lt ra la: lam the nao d~ nhan biet diroc mot bai toan hinh h9C nao do co kha nang giai diroc bang phirong phap bien hinh?

Muon vay, ta hay tro lai phan tfch tirng thi du dii diroc chi ra a tren. Co sa cua viec c6 the sir dung phep bien hinh nay no vao moi bai toan (thi du) dii diroc chi ra. Thirong thi trong dtr kien cua bai toan va (hoac) trong tinh chat cua hinh doi hoi phai thiet lap (chirng minh) hoac trong dieu kien doi hoi a hinh can dung da xuat hien nhtmg yeu to co moi lien he dang clni y den mot phep bien hinh cu th~ nao d6. Ching han, trong thi du khi thiet lap mot tinh chat cua tnrc tam tam giac, sau khi sir dung mot tinh chat khac dii biet cua tnrc tam la cac diem doi xirng cua n6 qua moi canh deu nam trong mot dirong tron ngoai tiep tam giac thi urc khac lam xuat hien hai duong tron bang nhau Ia (ABC) tam (0) va (H BC) tam (0') trong do H la tnrc tam tam giac ABC con 0' doi xirng v6'i 0 qua BC. Hai dtrong tron nay vira Ia nrong irng v6'i nhau khong nhirng trong phep doi xirng true 'D(BC) va trong phep doi xirng - tam 'D(Ao) trong do Ao la chung diem chung cua BC va 00' rna con nrong img voi nhau ca trong phep tinh tien 'Y(~) theo

---+

vecto ~ = 00'. Chfnh moi lien he nay cua hai dirong tron bang nhau dii giiip

---+ ---+ ---+ ---+

ra thiet lap diroc he thirc vecto AH = 00' = 20Ao (v~ phep tinh tien theo 00'

khong nhtmg bien 0 thanh 0' va (0) thanh (0'), trong do di~m A tren (0) bien thanh di~m H tren (0').

T6m Iai, muon nhan biet diroc mot bai toan hinh h9C nao do c6 kha nang giai diroc bang phirong phap bien hinh (cu the la phirong phap doi hmh, bao gam tinh tien, doi xirng tam, doi xirng - true, quay hoac phirong phap dang dang v.v ... ),

31

tnroc he"t cluing ta phai xem xet, phan tfch noi dung bai toan d~ tim ra yeu to nao trong do c6 moi lien he dang chii y den mot phep bien hinh cu the nao d6. Sau d6, van dung cac tinh chat cua phep bien hinh nay rna tim ra lai giai hay dap so cua bai toan diroc xet.

Dac biet dang chii y Ia phirong phap bien hinh ciing thirong gap va diroc sir dung trong viec giai mot bai toan quy tfch va dung hmh, nhat Ia toan dung hinh . Theo quan diem cua ly thuyet tap hop thi hinh la mot tap hop diem nao d6. B6i vay, viec sir dung mot hinh h9C nao do lai quy ve dung mot so diem htru han d~ d~ xac dinh, ciing c6 nghia la du d~ tao nen hinh d6. Trong mat phang thong thirong mot diem diroc xac dinh boi giao cua hai duong, trong do c6 duong thing va dirong conic rna dirong tron Ia mot elip dac biet. Trong hai dirong dung d~ xac dinh diem phai dung Ia mot trong cac giao diem cua cluing, thirong thi mot dirong da c6 san trong gitr kien cua bai toan con dirong thir hai la quy tfch cua nhtmg diem c6 mot tinh chat dac tnrng hinh h9C nao do va diroc suy ra tir mot dtrong da cho trong dir kien cua bai toan boi mot phep bien hinh nao d6. Phep bien hinh nay diroc phat hien d~ sir dung nho viec phan tfch cu th~ noi dung bai toan hinh h9C diroc d::;tt ra nhu da noi a tren. Cac thf du 3 va 5 trinh bay a tren da minh hoa cho nhtmg nhan xet chung vira neu xoay quanh viec sir dung phirong phap bien hinh vao viec giai toan dung hmh. B6i vay, hay tro lai cac thi du 3 va 5 nay, ta se thay sang to han trong viec van dung ve sau.

4 Dog dung phep doi xung - true vao viec giai toan hinh hoc phfiog

Thi du 6. Gia sir P la mot di~m nam trong tam giac ABC sao cho LP AC = 10°, LPCA = 20°, LPAB = 30° va LABC = 40°. Hay xac dinh d¢ 100 g6c LBPC.

Hinh 20

Nluin. xet. Day Ia mot bai toan thuoc Ioai tinh toan cac dai Iuong hinh h9C, cu th~ Ia tinh d¢ 100 (so do) cua mot g6c giira hai tia. Bai toan nay VI the co th~ c6 nhieu lai giai khac nhau, dac biet la huang tinh toan, dua tren each van dung dinh ly sin va eosin. Tuy nhien, doi voi tnrong hop bai toan nay, tir dac diem cua

32

tam giac ABC cluing ta lai nghi den viec Slr dung mot phep bien hmh. D6 chinh la phep d6i xirng - true.

That vay, theo gia thiet cua bai toan thi, r5 rang la tam giac ABC can 6 C, c6 g6c 6 dinh LC = 100° (VI hai g6c LA va LB moi g6c 40°). Do do, dirong cao ha tir C trung voi trung tnrc t[AB] Ia true d6i xirng cua tam giac ABC. G9i Ll = t[AB] Ll :3 C. Dung di~m Q = 'Dt,.(P), ta diroc CP = CQ va LQC B = LPC A = 20°. Tir do suy ra tam giac C PQ la deu (Hinh 20) va tinh diroc LBQP = 150°. VI vay LBQP = LBQC = 150° va DBQC' = DBQP (c.g.c). B6i the, BQ khong nhtmg Ia phan giac cua g6c LCBP rna can Ia phan giac cua g6c LCQP dong thai phep d6i xirng true 'D(BQ) bien DBQC thanh DBQP, trong do BC ---+ BP, BP ---+ BC va LBCP ---+ LBPC. VI vay, LBPC = LBCP = 20° + 60° = 80° va do do tam giac BCP can 6 B c6 g6c 6 dinh B bang 20°.

Tni lai: LBPC = 80°

Thi du 7. Trong mat phang cho hai duong thing d1 va d2 ceit nhau 6 0 va mot di~m P cO' dinh nam ngoai d1, d2. M9t dirong thing Ll quay xung quanh P cat d1 6 A va d2 6 B. Cac dirong thing Lll' Ll2 d6i xirng voi Ll liln Iuot qua d1 va d2 cat nhau 6 M. TIm hay quy tfch cua M.

Nluin. xet. Chung ta nhan thay cac yeu to' d6i xtrng - true dii xuat hien ngay trong cac dir kien cua bai toan, VI vay bai toan nay doi hoi phai Slr dung tinh chat cua phep d6i xirng - true va g6c dinh luning cua hai dirong thing (mod 7r) d~ tim quy tfch.

Hinh 21

G9i Pi la di~m d6i xirng v6'i P qua a; i = 1,2 (Hinh 21) VI P E Ll nen suy

33

fa Pi E ~i = 1) di ( ~ ), t11 d6 ta ta diroc

(~l'~) = 2(~1' dr) = 2(d1, ~), (mod Jr) (~, ~2) = 2(d2, ~2) = 2(~, d2), (mod Jr)

T11 d6 suy fa

(1)

trong d6 (d1, d2) = 5, (mod Jr). VI ~l n ~2 = M va Pi E ~i' (i = 1,2) nen (1) diroc viet lai nhu sau

(i)

Mat khac, VI Pi = 1)a;(P), (i = 1,2) va ° = d1 n d2 nen ta c6 cac ding thirc

(OP1,OP) = 2(~1,dr) = 2(dl,OP), ( ° P, ° P2) = 2 ( d2, ° P2) = 2 ( ° P, d2),

(mod Jr) (mod Jr)

(2) (3)

T11 (2) va (3), nho he thirc Salo ta diroc

(ii)

Dei chieu (i) va (ii) ta suy fa

(iii)

Ding tlnrc (iii) cluing t6 bon diem 0, PI, P2 va M = ~l n ~2 cung thuoc mot dtrong tron voi vi trf cua dirong thing ~ quay quanh diem P co dinh. Vay, ta di den ket Iuan

a) Neu dl va d2 khong vuong g6c v6i nhau a ° va do do, 0, P1 va P2 khong thing hang thi {M = ~ I n ~ 2} la dirong tron ngoai tiep tam giac ° PI P2

b) Neu dl vuong g6c voi d2 thi {M} la dirong thing di qua PI va P2

Thi du 8. TIm dirong di cua mot qua bi-a, sao cho sau khi cham hai liln vao thanh ban, n6 di t11 di~rn A den di~rn B ( Hinh 22).

34

Hinh 22

Tnroc het, d~ y rang do tac dong cua Iuc d~y ban vao qua hi-a cua nguoi choi, qua bi-a sau khi cham van thanh ban (ban bi-a c6 dang hinh chir nhat hoac vuong PQ RS) thi phan xa tro lai thea quy Iuat nhu phan xa anh sang, urc la g6c phan xa bang g6c tci tia phan xa doi xirng voi tia tci qua phap tuyen cua thanh ban tai diem cham (Hinh 22). Nhu vay, r5 rang phep doi xirng true da phat huy tac dung giiip ta giai bai toan dung hinh nay.

G9i M va N liln Iuot Ia diem cham liln thir nhat va liln thir hai cua qua bi-a voi cac thanh ban RS va S P sau khi xuat phat tir diem A (giln diem g6c cua ban) tren mat ban d~ roi tro lai diem B (giln diem g6c P cua ban) cling da diroc dinh san tren mat ban bi-a hinh chir nhat PQRS (Hinh 22). The thi, thea quy Iuat phan xa n6i tren, dirong thing M N chua tia phan xa M va cling Ia tia toi cua N phai di qua cac diem A' = V(A) va B' = V(B). Tir do suy ra each xac dinh cac vi trf M va N cuaquabi-ach~mvaocacthanhbanRSvaSP: [A'B']n{[RS],[SP]} = {M,N}

Kit ltuin, Duong gap khiic bon dot AM N R la dirong di phai cua qua bi-a sau khi cham hai liln lien tiep van cac thanh ban [RS] va [S Pl.

Chti j. Bien Iuan v~ kha nang c6 1ai giai cua bai toan phu thuoc vao vi trf cua cac diem A va B (da d::;tt san tren mat ban bi-a).

5 Dog dung cac phep quay va dOi hinh (n6i chung) vao viec giai toan

Thi du 9. Trong mat phang cho hai tam giac ABC, ADE c6 cac g6c 6 dinh chung A btl nhau, dong thai AB .L AD, AB = AD; AC .L AE, AC = AE va hai tam giac do khong co di~m chung nao khac ngoai dinh A. Chirng minh rang dtrong thing chua trung tuyen phat xuat tir dinh chung A cua tam giac nay cling chua dirong cao ha tir A cua tam giac kia.

Chang minh. That vay, thea gia thiet thi AB .L AD va AC .L AE. Dieu do goi Y cluing ta lien tuong tci phep quay g6c vuong xung quanh tam A.

35

Hinh 23

G9i M la trung di~m canh BC, H la chan dirong cao AH cua tarn giac ADE; ta cluing minh AM .L DE. Muon vay, ta thuc hien phep quay Q(A, 7r /2) (Hinh 23). Khi do, neu tarn giac ABC c6 huang thuan thi phep quay nay bien tarn giac AD E thanh tarn giac AB F do D f---7 B va E f---7 F = ']) ( C). Vi BF = Q(DE), trong do Q = Q(A, 7r /2) nen (theo tinh chat cua phep quay g6c +7r /2) thi BF .L DE va BF = DE, dong thai BF II AM va BF = 2AM (do AM la dirong trung binh cua DBCF). Tir do suy ra AM .L DE, cling nrc la (AM) = (AH). Day Ia dieu phai cluing minh.

Thi du 10. (Chung minh dinh ly Pompiu) Neu P Ia mot diem bat ky nam trong mat phang cua mot tarn giac deu ABC cho tnroc thi bao gio cling ton tai mot tarn giac T c6 dQ dai ba canh bang cac khoang each tir di~m P den cac dinh cu tarn giac deu da cho do, k~ eft tnrong hop khi T suy bien thanh doan thing, ky hieu T = T(PA, P B, PC). Ngoai ra, hay tim quy tfch cua diem P d~ T suy bien.

Nluin. xet. Dinh ly Pompiu tren day cho ta bier mot tinh chat dac tnrng cua tarn giac deu. C6 nhieu each cluing minh dinh ly Pompiu. Sau day Ia each cluing minh nho sir dung phep quay g6c 60°.

Ph/in. tich, Gia sir PA = max{PA,PB,PC}. Neu 3T(PA,PB,PC) thi at phai ton tai mot diem Q trong mat phang sao cho PQ = P B va Q A = PC (Hinh 24). The thi, hai tarn giac BAQ va BCP (da c6 BA = BC va AQ = CP) se bang nhau khi va chi khi BQ = BP va do do khi va chi khi LP BQ = 60°. SlJ phan tfch nay lam nay sinh y tuong sir dung phep quay g6c 60° xung quanh di~m B mot each he't sue nr nhien.

Hinh 24

That vay, neu tarn giac deu ABC da cho c6 huang thuan thi, phep quay Q(B, +60°) gill bat dong diem B, bien C thanh A va bien P thanh diem Q (sao cho BPQ la mot tarn giac deu va cling c6 huang thuan) va do do, bien tarn giac BCP thanh tarn giac BAQ (Hinh 24). Tir do suy ra PQ = QB = PB va

36

QA = PC. N6i khac di Ia, QAP chinh la mot tam giac T(PA, P B, PC) doi hoi, diroc sinh ra tir diem P do thuc hien phep quay Q(B, 60°). Day la dieu phai cluing minh.

Tam giac T nay diroc sinh ra boi diem P nrong img se suy bien thanh doan thing khi va chi khi P, Q, A thing hang, nghia la T (P A, P B, PC) suy bien nrong duong vci (QP, QA) = 0, (mod 180°) (1). Lai do tam giac BAQ bang tam giac BCP nen ta c6 (QB,QA) = (PB,PC) hay la (QB,QP) + (QP,QA) = (P B, PC), (mod 180°); 'lfp E mp(ABC) (2). M~t khac, do DBPQ la deu

-----+ -----+ -----+-----+

va cling huang voi DBC A deu, nen ta c6 (Q B, Q P) = (AB, AC) = 60°,

(mod 360°) (3). Doi chieu (1), (2) va (3) ta suy ra T(P A, P B, PC) = DQAP suy bien khi va chi khi (P B, PC) = (AB, AC) (mod 180°) (4). H~ thirc (4) cho ta bier {P} d~ T(P A, P B, PC) suy bien la dirong tron (ABC)

Thi d\111. CUng dung tfch cua hai phep quay)

Dung ra phia ngoai tam giac ABC (gift sir c6 huang thuan) hai hinh vuong AC I M va BC N P. Cluing minh rang neu gill hai dinh A, B co dinh va cho C chay khap mra mat phang rna (nira duong) c6 ba Ia dirong thing (AB) thi dirong thing M P Iuon di qua mot di~m co dinh. Hay xac dinh di~m d6.

Hinh 25

Lai giai. That vay, khi gill A va B co dinh, C chuyen dong se keo thea su chuyen dong cua hai diem M va P. Va ngiroc Iai, diem P chuyen dong keo thea C chuyen dong, roi M chuyen dong nho thuc hien lien tiep hai phep quay cling mot g6c ip = +1f /2 liln IUq1 xung quanh cac diem B va A. Dieu do n6i len rang M duoc suy ra tir P boi phep doi hinh V Ia tfch cua hai phep quay Q(B1f /2) va Q(A, 1f /2) thea thtr nr d6. Vay V Ia mot phep doi xirng - tam, bien P thanh M. D~ xac dinh tam 0 cua V, ta hay phan tfch m6i phep quay Q(B, 1f /2) va Q(A, 1f /2) thanh tfch cua hai phep doi xtrng - true d~ dira V ve dang chinh tac (xem bai tap 2,8), ta diroc dang phan tfch sau

V = [TI(Ax) 0 TI(AB)] 0 [TI(BA) 0 TI(BY)] = TI(Ax) 0 TI(BY) trong do Ax va By la hai true doi xtrng diroc xac dinh boi: (By, BA)

37

(AB,Ax) = 1f/4, (mod 1f) (2). G9i 0 = Ax n By, ta diroc OAB Ia mot tam giac vuong can a 0 va c6 lnrong thuan (Hinh 25). Vay 0 la mot diem co dinh thuoc mra mat phang rna (dirong) chua C va c6 be la dtrong thing AB, hoan toan diroc xac dinh boi cac ding tlnrc (2).

Kit ltuin, V = 'D(Ax) 0 'D(By) = 'D(O) bien P thanh M va do d6, doan thing M P Iuon nhan di~m 0 co dinh lam trung di~m vci moi vi trf cua C thuoc mra mat phang (rna) duong c6 be la (AB).

Thi d\112. Trong mat phang cho mot cung tron AaB. M9t tia Ax quay xung quanh A, dong thai mot dong til" M cling chuyen dong tren tia do sao h~ Ax cat cung AaB a mot diem N nao do thi Iuon Iuon ta c6 AM = BN. Tim hinh vach nen boi M khi N vach nen cung tron da: cho.

---+ ---+ ---+ ---+

Dat (NA,NB) = rp thi {N} = AaB. The thi, (BN,AM) = -rp (mod 21f),

BN = AM. Hai ding thirc nay hoan toan xac dinh mot phep doi hinh V (xem bai 2.1).

Hinh 26

D~ Y rang khi N = B thi M = A va rp =I 0, (mod 21f) nen V la phep quay.

D~ thay rang V c6 diem bat dong (tam quay) duy nhat la trung diem cua cung AaB (a do M = N = w) va do do V la phep quay Q(w, -rp) tam w, g6c quay -rp, (mod 21f). Phep quay nay bien B thanh A va A thanh A', xac dinh boi he

---+ ---+

tlnrc (wA, wA') = -rp va wA' = wA. N6 con cho ta biet A' = 'D(B) doi xirng

voi B qua (Aw) (Hinh 26).

Kit ltuin, Quy tfch cua M Ia cung tron AaA' diroc suy ra tir {N} = AaB boi phep doi xirng true 'D (Aw ).

6 Call hoi va Bai tap

A. V;ll.n du ng ph.ep tjrrh tie'n vito vi~c giai cac hili toan sau day

38

3.1 Chirng minh rang ton tai mot tam giac ABC nao do va dien tfch cua tam giac T rna cac canh bang cac dirong trung tuyen cua mot tam giac ABC nao d6 va dien tfch cua T bang 3/4 dien tfch tam giac ABC.

3.2 Hinh binh hanh ABC D co dirong cheo AC = a. Qua A ke cac dirong cao AE va AF xuong cac canh BC va CD. Tinh khoang each tir A den tnrc tam H cua tam giac AEF, cho bietg EF = b (b < a).

3.3 Trong mat phang cho mot dtrong thing ,6. va mot di~m A co dinh. MQt dtrong tron (v) c6 ban kinh r cho tnroc chuyen dong trong mat phang nhung Iuon di qua A. Tin quy tfch cac tiep diem cua cac giao tiep cua (v) c6 phirong cua duong thing ,6. da cho.

3.4 Hai diem M va N chuyen dong tren dirong thing chua canh AB cua mot

-----+ -----+

tam giac ABC sao cho M N = AB. G9i D va E Iiln hrot la hinh chieu cua

M tren (BC) va cua N tren (CA); S la trung diem cua AN va Q la tam dtrong tron (CDE). Chirng minh SQ c6 dQ dai khong dcii; tir d6 suy ra quy

-----+

tfch cua Q khi vecto M N tnrot tren dirong thing AB.

3.5 Cho duong tron (0) dirong kinh CD va mot day cung AB khong cat CD.

Tim tren dirong tron mot diem P sao cho g6c noi tiep A P B chan tren dirong kinh CD mot doan thing M N c6 dQ dai bang £ cho tnroc. Bien Iuan.

3.6 Cho hai dirong tron ngoai nhau (0) va (0') va mot dirong thing d. Hay dung mot dirong thing c6 phirong cua d cat (0) va (0') sao cho tong dQ dai cac day cung cua cluing dinh boi duong thing c6 mot dQ dai £ cho tnroc. Bi~n Iuan.

B. V;ll.n dung phep doi xiing tam vito vi~c giai cac hili toan sau day

3.7 Gia sir P Ia mot di~m bat ky nam trong mat phang cua hinh binh hanh ABCD. Qua A, B, C, D ke cac dirong thing thea thir nr song song voi C P, D P, AP, B P. Chirng minh rang cac dtrong thing vira dung dong quy tai mot diem Q nao d6 va duong thing PQ Iuon di qua mot diem co dinh khi P doi ch6 trong mat phang.

3.8 Hinh binh hanh M N PQ noi tiep hinh binh hanh ABC D sao cho M, N, P va Q Iiln hrot nam tren cac canh lien tiep AB, BC, CD va DA cua ABCD. Cluing minh rang hai hinh binh hanh M N PQ, ABC D noi, ngoai tiep nhau co cung tam doi xirng.

3.9 Hai ngiroi choi mot tro chi \ct~t dong xu len mat ban hinh chir nhat". Quy tac choi nhir sau: Dong xu diroc phep d~t van bat err ch6 trong nao, hai ngiroi Iiln Iuot d~t cac dong xu len mat ban. Ai den Iuot di rna khong th~ d~t diroc dong xu van dau thi bi thua. Chirng minh rang neu biet each choi thi nguoi dilu Iuon thang cuoc.

39

3.10 Ttr giac ABCD noi tiep dirong tron (0). Chang minh rang cac dirong thing di qua trung diem cua mot canh (hoac mot dtrong cheo) va vuong g6c voi canh doi dien (hoac dirong cheo kia) thi dong quy.

3.11 Cho dtrong thing a, dirong tron (0) va mot diem P khong nam tren a va ( 0). Tim tren a mot diem A va tren (0) mot diem B sao cho doan A B nhan P lam trung diem.

3.12 Hay ceit mot tam giac ABC cho tnroc bang ba duong thing song song voi cac canh cua tam giac d~ thu diroc mot Iuc giac (loi) ngoai tiep diroc mot dtrong tron.

3.13 Cho hai day cung khong cat nhau AB va CD cua mot dirong tron (0) va mot diem P tren day CD. Tim tren dirong tron mot diem S sao cho g6c noi tiep LAS B chan tren day CD mot doan thing M N nhan P lam trung diem.

C. V;ll.n dung ph ep dO"i xiing - true vao vi~c giai cac hili toan sau day

3.14 Mot dirong tron thtr ba (w) cat hal dirong tron dong tam 0 1iln Iuot len tren (w) a cac dl~m A, B, C, D. Chang minh rang neu A, B, 0 thing hang thi C, D, 0 cling thing hang.

3.15 Hai dirong tron bang nhau (01) va (02) cung tiep xiic trong voi dirong tron (0) 6 cac diem Al va A2• Mot diem M tuy y cua (0) diroc noi tiep voi Al va A2• Cac doan thing M A ceit (Oi) 6 cac diem B, nrong img (i = 1,2). Chang minh rang BIB2 II A1A2.

3.16 Mot diem M chuyen dong tren duong kinh AB cua mot duong tron (0).

Day cung CD di qua M cat AB va hop Val n6 mot goc 45°. Chang minh rang dai Iuong p = M C2 + M D2 khong phu thuoc van vi trf cua M tren AB (p khong d6i voi moi M tren [AB]).

3.17 Cho tam giac ABC can 6 A mot dirong thing ~ quay quanh A. G9i D la diem doi xtrng voi C qua D. Tim quy tfch giao diem M cua dirong thing BD va A.

3.18 Tinh g6c tao boi hai girong phang biet rang mot tia sang bat ky sau bon 1iln chieu van ca hal girong (moi girong dap van hal Ian) thi phan xa thea huang ngiroc lai huang ban dau.

3.19 Cho dirong thing xy va hal di~m A, B nam cung phia Val xy. Hay tim tren dtrong thing xy mot diem P sao cho g6c LAPx bang hai 1iln g6c LBPy.

3.20 Chang minh rang

a) Dien tfch cua mot tu- giac loi bat ky khong 100 han mot mra tong cua tfch d¢ dai cac canh doi dien S(ABCD) :::; ~(AB.CD + BC.DA). (*)

40

b) Dau ding thirc trong (*) dat diroc khi va chi khi trr giac noi tiep diroc mot dirong tron va hai dirong cheo vuong g6e voi nhau.

D. V;ll.n dung phep quay va doi hinh vao vi~c giai cac bai toan sau

3.21 Cho hai dirong tron bang nhau (01) va (02). Tim tat ca cac phep doi hinh thuan va nghich (doi hinh va phan doi hinh) bien duong tron nay thanh duong tron kia.

3.22 Tren dirong thing a va b cat nhau 6 mot diem P e6 hai dong tir chuyen dong thing deu voi cling mot van toe nhung khong gap nhau 6 P: M; tren a va M2 tren b. Chirng minh rang 6 bat ky thai diem nao, dirong tron ngoai tiep tam giac MlM2P Iuon di qua mot diem co' dinh 0 nao d6, khac P.

3.23 Chirng minh rang hai dirong tron bang nhau va cat nhau a hai di~m thi nrong irng voi nhau trong mot phep quay rna tam quay la mot trong hai giao diem va cac dirong thing noi cac cap diem tuong irng tren hai dirong tron thi dong quy 6 giao diem thir hai.

3.24 Gia su P Ia mot diem nam trong hinh vuong AlA2A3A4. Qua Al dung dtrong thing Alx vuong g6e vci A2P, qua A2 dung A2y vuong g6e vci A3P qua A3 dung A3Z vuong g6e voi A4P roi qua A4 dung A4t vuong g6e voi AlP. Chirng minh rang bon dirong thing Alx, A2y, A3Z va A4t vira dung dong quy 6 mot diem Q nao d6.

3.25 Cho ba diem A, C, B phan biet va thing hang thea tlur nr d6. Dl!I1g hai tam giac deu BCM va CAN nam ve cling mot phia d6i vci dirong thing AB. G9i D va E liln Iuot la trung diem cua B N va AM. Chirng minh rang CD E Ia mot tam giac deu.

3.26 Lay cac canh cua mot tam giac ABC bat ky lam day, dung ra phia ngoai tam giac ABC ba tam giac deu BCA', CAB'va ABC'. Chirng minh rang cac tam Ao, Bo, Co cua ba tam giac deu vira dung Ia cac dinh cua mot tam giac deu (Bai toan Napoleon).

3.27 Cho mot tam giac deu ABC. Tim quy tfch nhtrng diem M trong mat phang sao eho M A, M B va M C Ia d¢ dai cac canh cua mot tam giac vuong nao d6.

Chuong 2

Cac phep dong dang phang

§ 4 Sir X3C dinh va dang chinh tac cua mot phep dong dang phang

1 Dai cirong ve cac phep dong dang phfing

a) Djnh nghia ph ep d6ng dung

Mot phep bien hinh f : P ---+ P diroc goi Ia mot phep dong dang cua mat phang hay van tit Ia phep dong dang phang, ky hieu Ia Z, neu voi bat ell hai di~m M, N nao cua P va cac anh M' = f(M) va N' = f(N) cua cluing ta deu e6 M' N' = kM N, trong do k la mot so duong xac dinh. So k diroc goi la ty so hay he so cua phep dong dang, hay n6i gon han la ty so (hay he so) dong dang. Phep dong dang ty so k diroc ky hieu boi Z ( k ).

N6i mot each ngan gon, phep dong dang phang ty so k la phep bien hinh cua mat phang, nhan khoang each giira bat ell hai di~m nao cua n6 vci cung mot so Iuong k xac dinh eho tnroc.

R5 rang la khi k = 1, phep dong dang tro thanh phep doi hmh, nghia la Z (1) = V va do do, doi hinh la tnrong hop dac biet cua dong dang. Tir dinh nghia cua phep dong dang, d~ dang suy ra rang phep dao ngiroc cua phep dong dang ti so k la phep dong dang ti so1/k ta viet Z-l(k) = Z(1/k) hay Z(k) oZ(1/k) = Id.

Tich cua hai phep dong dang e6 cac ti so k1, k2 Ia phep dong dang vci ti so k = k1.k2' nghia la, Z2(k2) 0 Zl (kd = Z12(k1k2), nhung Zl (kd 0 Z2(k2) = Z21 (k2 k1). Phep bien hinh dong nhat I d la mot phep dong dang.

b) Cac tinh chfit cua ph ep d6ng dang

41

42

Cling t11 dinh nghia cua phep dong dang, ta d~ dang suy ra cac tfnh chat sau day cua phep dong dang, trong do co nhtmg tinh chat cua phep doi hmh.

Dinh ly 15. Phep dong dang bao toan Sl)' thing hang cua ba diem va thir tir cua cluing tren dirong thing chua ba diem do. Cu the la: Phep dong dang bien ba diem A, B, C thing hang thea tlur nr do thanh ba diem A', B', C' thing hang cling thea thtr nr do.

H~ qua 1. Phep dong dang bien dtrong thing thanh mot dirong thing bien mot tia thanh mot tia, bien mot doan thing thanh mot doan thing co d9 dai diroc nhan len voi he s6 (ty so) dong dang (A' B' = kAB, \f{ A, B})

H~ qua 2. Phep dong dang bien mot tam giac thanh mot tam giac dong dang vci no; bien mot goc thanh mot goc bang no; bien mot dirong tron thanh mot dtrong tron, trong do tam bien thanh tam con ban kinh diroc nhan len voi he s6 (ti so) dong dang (R' = kR)

c) Phan loai cac phep d6ng dang

Cling gi6ng nhu phep doi hmh, phep dong dang ehia lam hai loai, Ioai mot va Ioai hai tuy thea no bao toan huang hay dao ngiroc huang cua hmh. Ta goi Ia phep d8ng dang thuan, hay van tat Ia phep dong dang, mot phep dong dang phang bao toan huang cua hmh. Ta goi phep d8ng dang nghicli hay con goi Ia dong dang girong, hoac phan dong dang, mot phep dong dang phang dao ngiroc huang cua hmh.

Chti thich, Phep dong dang tuy khong bao toan khoang each giira hai di~m nhirng gi6ng nhu phep doi hmh, phep dong dang bao toan goc (noi dung ra la bao toan d9 100 thong thirong cua goc, bao gam goc giira hai tia, giira hai vecto, giira hai dirong thangj.Vi the ngiroi ta con noi phep dong dang la mot phep bien hinh bao giac. Noi mot each chi tiet han thi: phep dong dang phang thuan con bao toan ca huang cua goc, VI the ta noi phep dong dang thuan bao toan d9 16'n dai s6 cua goc dinh huang (gitra hai tia, giira hai dirong thing). Phep dong dang nghich chi bao toan d9 100 s6 hoc (cling nrc d9 100 thong thirong) cua goc dinh huang (gitra hai tia, giira hai dirong thing) nhirng lam dao huang g6e, urc la bien goc dirong (do 100 dirong) thanh goc am (do 100 am) va ngiroc lai.

d) Kha] ni~m v~ hai hinh d6ng dang

Dinh. nghia. Hai hinh H va H goi Ia dong dang voi nhau neu co mot phep dong dang Z bien hinh nay thanh hinh kia: Z(H) = H'.

Neu phep dong dang Z bien hinh H thanh H', thi phep dong dang dao ngiroc

43

cua Z bien H' thanh H. Ky hieu hai hinh dong dang hai '""', chang han DA' B' C' '"'" DABC.

Hinh 27

Clui thich. Vi c6 hai Ioai phep dong dang, thuan va nghich nen d~ phan biet r5 hai hinh nao d6 Ia dong dang thuan hay dong dang nghich neu thay can thiet.

Sau day, cluing ta d~ cap den mot phep dong dang dac biet, d6 la phep vi tu.

2 Phep vi nr

(I)

a) Dinh nghia.

Trong mat phang cho mot di~m 0 co dinh va k Ia mot so thuc khac khong cho tnroc. Phep bien hinh cua mat phang bien m6i diem M thanh diem M' sao cho

-----+ -----+

(Hinh 28). OM' = kOM (1) diroc goi Ia mot phep vi tir tam 0, he so (ty so) k

va ky hieu la vj hay V (0, k). Diem 0 goi la tam vi nr va k goi la he so hay ti so vi tu.

Hinh 28

M9t phep vi nr hoan toan diroc xac dinh neu cho biet tam 0 va ty so k. Neu c6 ding tlnrc (1) thi ta n6i rang M' la anh hay diem tuong ling cua diem M qua phep vi nr V (0, k) hoac ngiroi ta ciing goi M' Ia hinh vi nr cua diem M. Neu phep vi nr V ( 0, k) bien mot hinh H thanh mot hinh H' (gam cac anh M' cua tat ca cac di~m M thuoc hinh H) thi ta cung n6i H' Ia hinh vi nr cua hinh H' hay H va H' Ia hai hinh vi nr voi nhau.

(l)Trong muc nay cluing ta chi nhac Iai din nghia va cac tfnh chat cua phep vi tv rna khong cluing rninh va c6 b6 sung mot vai tfnh chat can thiet cho viec trinh bay va giai toan tiep thea

44

Mot phep bien hinh di~rn cua mat phang rna cluing ta da gap trong Thi du 2 cua Bai 1 (muc 3) cho ta thi du d:iu tien ve mot phep vi nr phang. That vay, hay tro lai voi thi du nay, cluing ta thay ngay rang quy tac f : M f---7 M' tir P ----+ P chi ra trong thi du do tuong dirong voi quy tac 1', trong do moi diem M bat ky cua P ta xac dinh diroc duy nhat diem M' E P thoa man he thirc vecto

---+ 1 ---+

GM'=--GM 2

(2)

trong do G la trong tarn cua tarn giac ABC d~ cap den trong thi du do. Vi vay, phep bien hinh f trinh bay trong Thi du 2 cua bai tap 1 (muc 3) do chinh la phep vi tir V( G, -1/2) tarn G, ti s6 k = -1/2.

Ngoai ra, c6 hai tnrong hop dac biet, dang chii y cua phep vi tu, ling voi hai

---+ ---+

gia tri cua he s6 k Ia k = ±1. Neu k = +1, khi do OM' = OM('l/M). Vay

trong tnrong hop nay, phep vi nr ti s6 1 la phep dong nhar. Neu k = -1, thi

---+ ---+

OM' = OM, ('1/ M) urc M' doi xtrng voi M qua diem O. V ~y trong tnrong hop

nay phep vi tir ti s6 -1 Ia phep d6i xtrng qua tarn O.

b) Cac tinh chfit cua ph ep V! tv

D!nh ly 16. Phep vi nr ti s6 k Ia mot phep dong dang ty s6lkl, trong do neu phep vi nr V(O, k) bien M thanh M', N thanh N' thi M'N' = kMN.

Clui thich,

Hinh 29

---+

(i) Nguoi ta con n6i, phep vi nr ti s6 k bien mot vecto ~ thanh vecto v' bang

---+

k 1:in vecto 1!, nghia la v' = k 1! hoac n6i gon han la phep vi nr ty s6 k

nhan mot vecto len k Ian.

(ii) Vi phep vi nr Ia mot phep dong dang dac biet nen phep vi nr c6 moi tinh chat cua phep dong dang. Tuy nhien, phep vi tir con c6 nhtrng tinh chat dac tnrng rieng cua n6.

45

Dinh ly 17. Trong mot phep vi tu, tam vi nr Ia di~m bat dong duy nhat va mot dirong thing khong di qua tam vi nr bien thanh mot dirong thing song song voi no. Chum dtrong thing co tam 6 tam vi tir Ia tap hop nhtmg dirong thing bat bien duy nhat cua phep vi tu. Phep vi tir phang gay nen mot phep vi tir tren moi dirong thing (bat bien) di qua tam vi tu.

Dinh ly 18. MQi phep vi nr phang ti so duong hay am deu Ia mot phep dong dang thuan.

Clui thich, H~ so (hay ti so) vi tir k co the duong hay am. Vi the, tuy thea k duong hay am, nguoi ta goi phep vi tir V ( 0, k) Ia phep vi tir duong hay phep vi nr am. D~ thay rang phep vi nr dirong bao toan huang cua vecto can phep vi nr am thi d6i huang cua vecto (Hinh 30). Tuy nhien, d~ thay rang:

V(O, -k) = '])(0) 0 V(O, k) = V(O, k) 0'])(0), trong do k > O. Til do suy ra phep vi duong hay am deu bao toan huang cua goc dinh huang giira hai vecto hay giira hai dirong thing va do do, bao toan huang cua hmh. Hinh 30 minh hoa tinh chat nay.

Hinh 30

e) Tam vi tv cua hai dtrong trim

Dinh ly 19. Phep vi nr bien mot dirong tron thanh mot dirong tron. Dao Iai, neu (01, R 1) va (02, R2) la hai dirong tron phan biet cua mat phang thi, noi chung, co hai phep vi nr bien dtrong tron nay thanh duong tron kia rna tam vi nr la cac diem chia trong va ngoai doan noi tam thea ty so hai ban kinh.

GQi S Ia tam vi tu, k Ia he so vi tu. The thi, neu S = 0 thi phep vi nr V(O, k) bien dirong tron (0, R) thanh dirong tron (O,lkIR) dong tam 0, ban kinh R' = Ikl.R.

Neu S =I 0 thi V (S, k) bien dirong tron (0, R) thanh dirong tron (0, R') trong

---+ ---+

do 0' diroc xac dinh b6i SO' = kSO va R' = IklR cung tire Ia S chia 0'0 thea

ti so (O'O,S) = SO'/SO = k.

Dao Iai, gia sir (Oi, Ri), i = 1, 2 la dirong tron khong dong tam va cung khong cung ban kinh. The thi ton tai hai va chi hai phep vi tir V (Si' ki) ,i = 1, 2 tam Sl va S2 thea tlur nr chia ngoai va chia trong doan noi tam 0201 thea cac ti so

46

duong nrong irng k1 = R2/ R1 va k2 = - R2/ R1 bien (01, R1) thanh (02, R2) (Hinh 31). Cac diem Sl va S2 thea thir nr duoc goi Ia tam vi nr ngoai va tam vi tir trong cua hai dirong tron (Oi,Ri). C6 hai tnrong hop dac biet

a) Hai dirong tron dong tam nhung khong cling ban kfnh: 01 = O2 = 0, R1 =IR2. Khi do, d. hai phep vi tir cling tam S trung voi 0 va ti so vi tir k; = ±R2 / R 1 deu bien duong tron (0, R 1) thanh duong tron (0, R2).

b) Hai dirong tron khong dong tam nhung cling ban kinh 01 =I- O2, R1 = R2 = R (cling nrc Ia hai duong tron bang nhau (01, R) va (02, R). Trong tnrong hop nay, phep doi xirng tam 0 Ia phep vi nr duy nhat (c6 he so k = -1) bien (01, R) thanh (02, R), hoac ngiroc Iai, trong d6 0 la trung di~m doan 0102.

Hinh 31

Phep doi xirng tam 0 la phep vi nr dac biet tam 0 ti so k = -1: 'D ( 0) = V(O, -1).

Clui thich.

a) Neu hai dirong tron tiep xiic nhau 6 A thi tiep diem A cua cluing la mot trong hai tam vi tir Si, i = 1,2. A Ia tam vi tir trong hay tam vi tir ngoai cua hai duong tron (01, R 1) va (02, R2) tuy thea hai duong tron do tiep xiic ngoai hay tiep xiic trong voi nhau 6 diem A (Hinh 32a va Hinh 32b). Con tam vi nr thtr hai Ia di~m chia ngoai hay chia trong [0201] thea ti so ±R2/R1.

b) Neu hai dirong tron khong dung nhau, hoac ngoai nhau thi giao di~m cua ( 01 O2) va mot tiep tuyen chung ngoai hoac mot tiep tuyen chung trong cua cluing cling xac dinh tam vi nr ngoai Sl hoac tam vi nr trong S2 cua hai dirong tron (01, R1) va (02, R2) (Hinh 33a va Hinh 32b).

47

Hinh 32a,

Hinh 32b

Hinh 33a,

d) Tieh cua hai ph.ep V! tlj

Hinh 33b

Dinh ly 20. Tfch cua hai phep vi tir c6 11 s6 k1 va k2 HI. mot phep vi tu. Ti s6 k = k1 k2 c6 tam thing hang voi tam cua phep vi nr d6 hoac mot phep tinh tien tuy thea k1k2 =I- 1 hoac k1k2 = 1.

Chang minh. Gia sir V1 (01, k1) va V2 (02, k2) la hai phep vi tu. D~ thay rang neu 01 = O2 = 0 thi V2( 0, k2) 0 V1 (0, kd = V( 0, k1k2) neu k1k2 =I- 1.

Neu O2 =I- 01, hroc do cua tfch la M ~ M1 ~ M'. Theo dinh nghia, voi

---+ ---+ ---+ ---+

moi M, thi 01M1 = k101M va 02M' = k202M1 (Hinh 34). Khir diem trung

gian M, nho quy tac cong vecto

---+ ---+ ---+

02M1 = 01M1 = -0102

H~ tlnrc tren tro thanh

---+ ---+---+

\1M 02M' = k2{k101M - 0102}

(1)

v ~y diem 0 phai tim, neu ton tai thi diroc xac dinh boi (1) [bang each cho M'=M=O]:

---+ ---+---+

020 = k2{k1010 - 0102}

(2)

48

nghia la

---+ ---+

(1 - k1k2)010 = (1 - k2)0102

(2')

Hinh 34

Truong hop tong quat k1k2 =I- 1. Ding thirc (2) xac dinh mot diem 0 duy nhat, thing hang voi 01, O2. Bang each lay (1) trir (2) ve c16i ve, ta di den

---+ ---+

'11M, OM' = k1k20M

(3)

Truong hop dac biet k1 k2 = 1. Khong c6 mot diem bat dong nao, nhung voi k2 = l/k1 thi dang tlnrc (1) tro thanh

---+ ---+ ---+

'11M, 02M' = 01M - k20102

rna ta viet lai duoi dang

---+ ---+

'11M, MM'=(I-k2)0102

(4)

---+

H~ tlnrc (4) nay xac dinh phep tinh tien theo vecto ---;z: = (1, k2) 01 O2. Day la

dieu phai chimg minh.

3 SI! xac dinh mot phep dong dang phfing

Dinh ly 21. Cho ABC va A' B' C' la hai tam giac dong dang cho tnroc trong mat phang P

A'B' B'C' C'A'

AB = BC = CA = k

49

Bao giO' cling c6 mot va chi mot phep dong dang Z : P ----+ P bien A, B, C thea thir nr thanh A', B' , C'.

Chang minh. Xet phep vi tir V(A, k); n6 bien tam giac ABC thanh tam giac ABlCl trong d6 ABI = kAB, CIA = kCA (Hinh 35). Nhu vay phep vi nr V(A, k) bien tam giac ABC thanh tam giac LiABlCl = LiA'B'B'. Theo dinh ly 3 thi c6 mot phep doi hinh V duy nhat bien A, B1 ,C1 thea thtr nr thanh A', B', C'. Do d6, tfch Vo V(A, k) la mot phep dong dang Z(lkl) 11 solkl bien A thanh A', B thanh H' va C thanh C'.

Hinh 35

Bay gio gia su c6 hai phep dong dang Zl va Z2 deu bien ABC thanh A' B' C' thi phep Z:;l 0 Zl Ia mot phep dong dang 11 so 1, nrc Ia mot phep doi hinh V bien tam giac ABC thanh chinh n6, trong d6 bien A thanh A, B thanh B va C thanh C. Vi vay, thea dinh ly 2, Z:;l 0 Zl = Id va do d6 Z2 = Zl. Dinh ly da diroc cluing minh.

H~ qua 1. M9t phep dong dang phang bao giO' cling c6 the phan tfch diroc thanh tfch cua mot phep vi nr va mot phep ding cu (doi hinh hoac phan doi hinh) thea thtr nr d6 hay thea thtr nr ngiroc lai.

Chti thich,

a) Doi voi cac phep ding cu thi phep doi xirng - true d6ng vai tro phan tir sinh con doi v6'i cac phep dong dang thi d6ng vai tro phan tir sinh Ia phep vi nr va phep doi xirng - true, trong d6 phep vi nr d6ng van tro noi bat, phan biet S1)' khac nhau giira Z va V 611 so Ikl =11.

b) Neu Z(lkl) = V 0 V(A, k) thi Z-l(lkl) = V(A, 11k) 0 V-I = Z(I~I)

c) S1)' phan tfch n6i tren la khong duy nhat.

H~ qua 2. M9t phep dong dang thuan trong mat phang nrong irng voi tfch cua mot phep vi tir va mot phep quay hay mot phep tinh tien (theo thir tir d6 hay

50

thea thtr nr ngiroc lai).

H~ qua 3. Mot phep dong dang nghich trong mat phang nrong irng voi tfch cua mot phep vi tir va mot phep d6i xtrng - true hay mot phep d6i xtrng - tnrot (theo tlur nr d6 hay thea tlur nr ngiroc lai).

4 Di~m bat dQng va dang chinh tac cua mot phep dong dang phfing

a) Dang ehinh tl'ie cua mc)t phep d6ng dang thuan

Djnh ly 22 va Djnh nghia MQi phep dong dang thuan trong mat phang khac voi phep tinh tien deu e6 mot diem bat dong duy nhat 0 va tuong dirong voi tfch giao hoan cua mot phep vi nr va mot phep quay cling tam o. Tich giao hoan nay diroc goi Ia phep vi nr - quay va Ia dang chinh tac cua phep dong dang (thuc su (2») thuan trong mat phang.

Chang minh. That vay, neu Z Ia mot phep vi nr V thi tam vi Ia diem bat dong duy nhat cua n6. Gia sir Z =I- V va gia sir 0 la diem bat dong can tim cua phep dong dang thuan Z diroc xac dinh boi hai tam giac dong dang (thuan) nrong img ABC va A' B' C', trong d6 A', A; B', B; C', C la cac cap diem nrong irng. Vi Z =I- V nen A' B' lr AB (Hinh 36). Tu £:'OA' B' "" £:,OAB ta diroc

LOAP = LOA'P, LOBP = LOB'P, LAOB = LA'OB'

trong d6 cac g6e xet a day deu Ia g6e dinh luning (mod Jr) cua cac dirong thing. Tir d6 suy ra AA'OP va BB'OP Ia nhtmg trr giac noi tiep. Vay di~m bat dong o la giao diem thir hai cua hai dirong tron (AA' P) va (BB' P).

-----+ -----+ -----+-----+

Dong thai, tir ding tlnrc (OB',OA') = (OB, OA), thea he tlnrc Salo, ta diroc

-----+ -----+ -----+-----+

(OA, OA') = (OB, OB') = ip (mod 2Jr)

(i)

Ngoai ra, cling tir £:'OA' B' "" £:'OAB, ta con e6

OA' OB' A'B'

OA = OB = AB = k

(ii)

Vay, cac ding tlnrc (i) va (ii) con n6i len rang mot phep dong dang thuan Z trong mat phang bao gio cling phan tfch duoc mot each duy nhat thanh tfch giao hoan cua mot phep vi nr tam 0 ti s6 k va mot phep quay g6e rp (mod 2Jr) xung quanh 0, trong d6 0 la diem bat dong cua Z.

(2)WC Ia ti so dong dang k #- 1 va do d6 Z #- D

51

Tich giao hoan nay diroc goi la phep nr quay va g6c rp cling diroc goi la g6c dong dang.

Phep dong dang thuan tam 0 g6c rp va 11 s6 k diroc ky hieu la Z ( 0, rp, k).

Nhu vay,

Z(Orp, k) = Q(O, rp) 0 V(O, k) = V(O, k) 0 Q(O, rp)

Hinh 36

Chti thich, Tren hinh 36 ta khong ve cap diem nrong ling thir ba C, C' cua Z VI thuc ra chi can hai cap diem tuong ling A, A' va B, H' la du d~ xac dinh mot phep dong dang phang. Tuy nhien, can den mot cap diem tlur ba C, C' ntra d~ xac dinh huong, cling tire Ia d~ xac dinh Z Ia thuan hay nghich tuy thea hai tam giac dong dang ABC va A' B' C' Ia cung huang hay khac huang.

b) Dang ehinh tlle cua mQt ph.ep d6ng dung nghjch (phang)

Dinli lj 23 va Dinli nghia. MQi phep dong dang nghich trong mat phang khong phai Ia mot phep doi hinh nghich deu c6 mot diem bat dong duy nhat 0 va nrong ling vci tfch giao hoan cua mot phep vi nr duong tam 0 11 s6 k va mot phep d6i xirng qua mot dtrong thang ,6. di qua 0 (hoac mot phep vi nr am tam 0 11 s6 -k va mot phep d6i xirng - true ,6.' cling di qua 0 va vuong g6c voi ,6. (0). Tfch giao hoan nay diroc goi Ia phep vi tir - doi xtrng va true doi xtrng ,6. diroc goi la true dong dang, 0 diroc goi la tam dong dang. Phep dong dang nghich tam 0, true ,6. va 11 s6 k diroc ky hieu la Z ( 0 , ,6., k).

Chang minh. That vay, gia sir Z =I '])(,6.), Z ~(,6., v+) va gia sir 0 la di~m bat dong can tim cua phep dong dang nghich Z trong mat phang xac dinh boi hai tam giac dong dang ABC va A' B' C'. VI tam giac 0 A' B dong dang nghich voi

52

tam giac DAB nen ta c6 (Hinh 37)

---+ ---+ ---+ ---+

(OA', DB') = -(OA, DB)

(1)

Hinh 37

Tir d6 suy fa hai cap dirong thing 0 A, 0 A' va DB, 0 B' c6 chung nhau hai dirong thing phan giac dong thai cling la hai true doi xirng Os, Os', vuong g6c vci nhau aD.

G9i P va Q liln hrot la giao di~rn cua true Os vci cac doan thing AA' va BB' ta diroc

PA' OA' DB' QB'

P A = OA = DB = QB = k

Mat khac, goi Al = 2)os(A) va B, = 2)osl(B) va neu AB lr Os thi (A'Al) n (B' Bl) = O. Tir d6 suy fa di~rn bat dong 0 la giao di~rn cua hai trong ba dirong thing dong quy: PQ,A'AI va B'Bl:

(2)

Clui thich,

a) Neu d~ y rang VI k #1 nen khong nhtrng (AA') va (BB') cat Os tai P va Q rna (AA') va (BB') con cat d. Os' thea thir nr cac diem P' va Q' va ta cling c6 cac he tlnrc (2) trong d6 P va Q diroc thay d6i boi P' va Q', chi khac la trong tnrong hop nay thi P' va Q' deu chia ngoai cac doan A' A va BB' thea ti so so h9C k. B6i vay, diem bat dong 0 con la diem dong quy cua bon dirong thing PQ, P'Q', A'AI va B' Bl, trong d6 cac cap di~rn P, Q va P', Q' thea thtr nr Ia cac di~rn chia trong va chia ngoai cac doan thing [A' A] va [B' B] thea ti so h9C k, trong d6 k la ti so dong dang cua A' B' C' va ABC, cling urc la ti so dong dang cua Z nghich.

53

b) Tren day 1a tnrong hop A' B' lr AB. Ta con phai tnrong hop A' B' II AB.

Neu A' B' II AB, thi khi do, thea dinh ly Talet ta cling e6, PQ II AB II A' B' dong thai dirong thing PQ di qua giao diem cua A B' va A' B. Giao diem nay chinh la diem bat dong 0 cua Z nghich can tim (Hinh 38): 0 = AB' n A' BE (PQ). (PQ) chinh la true Os cua phep dong dang nghich.

Hinh 38

Cling t11 cac ding thirc (1) va (2) ta suy ra phep dong dang nghich Z trong mat phang khong nhtmg e6 diem bat dong duy nhat 0 xac dinh nhu tren rna con phan tfch diroc mot each duy nhat thanh tfch giao hoan cua mot phep vi nr tam 0 11 so k (hoac - k) va mot phep doi xirng true ,6. di qua 0 (hoac ,6.' vuong g6e voi ,6. 6 0). Tren Hinh 38 (PQ) = ,6., (P'Q') = ,6.'. Tich giao hoan nay diroc goi Ia vi nr doi xirng va ky hieu van tftt Ia Z(O,,6., k). Hai dang Z(O, cp, k) va Z(O,,6., k) - phep vi nr - quay va phep vi nr - doi xtrng goi la hai dang chinh tac cua phep dong dang phang.

5 Bai tap ve xac dinh va dang chinh tac ella mot phep dong dang phfing

4.1 Chirng minh dinh Iy: Trong mat phang eho hai doan thing AB va A' B' = kAB, (0 < k =I 1). The thi e6 hai va chi hai phep dong dang Z, : P ---+ P (i = 1, 2) cua P bien A thanh A', B thanh B', trong do mot phep la dong dang thuan con phep kia Ia dong dang nghich (dong dang girong hay phan dong dang).

4.2 Cluing minh rang hai tam giac ABC va A' B' C' e6 cac canh nrong img song song v6'i nhau nhung khong bang nhau Ia hai tam giac vi nr v6'i nhau. [N6i mot each khac, cac dirong thing AA', BB', CC' noi cac cap dinh nrong irng dong quy 6 mot diem 0 nao do. Diem 0 chinh la tam cua phep vi nr bien tam giac nay thanh tam giac kia.

4.3 Cho hinh thang ABC D e6 hai day la AB va CD. Chirng minh rang e6 the ehia hinh thang do thanh hai hinh thanh nho dong dang (thuan) voi nhau

54

(DC N M "" M N B A) bang mot doan thing M N song song vci hai day, M tren AD va N tren BC.

4.4 Hai dirong cheo AIA2 va B, B2 cua mot til giac (loi) noi tiep AIA2BIB2 cat nhau a di~m P. 09i Ao, va B; liln hrot la trung di~m cua PAl va P B" cluing minh rang AoAlB2 va BoBlA2 la hai tam giac dong dang nghich.

4.5 M9t hinh chir nhat rna moi canh (hoac canh keo dai cua no) clura mot dinh cua mot til giac (loi) ABC D goi la hinh chir nhat ngoai tiep til giac do, ngiroc Iai, ABC D goi Ia til giac noi tiep hinh chir nhat. Chirng minh rang co vo so hinh chir nhat M N PQ ngoai tiep mot til giac (loi) ABC D co hai duong cheo AC, BD vuong goc, va tat ca cac hinh chir nhat do deu dong dang vci nhau.

4.6 Cho AAl, BBI va CCI la ba duong cao cua mot tam giac ABC. 09i A', B', va C' Ia cac hinh chieu (vuong goc) cua mot diem M bat ky nam trong mat phang liln Iuot tren AAl, BB, va CC,. Chirng minh tam giac A' B'C' dong dang nghich voi tam giac ABC.

4.7 Cho hai tam giac deu (khong bang nhau) ABC va A' B'C'. Hoi bao nhieu phep dong dang (thuan va nghich) bien tam giac thir nhat thanh tam giac thir hai.

4.8 Chirng minh rang trong mot phep dong dang thuan thi tam dong dang, hai diem nrong irng M, M' va giao diem P cua hai dirong thing nrong irng a va a' xuat phat tir hai diem tuong irng do cling nam tren mot dirong tron.

---+ ---+

4.9 Chirng minh rang tam cua phep vi nr - quay bien AB thanh A' B' cling Ia

---+ ---+

tam cua phep vi tir - quay thir hai bien AA' thanh H H' (hai phep dong dang

thuan co tam chung).

4.10 09i C la giao diem cua hai dirong thing AA' va BB', C' la giao diem cua hai dirong thing A B va A' B'. Chirng minh rang bon dirong tron (AA'C'), (BB'C') va (A' B'C) dong quy a mot diem 0 nao do.

4.11 Cho hai dirong tron (0, R) va (0', R') cat nhau a hai diem P va Q. Chirng minh rang cluing tuong irng voi nhau trong mot phep dong dang thuan (vi nr quay) co tam a giao diem tlur nhat P (hoac Q) va hai diem nrong img M, M' thi thing hang vci giao di~m thtr hai Q (hoac P)

4.12 Gia sir P la mot diem nam trong doan thing [AB]. v« cling mot phia cua duong thing AB ta dung hai hinh vuong APEF va PBCD, cluing minh rang

a) Ba duong thing AD, BE va C F dong quy a mot diem Q nao do.

55

b) Khi P chuyen dong tren doan thing [AB] thi dirong thing (PQ) Iuon di qua mot diem co dinh S, each deu A va B. Tim quy tfch cua diem Q.

4.13 Cho mot tam giac (loi) ABC D co hai dirong cheo cat nhau a 0 nhung khong vuong goc voi nhau. 09i A' va C' liln Iuot la hinh chieu (vuong goc) cua A va C tren BD; H' va D' liln Iuot Ia hinh chieu (vuong goc) cua B va D tren AC. Cluing minh rang trr giac A' B' C' D' dong dang nghich voi trr giac ABCD.

4.14 Cho hai dirong thing a va b cat nhau 6 mot diem C. Voi moi diem M trong mat phang ta cho lien ket voi mot diem moi M' bang each sau day. Qua M ke mot cat tuyen m. cat a a A va b a B sao cho M Ia trung di~m cua doan [AB] (m la duy nhat). 09i A' la hinh chieu vuong goc cua A tren b, B' la hinh chieu vuong goc cua B tren a va M' la trung diem cua doan [A' B'].

a) Chirng minh rang anh xa f : M f---7 M, t11 P ----+ P Ia mot phep bien hinh cua mat phang.

b) Hay goi ten phep bien hinh do.

§ 5 V *n dung phep dong dang vao viec giai mot s6 bai toan hinh hQC phang - Thi du minh hoa va Bai tap

1 Dog dung phep vi nr vao giai toan hinh hoc

Thi du L Chirng minh rang trong mot tam giac

a) Ba dirong trung tuyen dong quy a mot diem, di~m nay each moi dinh cua tam giac bang 2/3 d¢ dai cua duong trung tuyen phat xuat t11 dinh do.

b) Trong tam G tnrc tam H va tam 0 dirong tron ngoai tiep cling nam tren mot dirong thing, goi Ia dirong thing dIe, va giira cac di~m do co he thirc GO 1

GH - 2

Chang minh.

a) That vay, thea ket qua neu trong bai tap 4.2 thi L,A' B' C' la hinh vi tir cua tam giac ABC (Hinh 39) trong phep vi nr tam G (la trong tam cua L,ABC). Ti so k = B'C'/BC = -1/2.

b) Vi tam 0 cua dirong tron ngoai tiep tam giac ABC trung vci tnrc tam H' cua tam giac A' B' C' nen 0 (cling nrc la H') la hinh vi nr cua tnrc tam H cua tam giac ABC trong phep vi tir V( G, -1/2) noi tren bien tam giac ABC thanh tam

56

---+ ---+

giac A'B'C', nghia la ta diroc (Hinh 40) GO = -~GH hay la GO/GH = -1/2.

Day Ia dieu phai cluing minh

Hinh 39

Thi du 2. (Dinh ly v~ dtrong tron dIe) Chirng minh rang trong mot tam giac, trung diem cac canh, chan cac dirong cao va trung diem cua cac doan thing noi tnrc tam voi cac dinh cung nam tren mot dirong tron, goi la dirong tron chin diem hay dirong tron dIe cua tam giac d6. Ban kinh dirong tron dIe bang mra ban kinh dirong tron ngoai tiep tam giac va tam 0' cua dirong tron dIe thing hang voi tam o dirong tron ngoai tiep, trong tam G va tnrc tam H tren dirong thing dIe sao cho 0' la trung dieme cua doan thing OH va bon diem H, G, 0, 0' lam thanh mot hang diem dieu hoa.

Hinh 40

Chang minh.

That vay, goi 0' Ia tam dirong tron (A' B'C') the thi 0' Ia hinh V! tir cua tam o dirong tron (ABC) trong phep V! tir V(G, -1/2). Do do 0' ciing nam tren

57

dtrong thing dIe OG H va ta c6 (Hinh 40).

---+ 1 ---+ GO'

GO' = -2GO, hay la GO

1 2

Tir he thirc tren ta d~ dang suy ra HG = 2GO = 40'G va OH = 30G; 30'G = 0'0 = -0' H. Va do ct6 ta diroc 0' H = -0'0 va

(HGOO') = OH O'H =-1 OGO'G

Day Ia dieu phai cluing minh. Cling tir ct6 suy ra

--- -

HO GO 2

---+ ---+

hay Ia H 0' = ~ H 0 va do ct6, H Ia tam vi nr tlur hai bien duong tron (0)

(ABC) thanh dtrong tron (0') = (A' B'C') thea ti so 1/2.

Trong phep vi nr thir hai V(H,1/2) nay, cac ctinh A, B, C cua tam giac ABC theo thir tir bien thanh cac trung diem A", B", C" cua cac doan thing noi tnrc tam H voi cac ctinh A, B, C va do ct6, cac diem nay nam tren dirong tron (0) = (A'B'C').

Ngoai ra, ct~ thay rang A' A", B' B" va C' C" la cac dirong kinh cua dirong tron (0'). Tir ct6 suy ra (0') cling di qua chan A1, B1, C1 cua cac dirong cao AA1, BB1, CC1 cua tam giac ABC. Chfnh VI vay, dirong tron (0') diroc goi la dtrong tron chin diem hay dirong tron dIe cua tam giac ABC.

Thi du 3. Su dung phep vi tu, cluing minh dinh ly Menelarmit (Menelaiis) Dinh lYe Dieu kien can va ctu ct~ ba diem A', B', C' nam tren ba duong thing chua cac canh BC, C A, AB cua mot tam giac ABC thing hang Ia

A'B B'C C'A =.=.=-+1 A'C B'A C'B

(*)

58

Hinh 41

Chang minh. Tnroc het, ta ky hieu nhir sau cho gon Vi = V ( G' , k1), V2 = V(B', k2), V1 = V(A', k3), trong d6

G'A B'G A'G

k=-k=-k=-

1 G'B' 2 B'A' 3 A'B

(1)

The thi ra rang ta c6 ket qua nhu sau

VI : B f---7 A, V2 : A f---7 G, V1 : B f---7 G

Mat khac, V2 0 VI : B f---7 G. Nhirng thea dinh ly 20 ve tfch cua hai phep vi tir thi V2 0 Vila mot phep vi tir c6 tam thing hang voi G' va B', va c6 ty so vi tir k = klk2 #1 (vi B'G' lr (BG) va (B'G') n (BG) = A'. L~i do V2 0 VI bien B thanh G, boi the tam cua phep vi nr tfch V2 0 VI phai Ia giao diem A' cua hai duong thing BG va B'G'. N6i khac di Ia V2 0 Vi = V3. Vay, ta phai c6 kl.k2 = k3, hay Ia k1.k2.k:;1 = +1 (2) . Cuoi cung, thay cac gia tri cua ki(i = 1,2,3) tir (1) va (2), ta thu diroc (*) can tim.

Thi du 4. Chirng minh rang quy tfch nhtmg diem cua mat phang rna ty so khoang each tir d6 den hai duong thing ceit nhau b mot diem 0 bang k (k > 0) khong d6i gom hai dirong thing di qua O.

Hinh 42

Chang minh. Gia sir x' x va y' y la hai dirong thing cho tnroc cat nhau b diem 0 (Hinh 42) va M la mot diem cua mat phang sao M H / M K = k (hang so dirong cho truce). G9i Ml la diem nrong img cua diem M trong mot phep vi nr thay d6Pl tam O. V ay quy tfch nay se gom nhtrng duong thing da: qua O. Dieu d6 khien ta nghi den viec tim nhtmg di~m thuoc quy tfch can tim tren mot cat tuyen AB cua hai dirong thing Ox va Oy sao cho 0 A = 0 B. V 6'i m6i diem

(3)ttrc Ia chi c6 thay d6 v6 gia tri cua ti so vi tv rna thai

59

M cua AB cac tam giac vuong M AH va M B K e6 cac g6e nhon bang nhau a A va B thi dong dang voi nhau va ta e6 M HIM K = MAIM B. Diem M se la mot diem cua quy tfch neu ty so MAIM B = k, va r5 rang la MHIM K = k hay la MAIM B = k. Khi ty so k #1 thi tren (AB) e6 hai diem M va M' dap irng dieu kien doi hoi cua bai toano V ay quy tfch can tim bao gom hai duong thing (OM) va (OM') lien hop dieu hoa doi vci hai dtrong thing Ox va Oy da eho (vi hanh diem (ABM M') la dieu hoa.

Thi du 5. Tam giac ABC noi tiep dtrong tron (0) eho tnroc e6 hai dinh A, B co dinh.

a) Tim quy tfch trong tam G cua tam giac ABC.

b) Tir d6 suy ra quy tfch tnrc tam H cua tam giac ABC.

Lei giai.

-----+ -----+

a) G9i Co la trung diem canh AB, ta e6 (Hinh 43) CoG = ~CoC. Suy ra {G}

nhan diroc tir {C} = (0), trong phep vi nr VI (Co, ~). Vay {G} la dirong tron tam 01, ban kinh R1 = R13, trong d6 Ria ban kinh cua (0) can 01 diroc xac dinh

-----+ -----+

boi 00' = ~OCo. (1)

Hinh 43

-----+ -----+

b) Ta e6 0 H = 30G (Thi du 2). Do do, {H} diroc suy ra tir {G} hai phep

vi nr V2(0,3) tam 0, ty so k = 3. Vay {H} la dirong tron(O',R), trong d6 0'

-----+ -----+ -----+-----+

diroc xac dinh boi 00' = 3001, hay (theo (1)) 00' = 20Co• (2)

H~ thirc (2) can cluing to rang 0' doi xirng vci 0 qua C va qua ca (AB). Tir d6 ta di den ket Iuan rang quy tfch tnrc tam H cua tam giac ABC la dirong tron (0', R) doi xtrng voi dirong tron (0, R) ngoai tiep ABC qua canh AB.

Nluin. xet. Theo lap Iuan thi {H} diroc suy ra tir {C} = (0, R) boi phep dong dang Z = V2 0 VIla tfch cua hai hep vi nr VI (Co, 1/3) va V2(0,3)

-----+ -----+

vecto ~O' = 20Co (khong d6i) thea Dinh ly 20. Ta lai thay ket qua da biet

60

---+ ---+

(CH = 20Co) nen trong thi du 2, muc 1 cua bai 3.

Thi du 6. Dung mot dirong tron tiep xtic val mot dirong tron (0) cho tnroc va voi mot dirong thing ,6. cho tnroc, biet mot trong cac tiep diem.

Lai guii. Gia sir (w) Ia mot dtrong tron tiep xuc voi dirong tron (0) 6 A va voi dirong thing,6. 6 B (Hinh 44), Ta nghi den mot phep vi tir tam A la tiep diem cua (0) va (w). That vay, (0) Ia anh cua (w) trong mot phep vi nr tam A bien B thanh mot trong hal d:iu nut C hoac D cua dtrong kinh CD cua (w) vuong g6c voi ,6.. Ta xet hai tnrong hop nhu bai toan da d::;tt ra (Hinh 44).

Hinh 44

1) Neu diem A cho tnroc tren dirong tron (0), the thi chang han, dirong thing C A cat ,6. 6 diem B va do d6, phep vi tir v (A, k = AB / AC se bien dirong tron ( 0) thanh mot duong tron (w) tiep xiic voi (0) 6 A va voi ,6. 6 R. (M9t Ioi giai thtr hal diroc thuc hien val dirong thing D A).

2) Neu diem B cho tnroc tren dirong thing ,6. the thi dirong thing C B cat lai (0) 6 diem A va phep vi nr v (A, k = AB / AC) se cho mot dirong tron (w) tiep xuc voi D 6 B va voi (0) 6 A. (M9t lai giai thir hai thuc hien tuong tir voi (DB)).

2 Dng dung phep dong dang thuan (V! nr quay) vao viec giai toan hinh hoc

Thi du 7. Xet mot tam giac ABC va mot diem D cho tnroc. Ta dung cac tam giac ADE, DBF dong dang (thuan) voi tam giac ABC. Hay so sanh cac tam giac ABD, ACE va CBF va tim hieu ban chat cua trr giac CEDF.

Lai guii.

Tir DADE"" DDBF ta suy ra DABC "" DADE (Hinh 45) va cung vay, DABD "" DACE. Cac tam giac BAC va BDF la dong dang thuan, tir d6 cung

61

suy ra DBAD "" DBCF. VI vay, ba tam giac ABD, ACE va CBF Ia dong dang thuan.

Hinh 45

Nluin. xet. Til ba tam giac ABC, ADE va DBF dong dang thuan ta da thu diroc ba ta tam giac ABD, ACE va CBF cling dong dang thuan. Nhu vay la ta c6 the trao d6i vai tro cua C va D cho nhau roi d6.

---+ ---+

Bay giO' ta d:;tt (AB, AC) = a (mod 27r) va AC = kAB. The thi phep

---+ ---+

dong dang thuan 2(A, k, a) se bien BD thanh CE, con phep dong dang thuan

---+ ---+

2(D,k,a) se bien BD thanh FD. Til d6 ta c6 CE = kBD va FD = kBD

---+ ---+ ---+ ---+

suy ra CE = F D (1). Mat khac (BD, CE) = a va (BD, F D) = a suy ra

---+ ---+ ---+ ---+ ---+

(CE,FD) = 0, (mod 27r) (2). Til (1) va (2) suy ra CE = FD va do d6,

C E D F la mot hinh binh hanh.

Thi du 8. Hay sir dung phep dong dang thuan d~ giai bai tap 3.26 (Bai toan Napoleon: V~ tam giac deu AoBoCo sinh boi mot tam giac bat ky ABC)

Hinh 46

D~ thay rang AoBC, BoCA va CoAB Ia ba tam giac can dong dang c6 g6c 6' day 30°. Ky hieu 21 = 2(C,300,V3) 22 = Z(B,300, ~). Xet tfch 22 0 21. D~ thay 21 : Ao f---7 A'; B; f---7 A va 22 : A' f---7 Ao : A f---7 Co. Do d6, 22021 : B; f---7 Co va Ao f---7 Ao. Mat khac, k1k2 = V3.~, CP1 + CP2 = 2.300 = 600

62

nen 22021 Ia mot phep doi hinh D, cu th~ Ia phep quay tam Ao bien B; thanh Co. Vay AoBoCo la mot tam giac deu. Bai toan diroc cluing minh.

Thi du 9. Tren hai dirong thing a va b cat nhau 6 mot diem C c6 hai dong tir chuyen dong thing deu nhung voi van toe khac nhau: A tren a voi van toe VI, B tren b voi van toe V2, VI =I- V2. Chung khong gap nhau 6 C.

a) Chirng minh rang a bat err thai di~m nao dirong tron ngoai tiep tam giac ABC cung Iuon di qua mot di~m c6 dinh 0 nao d6, khac C.

b) Tim quy dao chuyen dong cua dong tir M Iuon 6 vi trf trung diem cua [AB].

Lai guii. a) Tnroc het, ta cluing minh rang anh xa f : A(t) f--7 B(t) tir duong thing a den dirong thing b Ia mot anh xa dong dang (thuanj Z. That vay, gia su tl, t2 la hai thai di~m khac nhau nao d6, 0 :s; i, < t«. D~ cho gon ta ky hieu nhu sau A(ti) = A, B(ti) = B, i = 1,2. The thi

BIB2 = V2(t2 - td = V2 = k (0 < k khong d6i)

A1A2 VI(t2 - td VI '

-----+ -----+

va (AIA2' (BIB2) = rp (mod 2Jr), trong d6 (a, b) = rp (mod Jr) Ia g6c giira hai

dtrong thing a va b. B6i vay, trong mat phang c6 mot phep dong dang thuan 2(0, rp, k) bien a thanh b, trong d6 diem A(t) bien thanh diem B(t) (Hinh 47).

Hinh 47

Theo Dinh ly 22 thi tam dong dang (diem bat dong) 0 cua 2 la giao diem thir hai cua hai dirong tron (AoBoCo) va (ABC). Vay dirong tron (ABC) Iuon di qua 0, khac voi C co dinh.

b) Ky hieu An = A(to), B; = Bit; = 0), M; la trung diem cua doan [AoBo]; v: va V; la cac vecto van toe cua A va B.

63

Trti lai. Quy tfch cua M Ia dtrong thing Mam di qua M; va co vecto chi

---+

phirong la --;; = ~(~ + V;) (suy ra tir 2MaM = (~ + V;)t).

Thi du 10. Dung mot tam giac bier cac et9 dai ha, hb' he ba dirong cao cua no.

Ph/in. tick. 09i a, b, cia et9 dai cac canh cua tam giac ABC phai tim, ta co 1 1 1 hbhe a : b : c = - : - : - = -- : he : hb ha hb he s.

(*)

Cac eting thirc (*) cluing to rang neu ton tai, thi tam giac can dung se dong dang val mot tam giac T co et9 dai cac canh a', b' , c', ky hieu T ( a' , b' , c') val a' = h~~c, b' = he va c' = hb (**)

Vay, viec dung tam giac ABC phai tim dira ve viec dung tam giac T(A' B'C') co cac canh xac dinh boi (**):

DABC ~ DT(A'B'C')

Tuy nhien, viec dung tam giac T lai nIt don gian boi VI lai dira v~ viec dung doan ty 1~ tlur nr a'. Tir su phan tfch tren day, ta suy ra tirng burrc et~ dung diroc DABC can tim. Buoc dung cuoi cung Ia dung tam giac ABC, vi nr val tam giac ABaCa (Hink 48), trong phep vi nr V(A, k) tam A, ty so k = ~~, trong do ABaCa Ia mot tam giac dung bat ky, mien Ia DABaCa ~ D T(a', b'~ c') va h~ = AHa la chieu cao AHa ha tir A cua tam giac ABaCa. D~ dang cluing minh tam giac ABC co cac chieu cao Ia ha, hb' he.

Hink 48

Bieri ltuin, Bai toan co Ioi giai khi va chi khi ton tai tam giac T(a', b', c').

Boi vay, bai toan co nghiem khi va chi khi h1a' ~b ,~c bieu thi et9 dai cac canh cua mot tam giac nao do; cling nrc la ton tai

( 1 1 1)

T ha' hb' he

64

Thi du 11. Dung mot til giac (loi) noi tiep ABCD bier d9 dai cac canh AB = a, BC = b, CD = e, D A = d, trong do a, b, c la nhtmg d9 dai cho tnroc.

Ph/in. tick. Gia sir til giac ABC D can tim da dung duoc. Til giac ABC D noi tiep khi va chi khi LA + LC = 180° (hoac LB + LD) = 180°). Chinh vi vay, keo dai canh BC v~ phia C d~ xuat hien g6c LDCx = LBAD va k~ bu voi LDCB. Tren Cx lay di~m E sao cho DDCE "" DDAB. Bai toan dung til giac noi tiep ABCD diroc quy ve viec dung DDCE.

Hink 49

Gia su DDCE "" DDAB. Hai tam giac nay chung dinh D, boi vay DDCE

---+ ---+

diroc suy ra tir DDAR boi phep vi tir - quay Z(D, cp, k) voi cp = (DA, DC), k =

c / d). Ta nghi den viec su dung phep vi nr - quay d~ giai bai toan dung hinh nay

---+ ---+

Ia vi vay, Boi vay, di;it c/d = k, (DA, DC) = LADC = 6 = ip roi xet phep vi nr

- quay Z(D, 6, k). Z bien D f--7 D, A f--7 C va B f--7 E sao cho C E [BE] tHinh. 49). The thi DDCE "" DDAB va do do LDCE = LDABva B, C, E thing hang thea thir nr do, dong thai ta diroc LBDE = LADC = 6.

Nhu the Ia, bang each sir dung phep vi tir - quay Z nay ta da chuyen bai toan dung til giac noi tiep ABCD v~ bai toan dung tam giac DBE c6 cac yeu to da bier

C, / ac + bd DEc

BC=b CE=-a va do do BE= CD=c -=-

, d ' d' 'DB d

Bai toan quy ve dung diem D la mot trong cac giao diem cua dtrong tron 11 (C, c) va dirong tron Apoloniiiyt (12) c6 duong kinh I J rna I va J chia trong va ngoai doan [BE] thea ty so h9C k = cl d. Dinh A dirocdung sau cung.

Bieri ltuin, Bai toan c6 th~ c6 mot 1O'i giai hoac khong c6 1O'i giai nao tuy thea ( 11) va (12) c6 cat nhau hay khong.

65

3 Dng dung phep dong dang nghich (vi nr - doi xrrng) vao viec giai toan hinh hoc

Thi du 12. Trong mat phang cho hai tam giac deu ABC va A' BC cung huong, c6 dinh C chung sao cho A'va B' khong trung voi tam 0 dirong tron (ABC). 09i M va N liln Iuot Ia trung diem cua cac doan thing A' B va AB'. Chirng minh rang

a) Tam giac OB'M dong dang nghichvoi tam giac OA' N.

b) Hai g6c LA'OB' va LMON c6 chung nhau dirong phan giac

Lai guii. a) Xet phep 2(0,~,~) = V(A,~) 0 Q(C, ~). D~ thay 2 : A' f--7 B' f--7 N; 0 f--7 0' f--7 0 va do do, 0 Ia tam dong dang cua 2. Til do ta diroc

---+ ---+

OA' = 20N va (OA', ON) = 7r/3. Suy ra tam giacOAN vuong 6 N va la nih

cua mot tam giac deu. Chirng minh tuong tir ta c6 tam giac 0 B' M vuong 6 M va cling Ia nih cua mot D deu khac.

Hinh 50

Kit ltuin, Tam giac 0 B' M dong dang nghich voi tam giac 0 A' N.

b) Til (1) suy ra LA'OB' va LMON c6 chung nhau duong phan giac.

Thi du 13. Trong mat phang cho tam giac ABC. M9t dirong tron (0) thay d6i di qua A, khong tiep xiic voi cac dirong thing AB, AC va c6 tam 0 chuyen dong tren dirong thing BC. Duong tron nay cat tai cac dirong thing AB va AC liln hrot 6 M va N. 11m quy tfch tnrc tam H cua tam giac AM N (D~ thi HSO toan quoc, Bang A, 3/2002).

Huang ddn. giai. Gia sir LA =I 90° (vi neu A = 90° thi H = A vci moi (0) Lai giai 1. 09i D Ia di~m xuyen tam doi cua di~m A tren dirong tron (0).

The thi M va N theo thir nr chinh la cac hinh chieu (vuong g6c) cua D tren (AB) va (AC) va do do tnrc tam H cua AM N la diem doi xtrng voi D qua trung diem

66

M N. G9i u'-« N' l~n hrot Ia hinh chieu cua H tren (AC) va (AB). D~ thay rang tam giac AH M' dong dang nghich voi tam giac ADM. Til d6 ta diroc

(AH, AM') = -(AD, AM) (mod Jr)

AH AM'

AD = AM = ± cos LBAC = I cosLBACI

trong d6 a = LBAC va do d6 ~~ = 21 cos LBACI = 21 cos o]. Cac ding tlnrc tren n6i len rang (AH) d6i xtrng voi (AO) qua phan giac Ap cua g6c LA cua tam giac ABC va ~~ = k khong d6i, trong d6 k = 21 cos al. Vay H Ia anh cua o trong phep vi nr d6i xirng Z(A, Ap, k).

Hinh 51

Trti lai. Neu ky hieu (BC) = a, thi {H} Ia dirong thing a', anh cua a, trong phep dong dang nghich (vi nr - d6i xirng) Z(A, Ll = Ap, k = 2icoAI), bo di hai diem Hi la anh cua Oi(i = 1,2) tren a = (BC) 6 d6 LBA01 = LCA02 = 90° (Hinh 5.1)

Clui thich. {H} = a' di qua hai diem E va F, trong d6 E = P E .L AC, F = QF .L AB va P = 'D(A), Q = 'D(A) Duong thing (PQ) ciing diroc suy ra til a, = (BC) qua phep vi nr V(A, 2).

Ldi giai 2. G9i P = 'D(A), Q = 'D(A); E = P E -.l AC, F = QF -.l AB.

The thi 0 E (BC) hay la D E (PQ) (i). Ta c6 M H cling lnrong va bang DN, D N song song voi P E, M D cling htrong va btingH N, H N song song voi Q F. Suy ra D E (PQ) hay la

FM QD DN MH

FP QP PE PE

hay la H E (EF) (ii). Til (i) va (ii) suy ra {O} = (BC)\ {01, 02} hay la {H} = (EF)\ {H1, H2}; Hi = AOj n (EF), trong d6 E va F tuong irng la tnrc

67

tam cua tam giac AF E vuong va tam giac AFQ vuong a E) (i =I j{ i, j} = {1, 2})

4 Bai tap van dung phep dong dang vao viec giai toan hinh hoc phang

A. V;ll.n dung ph ep V! tv va vi~c giai cac bai toan sau day

5.1 G9i Ao, Bo, Co, Do, liln Iuot la trung diem cac canh AB, BC, CD, DA cua mot hinh vuong ABC D va P Ia mot diem bat ky cua mat phang. Chirng minh rang cac diem A1, B1, C1, va D1 doi xtrng voi diem P liln Iuot qua Ao, Bo, Co va Do, cling la cac dinh cua mot hinh vuong.

5.2 Tu giac loi A1A2A3A4 noi tiep dirong tron (0, R). G9i B, la trong tam tam giac AjAkAZ;i = 1,2,3,4 {i,j,k,l} = {1,2,3,4}. Cluing minh rang trr giac B1B2B3B4 noi tiep mot duong tron; hay xac dinh tam 01 va ban kinh R 1 dirong tron d6.

5.3 Cho mot hinh binh hanh ABCD \c6 khop nail! 6 cac dinh. N6i mot each chinh xac han Ia, d9 dai cac canh khong d6i, cac dinh A va B diroc gill ea dinh nhmrng cac dinh C va D thi chuyen dong trong mat phang. TIm quy dao chuyen dong qua tam 0 hinh binh hanh \c6 khop nail! d6.

5.4 Cho ba di~m A, B, C thang hang thea thu d6. G9i (V1), (V2) thea trr nr la cac dirong tron dirong kinh AB va AC. M9t diem M chuyen dong tren ( V1), dirong thang A M cat lai (V2) 6 diem N. TIm quy tfch giao diem P cua BN va CM.

5.5 Gift sir ba dirong tron (Ao), (Bo) va (Co) e6 cung ban kfnh, thea thir nr tiep xuc voi hai canh cua cac g6e A, B va C cua mot tam giac ABC. G9i (Do) Ia duong tron tlur tmr tiep xiic ngoai voi eft ba duong tron n6i tren. Cluing minh rang tam D ° thang hang voi tam cac dirong tron noi ngoai tiep tam giac ABC.

5.6 Duong tron (J) tiep xiic trong voi duong tron ngoai tiep ABC can 6 A, dong thai tiep xtic vci hai canh AB va AC a M va N. Chirng minh rang trung diem cua doan thang M N la tam dtrong tron noi tiep tam giac ABC. (D~ thi Toan quae te IMO, Rumani 1978).

5.7 Chirng minh rang neu goi 0 Ia trong tam cua hinh trr di~m phang {A1, A2, A3, A4, } va P la giao diem hai dmrong cheo A 1 A3 va A4A.5 thi trong tam T cua hinh trr giac phang Ia hinh vi tir cua diem P trong phep vi tir V ( 0, ~),

-----+ -----+

nghia la OT = -~OP.

68

5.8 Cho hai dtrong tron (Ol.Rd va (02.R2) tiep xtic ngoai nhau a cti~m C. Mot g6c vuong xCy quay xung quanh C; Cx va Cy cat lai (01) va (02) thea thir tir 6 A va B. TIm quy tfch hinh chieu vuong g6c H cua C tren AB.

5.9 Cho mot g6c xOy va mot cti~m A nam trong g6c ct6. Hay dung mot dirong tron di qua A va tiep xiic voi hai canh cua g6c cta cho.

5.10 Cho hai dirong tron (VI) va (V2) dong tarn O. Hay dung mot day cung AD cua dirong tron 100 cat dirong tron nho 6 B va C sao cho AD = 3BC.

5.11 Trong mat phang cho hai cti~m phan biet A va H va mot dirong tron (0) di qua A. Hay dung mot tarn giac ABC noi tiep (0) nhan H lam tnrc tarn.

5.12 Cho mot hinh vien phan A I B cua dirong tron (0) r xac dinh boi day cung AB va mot cung AB cua n6]. Hay dung hai dirong tron (VI), (V2) tiep xtic ngoai nhau va cling noi tiep hinh vien phan cta cho. Bien Iuan.

B. V;ll.n dung ph ep d6ng dang vao vi~c giai cac hili toan sau day

5.13 Cho CD la dirong cao ha xu6ng canh huyen AB cua mot tarn giac ABC vuong 6 C. Chirng minh rang cac trung tuyen A M va C N cua hai tarn giac ACD va CBD vuong g6c voi nhau.

5.14 Cho hai hinh vuong cling lnrong OABC va OA' B'C' c6 chung ctinh O.

a) Chirng minh rang AA', BB' va CC' dong quy; b) AA' vuong g6c va bang CC'.

b) TIm ct¢ 100 cua g6c giira cac tia AA' va BB', AA' va CC'

5.15 Gia sir OAA', OBB' va OCC' la ba tarn giac can cling ctinh 0, bang nhau va cling luning. Chirng minh rang cac cap nrong irng BC, B' C'; C A, C' A' cua hai tarn giac ABC va A' B'C' giao nhau thea ba diem Ao, B; va Co tao thanh ,6,AoBoCo ""' ,6,ABC ""' ,6, A' B'C'.

5.16 Cho ABCD la mot til giac loi noi tiep mot dirong tron tarn O. Phep quay g6c rp < 7r xung quanh 0 bien n6 thanh til giac A' B' C' D'. Chirng minh rang cac cap canh tuong irng AB, A' B', BC, B' C'; CD, C'D' va D A, D' A' cua hai til giac ct6 cat nhau tai cac diem M, N, P va Q Ia cac ctinh cua mot hinh binh hanh.

5.17 Dung ra phia ngoai mot tarn giac ABC bat ky ba tarn giac BCM, CAN va ABP sao cho LM BC = LCAN = 45°, LBCM = LNCA = 30° va LABP = LPAB = 15°. Chirng minh rang tarn giac M N P vuong can a P.

69

5.18 Mot hinh vuong ABCD co dinh D co dinh va dinh A chuyen dong tren mot dirong (I) cho tnroc trong mat phang, khong di qua D. Tim quy dao chuyen dong cua hai dinh B, C can lai va cua tam 0 hinh vuong ABC D trong cac tnrong hop sau day

a) (I) Ia mot dtrong thang a

b) (I) la mot dtrong tron tam S, ban kinh R.

5.19 Mot di~m P chuyen dong tren nih dirong tron dirong kinh AB. Tim quy dao chuyen dong cua hai diem M va N tren dirong thang (B M) sao cho PM=PN=PA.

5.20 Cho hai dirong tron (01, R 1) va (02, R2) cat nhau a A va B. Hai dong tir M; va M2 xuat phat t11 A Iiln Iuot chuyen dong tron deu tren (01) va (02) thea cling mot hmnrong, sau mot vong tro lai A cling mot hie.

1. Chirng minh rang

a) Tam giac AM1M2 Iuon dong dang voi chinh no va duong thang M, M2 Iuon di qua B.

b) Trong mat phang co mot diem P duy nhat Iuon each deu M; va M2 6 moi thai diem (D~ thi Toan quoc te IMO, London 1979).

2. Tim quy dao chuyen dong cua cac diem sau day

a) Trung diem M cua doan thang M1M2;

b) Tam C duong tron (AM1M2);

c) Trong tam G, tnrc tam H cua tam giac AM1M2.

5.21 Dung tam giac M N P noi tiep tam giac ABC da cho, co dinh P cho tnroc tren canh AB va dong dang voi mot tam giac XY Z cho tnroc.

5.22 Dung mot hinh binh hanh noi tiep mot hinh binh hanh ABC D cho tnroc va dong dang vci mot hinh binh khac M N PQ cho tnroc.

5.23 Dung mot til giac (loi) ABC D biet tong do Ion hai goc doi dien LA + LC = f) va d¢ dai cac canh AB = a,BC = b,CD = c,DA = d.

5.24 Cho tam giac ABC. Dung tam giac XY Z noi tiep tam giac da cho voi cac dinh X, Y, Z Iiln Iuot nam tren cac canh BC, C A, AB sao cho dong dang (thuan) vci tam giac ABC va co dien tfch nho nhat.

5.25 Dung ra phia ngoai tam giac ABC hai tam giac deu BOA va CAB1. G9i Ao va B; Iiln Iuot Ia tam cua cac tam giac do va P Ia trung di~m canh AB. Chirng minh rang

70

b) Hai g6c AoP B; va AlP B, c6 chung nhau dtrong phan giac.

5.26 Mot diem P chuyen dong tren duong thing chua canh BC cua mot tam giac ABC khong vuong eta cho. Cac dirong thing di qua P vuong g6c vci AC va AB thea thtr tv cat cac dirong thing AB va AC a M va N. Tim quy tfch cua diem Q, etO'i xirng voi P qua trung diem cua M N.

5.27 Cho mot dirong thing p, mot eti~m A tren p va hai dirong tron (VI) va (V2).

Dung mot tam giac ABC nhan p lam dirong phan giac trong cua g6c A, cac etinh B va C thea thir tv nam tren (VI) va (V2) va ABIAC = min (m va ti la hai so' duong cho truce).

(4)

(4)Cae bai toan 5.25, 526, 5.27 doi hoi van dung tinh chat cua phep dong dang nghich (vi nr - doi xrrng)

Chuong 3

Tra 100. va hurmg dan giai bai tap

§ 1

1.1 GQi M; va No l~n Iuot la tiep diem cua a va b voi dirong tron (0); noi khac di, ta co the viet nhu sau d~ dien ta ky hieu do: (MoMo) = a, (NoNo) = b. Ta xet hai tnrong hop: 1) Neu a II b thi diem M; tren a khong co anh tren b con diem No tren b khong co t<;10 anh tren a. 2) Neu an b = C thi M; tren a co anh la eben B, No tren b co t<;10 anh la C tren b; nhung M, tren a, 6 do tiep tuyen thir hai voi (0) song song voi b khong co anh tren b va M; tren b, 6 do tiep tuyen thir hai voi (0) song song voi a khong co t<;10 anh tren a. Nhu vay thi ca hai tnrong hop a song song voi b va a cat b, anh X<;1 f : M f---7 N tir a ----+ b khong phai Ia mot song anh. Ta di den ket Iuan rang muon cho f : M f---7 N tir a ----+ b Ia mot song anh thi moi dirong thing a va b deu phai bo di mot diem. Neu a song song voi b thi f la mot song anh tir a\ {Mo} den b\ {No}. Con neu an b = C thi f la mot song anh tir a\{M1} den b\{N1}.

Nluin. xet. Neu ta b6 sung van moi duong thing a va b mot diem moi rna thea tnrc giac, ta goi Ia diem 6 vo tan hay diem xa vo tan, xac dinh boi phirong cua cac dirong thing d6 Ia anh X<;1 f : M f---7 N tir a ----+ b lai tro' thanh mot song anh (vi neu a II b thi M; tren a co anh la diem Noo tren b va Moo tren b co t<;10 anh la M; tren a; cling vay, neu an b = C thi M; f---7 Noo tren b va Moo f---7 N1•

1.2 Neu a .L b = 0 thi 0 nrong irng voi moi diem M cua mat phang. Neu a J- b can xet hai tnrong hop.

1) an b = 0 (co nhien a J- b), hay chimg to rang M' la tnrc tam cua tam giacOAB. Tir d6 suy ra anh X<;1 f : M f---7 M' tir P ----+ P Ia mot toan anh (nrc f Ia mot phep bien hinh cua P) khi va chi khi a J- b.

2) a II b thi diem M' diroc hoan toan xac dinh va duy nhat. Co the cluing minh rang khi do f = 'D (~) Ia phep doi xtrng co true ~ song song va each

71

72

deu a, va b.

1.3 1) Hay cluing minh rang diem M phai tim Ia giao diem tlur hai cua duong thing ,6. va dirong tron (0 P N') trong do N' doi xirng vci N qua dirong thing a.

---+ ---+

2) M N = 2QO (khong d6i), trong do Q la hinh chieu (vuong g6c) cua P

tren ,6..

1.4 1) C6 the sir dung quy tfch nhtmg diem rna 11 so khoang each den hai canh cua mot g6c la mot hang so k la mot tia nam trong g6c do (khi k = 1 thi tia do la tia phan giac cua g6c). Cling c6 the xet cac diem A', B' va C' doi xirng voi diem M liln Iuot qua cac canh BC, C A, AB cua tam giac ABC, roi cluing minh AM, BM, va CM liln Iuot Ia phan giac cac g6c LB'A'C', LC'BA' va LA'CB'. Diem M'la tam dirong tron ngoai tiep tam giac A' B' C'.

2) Chirng to rang khi M = A thi M' Ia bat err di~m nao cua dirong thing (BC).

3) Chirng t6 rang Ax' n By' n C z' = M' khi va chi khi A', B', va C' khong thing hang roi ap dung dinh ly (can va du) v~ duong thing Simson.

4)T = P\To, trong do To = (BC) U (CA) U (AB) U (ABC) (Duong tron (ABC).

1.5 Sir dung tinh chat cua phep doi hinh (bao toan khoang each) neu trong dinh nghia.

1.6 Sir dung tinh chat cua phep doi hinh neu trong H~ qua 1 cua Dinh ly 1, suy ra phep doi hinh bao toan nrong quan lien thuoc giira diem va dirong thing, ket hop vci phirong phap cluing minh phan chirng.

1.7 GQi M Ia mot diem bat ky cua P va M' = !(M), ta cluing minh M' = M (\:j M) bang phirong phap phan cluing. Neu M' =I M (\:j M) thi A, B, C thing hang tren trung tnrc t[M M'], mau thuan vci gia thiet CA, B, C khong thing hang).

1.8 Sir dung cac tfnh chat cua phep doi hmh: bao toan khoang each, bao toan 11 so cua hai doan thing cung phirong, bao toan g6c va cac tinh chat dac tnrng cua trong tam, tnrc tam va tam cac dirong tron noi, ngoai tiep cua tam giac.

1.9 Sir dung tfnh chat: Neu A' va B' Ia anh cua A va B trong mot phep tinh tien

---+ ---+ ---+

'YC~) thi A'B' = AB. Suy ra A'B' II AB neu AB lrti va (A'B') = (AB)

---+ ---+

neu AB II v.

1.1 0 Tap hop nhtrng dirong thing bat bien trong phep doi xirng tam 0 la chum dirong thing tam O.

73

1.11 Nhimg dirong thing bat bien trong phep doi xirng - true Ll ky hieu 'D (Ll) Ia nhtmg dirong thing vuong g6c voi true doi xirng Ll, con nhtmg dirong thing c6 phirong bat bien qua 'D(Ll) bao gam nhtmg dirong thing bat bien vuong g6c voi Ll) va nhtmg dirong thing song song voi Ll.

1.12 C6 tat d. sau phep trong d6 c6 ba doi hinh thuan va ba doi hinh nghich (doi xirng) Ba phep doi hinh Ia Id,Q(0,27r/3),Q(O,47r/3). Ba phep doi xtrng la 'D(t[BC]), 'D(t[CA]), 'D(t[AB]), trong d6 t[XYlla trung tnrc cua [XYl.

1.13 C6 tat d. tam phep bao gam bon doi hinh thuan va bon doi hinh nghich; bao gam: Id = Q(0,O),Q(0,7r/2),Q(0,7r) = 'D(0),Q(0,37r/2), trong d6 0= ACnBD la tam hinh vuong, va 'D(AC), 'D(BD), 'D(t[AB]), 'D(t[BC]).

1.14 Chi c6 hai phep doi hinh la Id va 'D(O) bien hinh binh hanh ABCD thanh chinh n6, trong d6 0 = AC n BD la tam hinh binh hanh.

1.15 C6 tat d. bon phep: I d, 'D ( 0) la hai phep doi xtrng - true qua trung tnrc cua mot cap canh doi dien.

1.16 Neu Ll1 .L Ll2 = 0 thi ta c6 (voi ky hieu 'D(Di) = 'Di, i = 1,2)

'D2 0 'D1 = 'D1 0 'D2 = 'D(O); 'D1 0 'D(O) = 'D(O) 0 'D1 = 'D2, 'D2 0 'D( 0) = 'D( 0) 0 'D2 = 'D,

§ 2

2.1 Dinh ly nay diroc suy ra tir Dinh ly 3 v~ su xac dinh mot phep ding cu (bao gam doi hinh va phan doi hinh) phang. Cap diem nrong irng thir ba C, C' chi c6 y nghia d~ xac dinh phep doi hinh (ding cir) diroc xac dinh boi hai tam giac bang nhau ABC va A' B'C' (trong d6 cac cap diem tuong irng la A, A'; B, B' va C, C') la thuan hay nghich, tuy thea tam giac A' B' C' cling lnrong hay khac lnrong v6i tam giac ABC.

2.2 C6 tat d. hai phep doi hinh gam mot doi hinh thuan va mot doi hinh nghich bien tam giac tlur nhat thanh tam giac tlur hai, trong d6 A f---7 A', B f---7 B', C f---7 C' hoac A f---7 A', B f---7 C', C f---7 B'. Neu ABC la doi hinh thuan

thi ABC A A' B' C' la doi hinh nghich. Neu ABC A A' C' B' la doi hinh nghich.

2.3 C6 tat d. sau phep ding cu bao gam ba doi hinh va ba phep phan chieu (doi hinh nghich). Cac phep d6 diroc xac dinh boi cac cap tam giac deu bang nhau sau day: ABC, A' B' C'; ABC, B' C' A'; ABC, C' A' B'; ABC, A' C' B' va ABC, C' B' A'.

74

2.4 VI khong c6 mot cap canh nrong irng nao song song nen phep doi hinh duy nhat bien tam giac ABC thanh tam giac A' B' C' (hai tam giac nay nay bang nhau thea gia thict) phai la mot phep quay tam 0 nao ct6. Do ct6 ton tai 0 sao cho 0 = t[AA'] n t[BB'] n t[CC'].

2.5 Sir dung dang chinh tac cua phep doi hinh nghich trong mat phang. Cac diem Ao, Bo, Co thang hang tren true ~ cua phep ct6i xtrng - tnrot true ~'D(~, ti). Tuy nhien, cling c6 the cluing minh tnrc tiep tinh chat nay bang

-----+

each ke them dirong phu. Chang han, tnroc het thuc hien 'Y(ti) voi ti = AA'

bien ABC thanh A' B"C" sau ct6 cluing minh cac trung diem B1 va C1 cua cac doan thang B' B" va C' C" va A' Ia ba diem thang hang. Tir ct6 suy ra dieu phai cluing minh.

2.6 Ky hieu ~i' i = 1,2,3 Ia cac true ct6i xtrng va 'Di = 'D(~i). Xet tfch f = 'D3 ° 'D2 ° 'D1 (1). Tich 'D2 ° 'D1 la 'Y(ti) hoac Q(O, cp) tuy thea ~1 song song hay giao voi ~2 tai O. 09i ~4 la true thir tir diroc dung sao cho 'D3 ° 'D4 = 'D2 ° 'D1. Ton tai 'D4 thea cac Dinh ly 4' va 5'). Thay van bien thirc (1) cua f thi diroc f = 'D4. Day Ia dieu phai chimg minh.

2. 7 D~ Y rang Dinh ly 8 (Bai tap 2.6 a tren) la dieu kien ctu ct~ f = 'D3 ° 'D2 ° 'D1 Ia mot phep ct6i xtrng - true 'D4. Con trong bai toan nay ta phai thiet lap dieu kien can va ctu ct~ ba duong thang ~1' ~2' ~3 cling thuoc mot churn dirong thang (dong quy hoac song song). Do la f = 'D3 0'D2 0'D, = 'D4 hay la 'D2 ° 'D1 = 'D3 ° V4 (2). Cac tfch a hai ve cua (2) deu bien thi mot phep doi hinh V nao ct6, nghia la hoac mot phep quay Q ( 0, cp), hoac mot phep

tinh tien 'Y(ti) rna 0 = ~1 n ~2 = ~3 n ~4 hoac ti .L ~i' (i = 1,2,3,4). Tir ct6 suy ra dau hieu nhan bier can tim.

2.8 Su dung Dinh ly 5', phan tfch: Q1 (01, cpd = 'D2 ° 'D1 va Q2( O2, cp) = 'D3 ° 'D2 trong ct6 'D, = 'D(~i) va ~2 = (0102). Suy ra Q20Q1 = 'D30'D" trong ct6 (~1' ~2) = ~ (mod n) va ~2' ~3 = P;f va (~1' ~3) = ~(CP1 +CP2) +kit, Cu6i cling ctua den viec xet hai tnrong hop CP1 + CP2 =I- 2kn va cP + CP2 = 2kn

2.9 Sir dung cac Dinh ly 4' va 5' rdi phan tfch Q (0, cp) va 'Y(ti) thanh tfch cua hai 'D(~i) = Vi. Cu6i cling thu diroc f = 'Y(ti)oQ( 0, cp) = 'D30'D1 = Q(w, cp), trong ct6 0 E ~1 sao cho (~1' ~2) = cp/2 (mod n); 0 E ~2 .L ti va ~3 suy ra tir ~2 boi 'Y(t) va w = ~1 n ~3. D6i voi tfch f' = Q(O, cp) ° 'Y(ti) cling lam tuong tir nhu f. Ky hieu ~~ = 'D2(~3)' ta duoc: f' = 'D~ ° 'D~ = Q'(w', cp), trong ct6 'D~ = 'D(~D, i = 1,3 va w' = ~~ n ~~

2.10 Tir gia thiet ta c6 DABC = DA' B'C'. Sau ct6 sir dung Dinh ly 3. Ton tai mot phep dang cu f : A f---7 A', B f---7 B', C f---7 C'. Sir dung gia thiet hai til giac Ia 10i nen D nam khac phia voi B ct6i voi AC, D' khac voi B' ct6i voi A'C' roi suy ra DBAD = DB'A'D' va DBCD = DB'C'D' va f cling

75

bien D f---7 D' va do d6 A' B' C' D' = hoac . ABC D tuy theo hai til giac nay cling lnrong hay ngiroc huang.

2.11 Trti lai. C6 hai phep ding cu bien ABC D thanh A' B' C' D', trong d6 mot doi hinh va mot phan chien. Phep doi hinh la phep quay Q(A, rp), trong d6 rp = (AB, AB'), (mod 27r) hoac rp = (AB, AD'), (mod 27r) tuy thea hai hinh vuong cling lnrong hay khac luning. Phep phan chieu Ia phep d6i xtrng - true '])(~), trong d6 ~ = t[CC']la trung tnrc cua [CC'].

2.12 Sir dung he tlnrc vecto bien thi H Ia tnrc tam cua tam giac ABC. Tir d6 hay cluing t6 rang bon doan thing AHi c6 cling trung diem 0' doi xtrng voi tam 0 cua duong tron ngoai tiep AIA2A3A4 qua trong tam G cua til di~m {A1,A2,A3,A4}.

§ 3

A

3.1 GQi AD, BE va CF la cac trung tuyen cua tam giac ABC. Thirc hien

---+

viec tinh tien BE thea ~ = B D, dira BE den D P thi E tro thanh trung

---+ ---+ ---+ ---+

diem chung cua CA va PF. Tir d6 suy ra AD + BE + CF = o. Vay

::3 T(AD, BE, CF) = DADP

3.2 Trti lai. AH = va2 - b2

Cluing minh tnrc tam H cua tam giac AE F Ia dinh tlur tmr cua hinh binh

---+ ---+---+

hanh EC F H. Sau d6 thuc hien tinh tien thea H F roi suy ra A H = G F,

trong d6 G la dinh thir tu cua hinh binh hanh AH FG.

3.3 Tnroc het, {O} = dirong tron (A, r) trong d6 0 la tam cua dirong tron ( v ). D~ Y rang voi moi (0, r) chuyen dong di qua A c6 hai tiep tuyen ti, (i = 1, 2) song song voi ~ rna cac tiep diem T; d6i xirng nhau qua 0 sao cho t; T2 -.l ~.

Trti lai. {T1} va {T2} la hai dirong tron (K1, r) va (K2, r) diroc suy ra

---+

tir dirong tron (A, r) = {O} boi hai phep tinh tien thea AK 1 = 1 va

---+

AK2 = -1, trong d6 [K1K2]la dtrong kinh vuong g6c voi ~ cua (A, r).

---+ ---+ ---+ ---+ ---+ ---+

3.4 D~ Y rang M N = AB hay la AM = BN nrong dirong vci AS = SN

---+ ---+ ---+ ---+ ---+

thi MS = SB (va ngiroc Iai) dong thai AM = BN = HK, trong d6

K = (MD) n (NE) va CK Ia mot dirong kinh cua dirong tron (CDE).

76

---+ ---+ ---+ ---+---+

Cu6i cling, thiet lap he thirc 2SQ = BC + AH. Suy ra SQ = OC, trong

d6 0 la tam dirong tron ngoai tiep tam giac ABC.

3.5 Gia sir d(B, CD) < d(A, CD), trong do d(X, Y Z) la ky hieu khoang each

---+

tir di~m X den dtrong thing Y Z. Thirc hien 'J'(1J' = N B) thi N f--7 B va

M f--7 D sao cho M N BD la mot hinh binh hanh va do do DB 11= M N = f.

---+ ---+ ---+

Dat M N = f va goi f Ia vecto chi phirong cua dirong thing (CD) chua

---+ ---+

dirong kinh [CD] cua (0). Suy ra BD = -f va MD II (BN) = (BP), do

do LAM D = LAP B = sp. Diem M diroc hoan toan xac dinh, M la mot trong cac giao di~m cua [CD] va cung (r) chua g6c dinh huang cp dung

---+ ---+

tren doan AD, trong do B D = - f. Bai toan c6 hai, mot hoac khong c6

Ioi giai tuy thea (r) cat, tiep xiic hay khong cat [CD].

3.6 Gia sir da dung diroc cat tuyen Llil d va cat (0) va (0') thea hai day cung nrong irng la M N va M' N' sao cho M N + M' N' = f cho tnroc. Keo

---+ ---+

dai M N ve phia N lay diem M1 sao cho M M1 = l roi di;it M M1 = f

---+ ---+

va goi l la vecto chi phinrong cua d. Thirc hien 'J'( l) : (0) f--7 (01)

---+ ---+

c6 tam 01 xac dinh boi 001 = l. Sau do thuc hien tinh tien thea

---+ ---+ ---+

v = M' N = N'M1 bien (0') thanh (w). The thi Ll cat (w) thea day cung

MM1 = f = NM1 + MIN = M'N' + MN, trong do N1 la giao diem thir hai cua Ll va (01). Suy ra w = O'u: .L t[OOl], trong do t[OOl]la trung tnrc cua doan [001]. Ket Iuan: Cat tuyen Ll phai tim Ia duong thing di qua mot giao diem cua hai duong tron (01, R), (w, R') va song song voi d, trong d6 R va R' 1iln Iuot Ia ban kinh cua (0) va (0') da cho.

Bien. liuin: Bai toan c6 nghiem (hai hoac mot) khi va chi khi

B

3.7 Hay chimg to rang Ax n By n Cz n Dt = Q = 'Do(P), trong d6 0 la tam hinh binh hanh ABC D. Va do d6 anh xa f : P f--7 Q tir P ---+ P Ia mot phep bien hinh diem cua P: phep d6i xirng tam.

3.8 Ta c6 0 = AC n BD la tam d6i xirng cua hinh binh hanh ABCD. 'D(O) :

---+ ---+ ---+

M f--7 M' E CD va N f--7 N' E DA va M'N' = -MN = NM. Suy ra

DDM' N' = DDPQ, va M' N' song song va bang PQ. Do do, M' = P va N' = Q va 0 cung la tam d6i xirng cua hinh binh hanh M N PQ.

3.9 Chien thuat choi cua ngiroi thtr nhat nhir sau: Nguoi di dilu tien di;it dong xu van tam ban, sau do doi ngiroi thir hai di, di;it dong xu 6' bat ky dau tren ban, den Iuot minh di thi di;it dong xu van vi trf doi xtrng voi dong xu cua

77

ngiroi thtr hai (vira ct:;tt tnroc ct6) qua tam ban. Chien thuat choi cua ngiroi thir nhat ctr tiep tuc lap lai nhu mroc di ct:iu tien.

3.10 Diem dong quy cua sau dirong thing Ia cti~m 0' ctoi xirng vci tam 0 dirong tron ngoai tiep ABCD qua trong tam G cua \tu diem dong vien" {A,B,C,D}.

3.11 Thirc hien phep ctoi xirng tam 'D(P), bien a, ---+ a'. Diem B tren (0) phai tim la mot giao diem cua (0) va a'.

Bien. ltuin, Bai toan c6 hai hoac mot nghiem hinh (Ioi giai) neu P nam cling phia voi (0) doi voi a va d1 - 2d2 :s; R. Bai toan vo nghiem neu P cling phia voi (0) ctoi voi a va d1 - 2d2 > R hoac P nam khac phia voi (0) ctoi vci a, trong ct6 R la ban kinh cua (0), d1 = d( 0, a) va d2 = d(P, a).

3.12 Ba dirong thing ceit M N(II BC), PQ(II CA) va RS(II AB) la cac tiep tuyen song song voi BC, CA va AB cua duong tron (1) noi tiep tam giac ABC. Luc giac M N PQ RS thu diroc khong nhtmg ngoai tiep dirong tron (1) rna con nhan I lam tam ctoi xirng.

3.13 Thirc hien phep ctoi xirng - tam 'D(P) : M f--7 N, S f--7 S', A f--7 A' va B f--7 B'. Suy ra S' la ctinh thir tir chung cua ba hinh binh hanh M S N S', AS A'S', B S B' S' c6 cling tam ctoi xirng P. Suy ra A', N, S' thing hang va B', M, S' thing hang. Mat khac, g6c noi tiep LAS B = , (khong ct6i), suy ra LAMB' = LA' N B = 7r - ,. Ding thirc nay hoan toan xac dinh cti~m M hoac cti~m N. Ching han, cti~m M la giao cti~m cua day cung CD va cung chua g6c dinh huang 7r -, (mod 27r) dung tren doan thing [B'A], trong ct6 B' ctoi xirng voi B qua P. Suy ra S = [AM) n (0). Bai toan Iuon c6 1ai giai .

c

3.14 Hay ct~ y den tinh chat sau: Duong noi tam Ow cua hai dirong tron (0) va (w) Ia true ctoi xirng chung cua hai dirong tron ct6. Neu cluing ceit nhau thi dirong noi tam hai dirong tron Ia trung tnrc cua day cung chung cua cluing.

3.15 Hinh 1{ gam dirong tron (0) va hai dirong tron bang nhau (01), (02) cling tiep xiic trong voi (0) c6 true ctoi xirng Ia trung tnrc ,6. = t[Ol' O2] cua 01, O2 trung vci dirong kinh cua (0) vuong g6c vci A1A2 va do ct6 cung la t[A1' A2]. 'D(A) bien M f--7 M', B1 ---+ B~, B2 ---+ B~, suy ra M M' II B1B~ II B2B~ va A1, B~, M' thing hang va A2, B~, M' ciing thing hang. M:;tt khac, phep vi tir V(A1' Rd R2) bien B1B~ thanh M M' II B1B~ va do ct6 suy ra (B1B~) = (B2B~), nghia la B1B~ II M M'. Suy ra B1 B2 II A1A2. Chirng minh nay, ngoai phep ctoi xirng - true con sir dung tinh chat cua phep vi tu.

78

Chti thich, C6 th~ cluing minh tnrc tiep tir LiA101B1 "" LiA10M va LiA202M2 "" LiA20M, sau d6 ap dung dinh 1y Talet cluing minh ~~~ =

~~~ suy ra B, B2 II A1A2.

3.16 Phep 'D(AB) : CD f--7 C'D' -.l CD = M E AB. Suy ra MC' = MC. M~t khac, VI LC' C D = 450 nen C'D khong d6i.

Trti lai. MC2 + M D2 = 2R2 (R = ~AB).

3.17 {M = (BD) n~} la duong tron ngoai tiep tam giac can ABC da: cho. 3.18 Gia sir hai girong thing gl va g2 diroc d~t sao cho hai mep girong cua cluing ap sat nhau tao thanh canh c cua mot g6c nhi dien LC(gl' g2) c6 g6c phang Ia g6c (nhon) xOy: Ox .L c, Oy .L c. D~ tinh dQ 1&1 cua g6c LxOy ta phai su dung dinh Iuat quang hoc: g6c phan xa bang g6c toi.

Trti lai. LxOy = 450

3.19 Tra Ioi. Diem P can tim Ia giao diem cua xy va mot tiep tuyen ke tir A voi dirong tron tam B' tiep xtic vci xy, trong d6 B' doi xirng vci B qua xy.

3.20 1) GQi D' = 'D(D), trong d6 ~ = t[AC]la trung tnrc cua AC. Khi d6 VI ABC D Ia mot trr giac 10i, ta thu diroc bat ding thirc can tim.

2) Dau ding tlnrc 6 (*) dat diroc khi va chi khi dong thai ta phai c6 s(BAD') = ~AB.AD' va s(B'D'C) = ~BC.CD' va do d6 LBAD' = LBCD' = 900, urc la ABCD' va do d6 ABCD la mot trr giac noi tiep c6 hai duong cheo AC, BD vuong g6c.

D

---+

3.21 Trti lai. Cac phep doi hinh bien (Od thanh (02) gom 'J(ti = 0102), 'D(O),

trong d6 0 la trung diem cua [0102] va vo so phep quay c6 tam w nam tren ~ = t[0102]. Cac phep phan chieu gom 'D(~) va vo so phep doi xirng - tnrot c6 true d Ia mot duong thing bat ky di qua O.

3.22 Tnroc het, cluing minh tinh chat sau day cua tam quay: Trong mot phep quay khong phai la mot phep doi xirng - tam thi tam quay, hai diem nrong irng va giao di~m cua hai dirong thing nrong irng phat xuat tir hai di~m d6 Ia bon diem cung thuoc mot dirong tron. (Sir dung g6c dinh huang (mod Jr) cua hai dirong thing). Hoac cluing minh tinh chat: Trong mot phep doi hinh (thuan), g6c giira hai dirong thing tuong irng c6 dQ 1&1 (dai so) khong d6i.

GQi ip = (a, b), (mod Jr) va 0 Ia tam cua phep quay Q(O, rp) bien doan thing M1(tdM1(t2) thanh doan thing M2(tdM2(t2), trong d6 t1, t2 la hai thai di~m nao d6 (t2 > td. The thi phep quay Q ( 0, rp) se bien M, ( t) thanh M2(t) 6 moi thai diem t.

79

---+ ---+

3.23 Gia su (01, R) n (02, R) = (P, Q) va goi (POl, P02) = o (mod 27r). Gia

su mot cat tuyen Ll bat ky di qua Q cat lai (02) a Mi, i = 1, 2; hay cluing minh DPM1M2 ~ DP0102.

3.24 GQi 0 la tam cua hinh vuong A1A2A3A4 gia su da duuoc dinh htrong dironng. Xet phep quay Q ( 0, 7r / 2) bien hinh vuong thanh chinh n6, A f---7 Ai+1 (voi quy tree As = A1). D~ thuan tien, ta ky hieu lai x, y, Z, t theo thir tir d6i thanh X1,X2,X3,X4 (X5 xem Ia Xl). The thi Q(0,7r/2): A f---7 Ai+1; AXi f---7 Ai+1Xi+1. Auy ra phep quay Q( 0, -7r /2) : AjP f---7 Aj-1xj-1 voi quy uoc x; Ia X4.

Trti lai. Diem dong quy cua AiXi Ia di~rn Q diroc xac dinh hai OQ = 0 P va

---+ ---+

(OP,OQ) = -7r/2, (mod 7r) duqesuyratltPbaiphepquayQ(0,-7r/2).

3.25 Xet phep quay Q(C, +60°) : B f---7 M', N f---7 A va do do D f---7 E. Day Ia dieu phai cluing minh.

3.26 Sir dung tfch cua hai phep quay (dinh ly ve tfch cua hai phep quay da diroc cluing minh trong bai tap 2.8). Gia sir tam giac ABC e6 lnrong dirong. Ta ky hieu Q1 = Q(B, -27r /3), Q2 = Q( C, -27r /3) roi xet tfch f = Q2 0 Q1; cluing to rang f = Q(Ao, -7r /3) tir d6 suy ra tarn giac AoBoCo Ia deu.

3.27 Hay sir dung phep quay g6e 60° theo y tuong neu trong lai giai cua Thi du 10; § 3, 3.5. (irng dung phep quay). Ching han, ta thuc hien phep quay Q(B, 60°) 0 Q(B, 60°) : C f---7 A, M f---7 N va, do d6 , DBCA f---7 DBM N la deu. Xet DNAM e6 MN = MB va NA = MC va do do, NAM chinh la tam giac T(M A, M B, MC) e6 cac canh bang M A, M B, MC :

T(M A, M B, MC) = DN AM. Neu T la vuong, chang han e6 canh huycn la M A, the thi g6e LM N A la vuong. Hay thiet lap he thirc (dong nhat thirc) sau day

(NB,NA) = (MB,MC) = (NM,NA) + (AB,AC), '11M E P

Tlt d6 suy ra (N M, N A) = 90° hay la (MC, M B) = 30°, (mod 180°)

Trti lai. {M} trong mat phang sao eho tarn giac T e6 ha canh bang M A, M B va M C la tam giac vuong gom ba dirong tron bang nhau e6 ban kinh bang canh cua tam giac deu ABC va doi mot tiep xuc nhau a cac dinh cua ABC. Tam cua ba duong tron quy tfch nay Ia cac diem Ao, B; va Co Ia cac diem Ao, B; va Co Ia cac dinh cua ba tam giac deu d6i xirng voi tam giac ABC qua cac canh cua n6.

80

§ 4

4.1 Suy ra dinh Iy 21 v~ su xac dinh mot phep 2 phang (xem chii thfch 6 cuoi cluing minh dinh Iy 22, § 4.4a). Cach cluing minh nrong 1lJ each giai hal tap 2.1

4.2 Theo giai thiet L,A' B' C' ""' L,ABC nen ton tai mot phep 2 thuan, d~ L,ABC f---7 L,A' B' C'. Nhung VI cac canh nrong irng cua hal tam giac nay lai song song voi nhau, nrc la g6c dong dang bang 0, nen 2 khong the la mot phep vi 1lJ - quay rna chi c6 the Ia mot phep vi 1lJ, (2 cling khong the Ia mot phep tinh tien VI k = B'C' / BC =11) va bat err dtrong thing nao di qua hai diem nrong img deu di qua tam vi 1lJ.

Chti thich, Cling c6 th~ cluing minh tnrc tiep bang each sir dung dinh Iy Ta let.

4.3 Thirc chat day la mot bai toan dung hmh. Gia sir c6 diem M E [AD], N E [BC] sao cho MN II AB II CD va [DCNM] ""' [MNBA]. G9i 0 =

, --2 -- --2 -(AD) n (BC), hay cluing to rang OM = OA.OD va M N = AB.DC.

Suy ra OD/OM = OM/OA = /Wi va do do, phep vi 1lJ duong V(O, iJ

bien D f---7 M,M f---7 A,C f---7 N,N f---7 B tire Ia V(O,/Wi) bien hinh thang DC N M thanh hinh thang M N B A.

4.4 Suy ra tir viec tam giac P AIB2 dong dang nghich voi tam giac P BIA2 va cap tam giac dong dang nghich nay c6 cap dtrong trung tuyen tuong irng Ia AoB2 va BoA2 cling voi cap dinh nrong img AI, BI.

4.5 Tnroc het, hay thiet lap dau hieu (dieu kien du) nhan bier hai hinh chir nhat la dong dang (suy tir dau hieu nhan biet hai tam giac vuong dong dang). D6Ia ty so hal kfch thtroc cua hal hinh chir nhat, bang nhau. Gia sir hinh chir nhat MNPQ ngoai tiep trr giac (loi) ABCD c6 AC ..1 BD; hay cluing to rang MN/MQ = BD/AC (neu A E MN,B E NP,C E PQ va DE QM).

4.6 Sir dung g6c dinh lnrong cua hal dirong thing (rnod sr ) cluing minh cac he tlnrc (A' B', A' C') = (H B', H C') = - (AB, AC) va hai he tlnrc nrong 1lJ suy ra tam giac A' B'C' dong dang nghich voi tam giac ABC.

4.7 Trti lai. C6 tat ca sau phep dong dang (thuan va nghich) nhir vay gam ba phep 2 thuan va ba phep 2 nghich bien tam giac deu thir nhat thanh tam giac deu thir hai. Hay chi ra du sau cap tam giac dong dang (goi ten) ca thuan va nghich (lien he voi bai 2.3).

4.8 Menh d~ doi hoi cluing minh da la mot phan noi dung cua cluing minh Dinh Iy 22 ve diem bat dong va dang chinh tac cua mot phep dong dang thuan.

81

4.9 Hay cluing minh rang neu 0 Ia tam cua phep dong dang thuan (vi nr quay)

-----+ -----+

Zl(O,cp,k) bien AB thanh A'B' thi DOAA' ~ DOBB' va cling vay

DOA' B' ~ DOAB. TiI d6 suy ra 0 cling Ia tam cua mot phep Z2 thuan

-----+ -----+

khac bien AA' thanh B B' .

4.10 Lai gidi 1. Sir dung cac ket qua neu ra trong cac bai tao 4.8 va 4.9 6 tren. Diem dong quy 0 cua bon dirong tron (AA'C'), (BB'C'), (ABC) va (A' B' C) chinh Ia tam chung cua hai phep dong dang Zl bien (01) = (ABC) thanh (02) = (A' B'C); va Z2 : (03) = (AA'C) f--7 (04) = (BB'C).

Lai gidi 2. Sir dung dinh ly ve dirong thing Simson

-----+ -----+

4.11 Dat (PO, PO') = cp (mod 27r), k = R' j R. Xet phep vi nr quay Z(P, cp, k)

bien (0, R) thanh (0', R'), trong d6 0 f--7 0'. Gia su Z : M E (0) f--7

-----+ -----+ -----+-----+

M' E (0'). Hay chimg to rang Z: (OP,OM) f--7 (O'P,O'M'). TiId6 suy

ra (M, M') :3 Q

4.12 1) Ky hieu (01) = (APEF),(02) = (PBCD) va (Or) n (02) = {P,Q}.

Su dung ket qua bai tap 4.11 6 tren hay cluing to rang phep vi nr - quay tam P da bien (02) thanh (01) thi cling bien hinh vuong P BC D thanh hinh vuong P E F A, trong d6 B f--7 E, C f--7 F, D f--7 A.

2) Diem c6 dinh S la tam hinh vuong dung tren [AB] nhung nam khac phia voi hai hinh vuong diroc xet d6i voi (AB). Con {Q} la mra dirong tron duong kinh [AB] nam khac phia voi S d6i voi (AB).

4.13 Chirng t6 rang c6 mot phep vi tir d6i xtrng tam 0 bien A f--7 A', B f--7 B', C f--7 C', D f--7 D'.

4.14 1) D~ thay f : M f--7 M' tir P ----+ P la mot song anh tir P vao chinh n6.

2) f Ia mot phep dong dang nghich Z cua mat phang; Z = Z(C,.6., k) tam C = an b, true .6. Ia phan giac Cp cua g6c tao boi a va b, k = cos" trong d6 , Ia g6c nhon tao hai a va b; f chinh Ia mot phep vi nr d6i xirng.

§ 5

A

5.1 D~ Y rang Al = 'D(P) = V(Ao), trong d6 V = V(P, 2), vv ...

5.2 G9i G la trong tam cua til diem {A1A2A3A4} cluing t6 rang B, diroc suy ra tir A, (i = 1,2,3,4) trong phep vi tir V(G,-lj3). Tam 01 cua dirong

-----+ -----+

tron (B1B2B3B4) diroc xac dinh boi G01 = -~GO, R1 = ~R.

82

5.3 Dat AD = BC = d (khong d6i) suy ra {C} = (B,d) va do do, {O} diroc

-----+ -----+ -----+ -----+

suy ra tir {C} nho he tlnrc AC = 2AO, hay AO = ~AC.

5.4 Dat AB = a, AC = b; suy ra BP : BN = k = a~b va tim diroc {P}.

5.5 GQi I va 0 liln hrot Ia tam cac duong tron noi va ngoai tiep tam giac ABC.

Chirng to rang ABC va AoBoCo Ia hai tam giac vi nr va Do Ia tam dirong tron ngoai tiep tam giac AoBoCo.

5.6 GQi K la tiep diem cua (J) va dirong tron (ABC), H la trung diem BC va I la trung di~m M N. Xet phep vi nr v (A, k = AB / AH) : DABC f--7 DAD E trong do H f--7 K la trung diem cua doan DE. Hay cluing minh rang AI / AJ = AH / AK va d~ y rang (J) Ia dtrong tron noi tiep tam giac ADE. Tir do suy ra I la tam dirong tron noi tiep tam giac ABC.

5.7 Hay chia mieng phang dong chat c6 dang mot nih tu giac loi (hai chieu) [A1A2A3A4] thanh hai mien tam giac bang mot dirong cheo nao do, A1A3 hoac A2A4. Sau do dua van trong tam cua hai tam giac thanh phan (rna ta da biet each tim) thi xac dinh diroc trong tam cua toan hinh T = [B1B3]n[B2B4] diroc suy ra tir P = [A1A3] n [A2A4] hai phep vi nr V( 0, -i).

5.8 Can xet hai tnrong hop: R1 =I- R2 va R1 = R2. Neu R1 =I- R2 goi I la tam vi nr ngoai cua (Oi,Ri),i = 1,2 va V = ,])(C). Xet phep vi nr V(I,k), vci k = R2/ R1 cluing minh [CAl f--7 DB]; tir do suy ra (AB)3 I va do do, LCHI = 90°.

Tra lai. Neu R1 =I- R2 thi {H} la cung tron H1 C H2 cua dirong tron dirong kinh [I C] c6 hai dilu rmit Hi, (i = 1, 2) nam tren hai tiep tuyen chung ngoai cua (01) va (02). Neu R, = R2 = R thi {H} Ia doan thing [H1H2] cua dirong trung tnrc t[0102] cua doan noi tam hai dirong tron va Hi nam tren cac tiep tuyen chung ngoai cua hai dirong tron (Oi, R).

5.9 Tarn thai khong doi hoi dieu kien dtrong tron phai di qua di~m A thi r5 rang dirong tron (w) can dung vi nr voi mot dirong tron (1) bat ky tiep xiic voi hai canh cua g6c xOy da cho. Suy ra diem A la hinh vi nr cua mot trong hai giao diem M1 va M2 cua dtrong thing 0 A voi dirong tron (I) do.

Trti lai. Bai toan c6 hai nghiem hinh nhan tir tir I boi phep vi nr v ( 0, k; = OA/OM); i = 1,2

5.10 Theo gia thiet, neu day cung AD cua dirong tron 100 (V1) da dung diroc thi ta c6 AB = BC = CD = iAD, hay AB = ~AC. Phep vi nr V(A,~) bien C ----+ B. Suy ra B la mot giao di~m cua (V2) va (V2) nhan diroc tir (V2) boi phep vi nr n6i tren, voi tam la mot diem tuy y A. Bai toan c6 the c6 hai, mot, hoac khong lai giai voi moi diem A diroc chon tren V1 tuy thea R2 < R1 § 3R2, trong do R; la ban kinh cua (Vi), i = 1,2.

83

5.11 G9i Ao, Ia trung di~rn cua BC va A' = 1>o(A). Hay cluing to rang A' = 1>A)H) va do do, Ao = V(A'), V = V(H, ~).

5.12 G9i C Ia trung di~rn cua cung bu vci cung ArB cua hinh vien phan da cho; Di, E, l~n hrot Ia tiep di~rn cua (V2), (i = 1,2) vci day AB va vci cung ArB. Hay sir dung tinh chat cua phep vi tu, cluing rninh (DiEi) di qua C, roi cluing rninh C nam tren true ding phirong cua (VI) va (V2).

Trti lai. Bai toan c6 vo so nghiem. Quy tfch tiep diem cua (VI) va (V2) Ia cung tron AB cua dirong tron (I') tarn C, ban kinh p = CA = CB nam trong hinh vien phan da cho.

B

5.13 Xet phep vi nr quay 2(D, 90°, k), trong do k = AC /CB, bien BD thanh CD, C thanh A va do do, bien C N thanh AM.

5.14 1) G9i (w) va (w') Ia cac dtrong tron ngoai tiep hai hinh vuong da cho va P la giao diem thir hai cua hai dirong tron do. The thi a) AA'nBB'nCC' = P. b) CC' nhan diroc tir AA' boi phep quay Q(O, 90°).

2) Phep dong dang 2(0,45°, k) trong do k = y'2 bien AA' thanh BB'.

5.15 Tarn giac A' B'C' diroc suy ra tir tarn giac ABC boi phep quay g6c rp =

---+ ---+ ---+ ---+ ---+---+

(OA,OA') = (OB,OB') = (OC,OC') xung quanh tarn O. G9i Al va

A~ Ia cac hinh chieu cua 0 l~n hrot tren BC va B' C', hay cluing to rang Ao = BC n B'C' nhan diroc tir Al bai phep vi nr - quay 2(0, 3, COS(~/2)). Suy ra, DABC "" DA1B1C1 "" DABC = DA'B'C'.

5.16 G9i E, F, G, H la cac hinh chieu cua tarn 0 l~n hrot tren cac canh AB, BC, CD, DA cua til giac ABCD noi tiep trong (0); cung vay, E', F', G', H' la cac hinh chieu cua 0 tren cac canh nrong irng cua A' B' C' D'. Hay cluing t6 rang M N PQ nhan diroc tir hinh binh hanh E FG H boi phep vi nr - quay tarn 0, g6c rp/2, ty so k = 1/ cos(rp/2).

5.17 Sir dung tfch cua hai phep dong dang thuan. Xet tfch 22 021, trong do 21 = 2(B, 45°, kd va 22 = 2( C, 45°, k2) trong do kl = BC / BM = AC / AN = 1/ k (vi tarn giac CAN dong dang nghich voi tarn giac C B M). Hay cluing to rang 22 0 21 Ia mot phep doi hinh c6 mot diem bat dong P duy nhat va g6c doi hinh rp = 90°. Tir do suy ra PM vuong g6c va bang PN.

5.18 Gift su hinh vuong ABCD co huang thuan. The thi, 2(D, ~,j2) : A f--7 B; Q(D, ~); A f--7 C va V(D,~) : B f--7 O.

Trti lai. 1) G9i H la hinh chieu cua D tren a roi dung hinh vuong D H H' K c6 huang thuan. The thi ta diroc {B} Ia dirong thing b vuong g6c voi D H' 6 H'; {C} = c, c -.l DK = K; {O} = (HK).

84

2) Dung hinh vuong D 5 5,52 c6 huang thuan (cling huang Val ABC D); goi w HI. trung diem cua D51• Chirng t6 rang quy tfch cua B HI. dirong tron (51RV2), quy tfch cua C HI. dirong tron (52, RV2) va quy tfch cua 0 HI. dirong tron (w, RV;).

5.19 G9i ten cac dl~m M va N sao cho tam giac AP N ngiroc huang Val tam giac AP M diroc gia sir la c6 huang thuan, nghia la

-----+ -----+ -----+ -----+ 7r

(AP, AN) = -(AP, AM) = -"4 va AM = AN = APV2

Trti lai. Dung tam giacc AB1B2 vuong can a A, nhan AB Ia dirong cao ha xuong canh huyen B1B2 va cling huang nghich voi tam giac AM N vuong can 6 A. Chirng t6 rang {M} Ia mra dirong tron (ABB1) dtrong kinh AB1 va {N} la mra dirong tron (AB~ B2) duong kinh AB2 (c6 tam W2 la trung diem cua mra duong tron da: cho), trong do B~ doi xirng voi B1 qua trung dl~m 0 cua [AB], dong thoi B~ = 'Dw2(B) va cling Ia dinh thtr tir cua hinh vuong ABB2B;.

-----+ -----+ -----+-----+

5.20 1) Cluing minh (OlA, 01M1) = (02A, 02M2), 'lit, tir do suy ra (M1M2) :3

B va tam giac AM1M2 dong dang Val tam giac A0102. b) Trung tnrc t[M1M2J :3 Pia trung diem cua doan thing B1B2, trong do B, = 'Do(B), (i = 1,2)

2) a) {M} la dirong tron (0) dirong kinh BP c6 tam la trung diem 0 cua 0102; can hru y them rang {M} cling di qua B vi c6 (M A, M B) = / (khong d6i) do tam giac AM1M cling Iuon dong dang Val chinh n6. b) Quy tfch cua C la dirong tron ngoai tiep tam giac A0102. c) G diroc xac dinh

-----+ -----+ -----+ -----+

hal AG = ~AM; H dUQ'C xac dinh hal CH = 3CG.

Trti lai. {G} la dirong tron (Go,P1 = GoA), trong d6 Go la trong tam cua tam giac A0102; {H} la dtrong tron (Ho,P2 = HoA), trong do H; la tnrc tam cua tam giac A0102.

5.21 Ph/in. tick. Gia sir rang tam giac M N P da: dung duoc, nghia la M E

-----+ -----+ -----+ -----+

BC,N E CA sao cho (PM,PN) = /, ;~ = k, trong d6 / = (ZX,ZY)

va k = ZY /ZX. Suy ra, N = (CA)na', trong do a' nhan diroc tir a = (BC) boi phep vi nr - quay Z(P, /, k).

Bien. ltuin, Bai toan n6i chung c6 mot nghiem hinh duy nhat, trir tnrong hop a' II CA hoac a' = (CA) thi so nghiem bang 0 hoac 00.

5.22 Neu hinh binh hanh M'N'P'Q' noi tiep hinh binh hanh ABCD thi cac tam 0' va 0 cua cluing trung nhau (Bai tap 3.8). Nhu vay bai toan dUQ'C quy v~ hal toan 5.21 a tren. D6 la hal toan: Dung tam giac OM' N' noi tiep tam giac ABC c6 dinh 0 Ia trung dl~m canh CAva dong dang Val tam giac wM N cho tnroc, trong do M N la mot canh va w la tam cua binh hanh M N PQ cho tnroc.

85

5.23 Tnroc het, d~ y rang neu LA + LC = f) = n thi noi dung hal toan chinh HI. thi du 11 trong 5.2. § 5. Nhu vay, bai toan nay la mot dang khai quat hoa thi du d6. Tuy nhien, lai giai cua bai toan lai hoan toan tuong tir voi lai giai trong thi du d6. Trong bai van quy ve dung L,DCE ""' L,DAB, chi khac Ia E t/:. (BC) VI

LBCE = LBCD + LDCE = LC + LA = () =In

5.24 Hay cluing to rang bon dirong tron vl(AYZ),V2(BZX),V3(CXY) va V4 (XY Z) c6 cling ban kinh. Sau d6, su dung phep chieu vuong g6c, chieu Y Z len BC, chirng minh ~~ 2: ~.

Ket lu~n.min Sl(L,XY Z) = ~S(ABC) dat khi va chi khi ~~ = k = ~ nrong dirong voi L,XY Z = L,AoBoCo.

Trti lai. Tam giac can tim Ia tam giac trung binh AoBoCo cua tam giac ABC.

5.25 1) Chirng minh cac tam giac AoBlP va BoAlP Ia nhtmg mra cua hal tam giac deu, vuong 6 P, c6 LAo = LBo = 60° va LBI = LAI = 30° (sir dung dinh ly ve tfch hai phep quay, bai 2.8).

2) TiI viec tam giac AoBlP dong dang nghich voi tam giac BoAlP suy ra cac doan [AoAlJ, [Bo, PI] diroc nhin tir P duoi nhtmg g6c bang nhau.

5.26 Chirng minh P Ia tnrc tam cua tam giac AM N va do d6, Q Ia dl~m xuyen tam - d6i voi diem A tren dirong tron (0) ngoai tiep tam giac AM N.

Trti lai. Quy tfch cua Q Ia dirong thing q, nhan diroc tir dirong thing a = (BC) boi phep vi tir- d6i xirng Z(A, Ll = Ap, k), trong do Ap Ia phan giac g6c LBAC va k = ICO~AI = ±co~A tuy thea LBAC 100 han hay nho han 90°.

Chti thich, Bai toan nay chinh la dang dao cua Thi du 13 (VMO, 3/2002).

5.27

86

Hinh 52

Lai giai. Gia sir rang tam giac ABC da dung duoc, thoa man cac doi hoi cua bai toano The thi diem C diroc suy ra tir diem B boi phep dong dang nghich (vi nr - dO'i xirng) Z(A,p, k = -;;) tam A, true p va ty sO' k = nf m. Do do diem C can tim la mot diem chung cua hai dirong tron, do la dirong tron (V2) va duong tron (v~) nhan diroc tir dirong tron (VI) boi phep dong dang nghich Z(A,p, n/m) noi tren (Hinh 52). Bai toan co th~ co hai, mot hoac vo nghiem tuy theo (v;) cat, tiep xiic hoac khong cat (V2).

Chti thich, D~ thay rang bai toan se vo nghiem khi va chi khi dirong tron (V2) nam ngoai g6c Lt~ At; anh cua g6c tIAt2 trong phep dO'i xtrng truc'D (Ap) trong do Ati Ia hai tiep tuyen ke tir A den (VI)' Day Ia mot thi du img dung phep dong dang nghich (vi nr - dO'i ximg) vao giai toan dung hmh, xem la mot thf du he) sung vao 5.3, § 5 (Thi du 1.4).

Chuong 4

Bai tap b6 sung va On tap tu giai

§ 1 B6 toe ve goc dinh hurmg trong mat phang

1.1 a) Gia sir M HI. mot diem nam trong mat phang cua mot g6e xOy da eho.

G9i X va Y liln Iuot HI. diem doi xtrng cua M qua cac canh Ox va Oy. Chirng t6 rang dQ 100 cua g6e LXOy khong phu thuoc va vi trf cua diem M (M =I 0).

b) Cho M la mot di~m bat ky nam trong mat phang cua mot tam giac ABC da eho. G9i A', B', C' liln Iuot Ia cac diem doi xirng cua M qua cac dtrong thing BC, CAva AB. Chirng minh rang cac dirong phan giac Ax,By va Cz cua cac g6e LB'AC', LC'BA' va LA'C'B dong quy hoac song song. 11m quy tfch nhtmg diem M trong mat phang d~ Ax, By va Cz song song vci nhau.

1.2 a) Cluing minh rang g6e cua cac dirong thing chua hai day eung AB va

CD cua mot dirong tron (0) diroc eho hai cong thirc

1 -----+ -----+ -----+-----+

(AB, CD) = "2{(OA, ~C) + (OB, OD)} + br

b) G9i 0: la so do dai so cua eung AC nam trong g6e LADC va (3 la so do dai so cua eung BD nam trong g6e LBAD, cluing minh rang

-----+ -----+ -----+ -----+ -----+ -----+ 1

(AB, CD) = (AB, AD) + (DA, DC) = "2(0: + (3) + 2br

1.3 Gia su hai dirong tron (ABC) va (ABD) tnrc giao vci nhau. Chirng to rang hai dirong tron (ACD) va (BCD) cung vay,

1.4 G9i B' la di~m doi xirng cua B qua canh CA va C' la di~m doi xirng cua C qua canh AB cua mot tam giac ABC. Tinh B'C' thea LA = 0: va OA = b, AB =c

87

88

1.5 GQi A', B' va C' thea thtr nr Ia cac di~m doi xirng cua cac dinh A, B va C cua mot tarn giac ABC qua cac canh BC, CAva AB. 11m dieu kien can va du ve dang cua tarn giac ABC d~ A' B' C' la mot tarn giac deu (D~ thi HSG toan quoc, VMO 3/1994)

1.6 Gia sir D, E, F la ba diem bat ky diroc lay l~n Iuot tren cac dirong thing chua cac canh BC, CA va AB cua mot tarn giac ABC.

a) Chirng t6 rang ba dirong tron (AEF), (BFD) va (CDE) dong quy 6 mot diem M, goi la diem Mi-ken (B6 d~ Miquel).

b) Tfnh g6e (DE, DF) thea cac g6e (AB, AC) va (M B, MC);

c) 11m quy tfch cua diem M khi D, E va F thay d6i nhung Iuon thing hang

1.7 Cho mot tarn giac ABC. Dung ba dirong thing Ax, By va Cy sao eho

(AB,Ax) = (CB,Cz) va (AC,Ax) = (BC,By)

Chirng minh rang Ax, By va Cz dong quy 6 mot diem D tren dirong tron (ABC).

1.8 Chirng minh dinh Iy: Cac dirong tron ngoai tiep bon tarn giac tao h6i cac canh cua mot tu giac (trr canh) toan phan dong quy 6 mot diem w rna cac hinh chieu cua n6 tren cac canh la bon diem thing hang.

1.9 Chirng t6 rang cac hinh chieu cua mot diem w nam tren mot dirong tron tren sau canh cua mot tu diem noi tiep dirong tron do la cac dinh cua mot trr giac (nr canh) toan phan.

1.10 Dinh ly Miquel ve sau dirong tron: Bon dirong tron VI, V2, V3 va V4 sap d:;tt trang mat phang sao eho VI n V2 = {A, A'}, V2 n V3 = {B, B'}, V3 n V4 = {C, C'} va V4 n VI = {D, D'}. The thi cac diem A', B', C', D' dong vien (hoac thing hang) khi va chi khi cac diem A, B, C, D dong vien (hoac thing hang).

1.11 Cho mot duong tron (0) va hai diem A, B co dinh tren duong tron do sao eho khong Ia hai di~m xuyen tarn - doi, mot dtrong kinh XY thay d6i cua (0). 11m quy tfch giao di~m P cua (AX) va (BY). (D~ thi Olympic Toan cua My, USOMO, 1976).

1.12 Trang mat phang eho tarn giac ABC noi tiep dirong tron (0). Xac dinh mot diem P khong thuoc dirong tron (0) d~ cac dirong thing P A, P B, PC cat lai (0) l~n Iuot 6 A', B', C' sao eho tarn giac A' B' C' vuong can day B'C' (D~ thi HSG toan quoc, VMO 3/1999, bang B).

89

§ 2 Dai cuong ve cac phep bien hinh va cac phep diri hinh phang

2.1 Trong mat phang cho ba diemt A, B, P phan biet, thing hang va mot dirong thing ~ vuong g6c voi (AB) 6 D. M9t diem C chuyen dong tren zx. 09i A' va R' la hinh chieu cua A va B liln Iuot tren BC va CA, AA' cat RR' 6 diem H tren ~.

a) Hoi anh xa f : C f--7 H tir ~ ---+ ~ c6 phai la mot phep bien hinh cua ~ khong?

b) 09i M la hinh chieu cua C tren dirong thing (PH). Hoi anh xa h : H f--7 M tir duong thing ~ len (I) = {M} c6 phai Ia mot song anh khong [ bien ~ thanh I = h(~)]?

2.2 Trong mat phang cho trr giac loi ABCD noi tiep mot dirong tron (0). M9t diem P chuyen dong tren duong tron (0). Cac duong thing D P va C P cat dirong thing ~ di qua A va B thea thtr nr a M va N.

a) Chirng minh rang neu b6 sung van dirong thing ~ mot diem moi rna thea tnrc giac goi Ia diem vo tan hay diem xa vo tan cua ~ thi anh xa f : M f--7 N tir ~ van chinh n6 Ia mot phep bien hinh cua dirong thing ~.

b) Chirng minh rang khi diem P chuyen dong tren dirong tron (0) thi ty so kep (AB / M N) c6 mot gia tri khong d6i k xac dinh.

c) Tir tinh chat n6i tren , cluing minh rang khi P chuyen dong tren (0) thi dai Iuong AM.N B / M N = p (khong doi).

2.3 Trong mat phang P cho mot tam giac deu ABC. 09i A', B' va C' thea thir nr la cac hinh chieu tren cac dirong thing BC, CAva AB cua mot diem M bat ky tren mat phang va G Ia trong tam cua tam giac A' B' C'. Chirng minh rang anh xa f : M f--7 G tir P ---+ P Ia mot phep bien hinh cua mat phang.

Huang ddn. 09i 0 la trong tam cua tam giac ABC, hay cluing minh G la trung diem cua OM).

§ 3 SO' xac dinh va dang chinh tac cua mot phep dOl hinh phang

3.1 Trong mat phang dinh hu6ngP cho hai doan thing bang nhau AB va A' B'.

V 6i moi ditim M bat ky cua p, hay chi r5 nhtmg he thirc v~ d9 dai va v~ g6c (g6c dinh hirong) lien he giira cac cap ditim A' A; B, B' va M, M' dti

90

M'la anh cua M trong phep doi hinh thuan hoac trong phep doi hinh nghich (phan doi hinh) bien A thanh A', B thanh B' (va co nhien la M thanh M').

3.2 Chirng minh dinh 1y sau day (neu len tfnh chat dac tnrng cua phep doi hinh thuan trong mat phang).

---+ ---+

Dinh 1Y. Phep doi hinh phang bien mot vecto AB thanh mot vecto A' B' co

cung etQ dai va trong moi phep doi hinh thuan, mot vecto eta cho bat ky t<;10

---+ ---+

voi vecto anh cua no mot goc khong etch (AB, A' B') = ip khong et6i, goi

la goc doi hmh. Til et6 suy ra rang phep doi hinh (thuan) la mot phep quay hoac mot phep tinh tien tuy thea goc doi hinh cua no khac khong hay bang khong. Va etoi voi phep quay xung quanh mot diem thi goc doi hinh chinh la goc quay.

3.3 Chirng minh cac menh et~ sau

a) MQt phep dang cu trong mat phang neu eta c6 hal etl~m bat dong phan biet P va Q thi moi diem khac tren dirong thing (PQ) cung deu la diem bat dong va phep dang cu do la phep etoi xirng true va true etoi xtrng Ia dirong thing (PQ).

b) MQt phep dang cu trong mat phang co mot diem bat dong 0 duy nhat Ia mot phep quay xung quanh tam 0 mot goc ip (sai khac 2k7r) nao do.

3.4 Hay sir dung ket qua neu trong bai tap 2.7 (thuoc muc 2.5 cua § 2) ve tfch cua ba phep etoi xirng - true co cac true cung thuoc mot chum duong thing et~ he) sung mot 10'1 giai ntra cua hal tap 1.4* 1) va 3) thuoc muc 1.4. cua § 1, trong et6 khong han che etl~m M phai thuoc mien [L::.ABC], rna M la mot diem bat ky cua mat phang cua tam giac ABC.

3.5 Trong mat phang cho ha dirong thing phan biet ~l, ~2 va ~3. Ky hieu 'Dk la phep etoi xirng - true 'D(~k)' k = 1,2,3. Xet phep bien hinh 9 = .[2, trong do j = 'D3 0 'D2 0 'Dl. TIm dieu kien can va etu ve ba dtrong thing ~k noi tren et~ 9 J d.

3.6 Ky hieu .ft, h va h l~n Iuot la cac phep etoi xirng true 'D(AD), 'D(BE) va 'D ( C F) thea thir tir qua cac dirong phan giac AD, BE va C F cua mot tam giac ABC. Xet tfch cua phep bien hinh j = h 0 h =L, 9 = h 0 h 0 h, va h = I, 0 h 0 h. Cluing minh rang

a) i, 9 va h deu Ia nhtmg phep doi xtrng true, hay xac dinh cac true doi xirng.

b) j = 'D(I B'), 9 = 'D(I A') va h = 'D(IC'), trong do I la tam duong tron noi tiep tam giac ABC; A', B' va C' l~n hrot la cac tiep eti~m cua (I) tren cac canh BC, CAva AB.

91

§ 4 V *0 dung phep dOi hinh vao viec giai mot s6 hili toan hinh hoc phang

4.1 Chirng minh rang trong so nhtmg hinh binh hanh thi duy nhat chi co hinh thoi ngoai tiep diroc mot dirong tron.

4.2 Tren doan vuong goc chung AB cua hai dirong thing song song a, va b (A tren a, va B tren b) ta lay hai di~m M va N sao cho AM = N B. Tren a, lay diem P va tren b lay diem Q sao cho doan PQ diroc nhin tir M duoi mot goc vuong. Chirng minh rang PQ cling diroc nhin tir N duoi mot goc vuong.

4.3 Hay chi fa each khoi phuc mot hinh ngfi giac ABC D E da bi xoa di nhung chi con d~ lai cac trung diem 01, O2, 03, 04, va 05 cua cac canh lien tiep AB, BC, CD, DE va EA cua no.

4.4 Trong mat phang cho ba dtrong thing x' x, y' y va z' z dong quy 6 mot diem o va mot diem P nam ngoai ba duong thing do. Cluing minh rang co mot tam giac duy nhat nhan x, y, z lam ba duong trung tnrc cua cac canh va di~m P da cho nam tren mot canh nao do cua no.

4.5 Hinh thang ABC D co hai day AB = a, CD = b (a < b) cac canh ben BC = c, D A = d. Cac tia phan giac trong cac goc LA va LD cat nhau 6 M va cac tia phan giac trong cac goc LB va LC cat nhau 6 N. Chirng minh M N II AB II CD va tinh khoang each M N thea a, b, c va d.

4.6 Chirng minh dinh ly Pappus):

Lay cac canh AB va AC cua mot tam giac ABC lam day duci, dung ra phia ngoai tam giac da cho hai hinh binh hanh tuy y ABDE va ACFG. Cac day tren keo dai cua hai hinh binh hanh do cat nhau 6 mot diem P.

a) Chirng minh rang hinh binh hanh BCM N dung ra phia ngoai DABC co cac canh ben song song, cling lnrong va bang P A co dien tfch bang tong dien tfch hai hinh binh hanh vira dung.

b) Tir dinh ly Pap Puyt hay suy fa dinh ly Pitago (Pythagore).

4.7 M9t diem M nam trong tam giac ABC, chuyen dong song song voi canh BC cho den khi cat canh CA; sau do lai chuyen dong song song voi AB cho den khi c:it canh BC, roi lai chuyen dong song song voi CA cho den khi cat AB, vv... Chirng minh rang sau mot so biroc, quy dao chuyen dong cua di~m M se diroc khep kin.

4.8 Gia sir M Ia mot diem bat ky thuoc mat phang cua tam giac ABC, lay diem M, doi xirng vci M qua (BC), M2 doi xirng vci M, qua (CA) va M3 doi xirng voi M2 qua (AB). Sau do, lai tir diem M3 (dong vai tro nhu diem M)

92

lay d6i xirng lien tiep qua (BC), (CA) va (AB) thi diroc cac di~m M4, M5, va M6; ky hieu M6 = M'. Chirng to rang anh xa f : M f---7 M' Ia mot phep tinh tien cua mat phang; hay xac dinh vecto tinh tien.

4.9 Chirng minh rang neu doan thang M N n6i trung di~m cap canh d6i dien BC va DA cua mot til giac loi ABCD bang trung binh cong hai canh d6i dien con lai A B va CD thi til giac do la mot hinh thang.

4.10 Hai dinh cua mot tarn giac tl tren hai dirong thang song song cho tnroc.

Tim quy tfch cua dinh thir ba.

4.11 Qua mot diem chung A cua hai duong tron ceit nhau (01, R1) va (02, R2) da cho, ngiroi ta ke mot cat tuyen ,6. thay d6i cat (Oi) a Ci, i = 1,2. Tren ,6. ngiroi ta lay ve hai phia cua A cac diem M va N sao cho AM = AN = ~C1 C2. Tim quy tfch cac diem M va N.

4.12 Dung mot doan thang co phirong p cho tnroc, co d9 dai bang £ cho tnroc va hai d:iu nut tua tren hai dirong tron cho tnroc.

4.13 Dung mot til giac cho biet d9 100 cac goc va d9 dai hai duong cheo.

4.14 Hai thanh pho A va B bi ngan each nhau boi mot con song, cling muon xay mot cay cau chung CD bac qua song d~ thuan tien cho viec di lai sao cho doan dirong bo AC DB n6i hai thanh pho la ngan nhat.

a) Hoi nguoi ta dii tim ra vi trf d~ bac cay cau do nhu the nao? (Cac be song diroc giai thiet la la song song va cay cau bac qua song dmnroc gia thiet la vuong goc voi hai bo song)

b) Cling cau hoi nhu tren neu cac thanh pho A va B bi ngan each boi nhieu (lOO han hai) con song cling co nhuir cau phai bac nhtmg cay cau qua tat ca cac con song do sao cho doan dirong bo n6i hai thanh ph6 la ngan nhat.

4.15 Chirng minh rang

a. M9t til giac co tarn d6i xirng Ia mot hinh binh hanh;

b. M9t hinh gioi noi khong the co qua mot tarn d6i xiing;

c. M9t hinh Iuc giac loi ABCDEF co cac canh doi dien song song va bang nhau thi co tarn d6i ximg;

d. Khong co hinh nao co dung hai tarn d6i xirng.

4.16 Gia sir M Ia mot diem nam trong (mat phang cua) mot tarn giac ABC co cac hinh chieu tren cac canh BC, CAva AB nrong img Ia P, Q va R. Duong tron ngoai tiep tarn giac PQ R cat lai cac canh noi tren thea thtr nr a P', Q', va R'. Chirng minh rang cac di~m P', Q' va R' cling Ia hinh chieu tren cac canh nrong irng cua tarn giac ABC cua mot diem M' nao do cua mat phang dong thai la cac cap dirong thang AM, AM'; B M, B M' va

93

C M, C M' thea thtr nr d6i xirng vci cac dirong phan giac cua cac g6c A, B va C cua tam giac ABC.

4.17 Trong mat phang cho ba di~m phan biet 01,0203, va M la mot di~m bat ky cua mat phang. G9i M1 Ia di~m d6i xirng vci M qua 01, M2 d6i xirng voi M1 qua O2, M3 d6i xirng voi M2 qua 03, M4 d6i xirng voi M3 qua 01, M5 d6i xtrng voi M4 qua O2, M6 d6i xtrng voi M5 qua 03• Chirng minh rang M6 trung voi M.

4.18 Cho mot g6c xOy va mot diem P nam trong g6c d6. Hay ke mot cat tuyen qua P cat Ox 6 A va Oy 6 B sao cho doan AB nhan P lam trung diem.

4.19 Cho mot til giac loi M N PQ. Chirng minh rang hoac khong c6, hoac neu c6 thi c6 vo s6 til giac loi ABC D ngoai tiep til giac M N PQ da cho sao cho M, N, P, Q la trung diem cac canh lien tiep AB, BC, CD, DA cua ABCD. Hay xac dinh tap hop cac dinh A, B, C va D.

Huang ddn. {A}, {B}, {C}, va] D} deu la nhtmg mien da giac phang; hay xac dinh cu the nhtmg mien d6 thea M N PQ da cho trong tnrong hop ton tai ABC D ngoai tiep.

4.20 Chirng minh rang neu mot hinh phang c6 mot tam d6i xirng duy nhat va mot true d6i xirng thi tam d6 nhat thiet phai nam tren true va do d6, n6 can c6 true d6i xtrng thir hai vuong g6c voi true thir nhat 6 tam d6i xiing.

4.21 TIm hinh dang cua tat d. cac til giac c6 dung mot true d6i xirng.

4.22 Dung til giac ABC D cho bier d9 dai cac canh va dirong cheo AC Ia phan giac cua g6c LA.

4.23 Dung til giac ABC D ngoai tiep diroc mot duong tron cho biet d9 dai hai canh k~ nhau DA = a, AB = b va cac g6c LB = (3, LD = 6.

4.24 Dung tam giac ABC biet canh AB = c, chien cao he va hieu cac g6c A va B la LA - LB = ().

4.25 Cho hai dirong tron (Or) va (02) va mot dirong thing ~. TIm tren tam giac mot diem P sao cho hai tiep tuyen voi (Or) va (02) ke tir P nghieng deu vci ~.

4.26 Cho mot g6c nhon xOy va hai diem P, Q nam trong g6c d6. Hay tim tren Ox mot di~m A sao cho g6c LP AQ dinh ra tren canh Oy mot doan thing BC Ia day cua mot tam giac ABC can a A.

4.27 Dung mot tam giac c6 chu vi nho nhat noi tiep mot tam giac nhon ABC cho tnroc.

4.28 Cluing minh cac tinh chat sau day cua tam quay

94

a) Trong moi phep quay khong phai la phep doi xirng-tam, tam quay, hai diem nrong irng va giao diem cua hai dirong thing nrong irng phai xuat tir hai diem do thi dong vien (nrc la cung nam tren mot dirong tron).

b) Duong phan giac ngoai cua goc giira hai true nrong irng trong mot phep quay thi di qua tam quay.

e) Neu trong mot phep quay nao do rna A' la anh cua A, H' la anh cua B va hai duong thing AA', B B' ceit nhau 6 mot diem Co thi giao diem thtr hai 0 cua hai dirong tron (ABCo) va (A' B'Co) Ia tam cua phep quay do.

4.29 Lay cac canh AB, BC, CD va DA cua mot trr giac loi ABCD lam canh, dung ra phia ngoai trr giac do bon hinh vuong eo tam goi thea thtr nr Ia M, N, P va Q. Chirng minh rang M P vuong goc va bang N Q.

4.30 Dung ra phia ngoai tam giac ABC hai hinh vuong ABM N va BCPQ.

Chirng minh rang cac trung di~m cac canh cua trr giac CAM Q Ia cac dinh cua mot hinh vuong.

4.31 GQi P va Q thea thtr nr Ia trung di~m cua dirong cheo BD va canh EF cua mot hie giac deu ABCDEF. Chirng minh rang APQ Ia mot tam giac deu.

4.32 Luc giac loi ABCDEF noi tiep mot duong tron tam 0 ban kinh R, eo d9 dai cac canh AB = CD = E F = R. Chirng minh rang

a) Cac trung diem M, N, P cua cac canh BC, DE va FA la cac dinh cua mot tam giac deu;

b) Cac trung diem I, J, K cua cac duong cheo AD, C F va BE ciing la cac dinh cua mot tam giac deu.

4.33 Cluing minh Dinh 1y v~ duong thing Stay-ne (Steiner): Cac diem doi xirng vci mot di~m M tren dirong tron ngoai tiep doi vci cac canh cua mot tam giac ABC thi nam tren mot dirong thing di qua tnrc tam cua tam giac do. (Duong thing nay goi la dirong thing Stayne).

4.34 Cho dirong tron (0) va mot di~m A eo dinh tren dirong tron. M9t day eung M N cua (0) eo d9 dai l eho tnroc chuyen dong sao eho eung M N eo huang khong d6i (chang han ngiroc chieu kim dong ho). TIm quy tfch cua diem P, doi xtrng voi M qua dirong thing AN.

4.35 M9t tam giac deu ABC eo dinh C eo dinh va dinh A chuyen dong tren mot dirong thing a eho tnroc, khong di qua C. TIm quy tfch cua dinh B.

4.36 Trong mat phang eho hai dirong thing a, b va mot diem C nam ngoai hai dirong thing do. Hay dung mot tam giac ABC vuong can 6 C sao eho A tren a va B tren b.

95

4.37 Dung mot tam giac deu co dinh nam tren ba dirong tron dong tam cho tnroc.

Bien Iuan.

4.38 Cho tam giac ABC va mot diem P tren canh BC. Hay dung mot tam giac M N P can 6 dinh P, noi tiep tam giac da cho va co goc 6 dinh L M P N = rp cho tnroc.

4.39 11m trong tam giac ABC mot diem P sao cho neu goi A', B' va C' liln hrot la hinh chieu cua P tren cac canh BC, CA va AB thi ta diroc BA' = CB'=AC'.

4.40 G9i x, y, z la cac khoang each tir mot diem bat ky nam trong mot tam giac deu canh a den cac dinh cua tam giac do. Chirng minh he tlnrc

4.41 Trong mat phang cho tam giac ABC noi tiep mot tam giac AoBoCo cho tnroc. Chirng minh rang ngoai tam giac ABC ra, con mot tam giac thir hai A' B'C' cling noi tiep tam giac AoBoCo va bang tam giac ABC da cho.

4.42 Tren hai canh AC va AB cua mot tam giac deu ABC liln Iuot Hfy hai diem P va Q sao cho ca hai tam giac ABP va ACQ deu nhon. G9i R, S liln hrot Ia tnrc tam cac tam giac ABP va ACQ, T Ia giao di~m cua BP va CQ. Hay xac dinh tat ca cac gia tri cua so do cac goc LCBP va LBCQd~ tam giac T RS Ia deu. (Bai toan thi Olympic Toan Chan A-Thai Bmh Duong - APMO XIV, 3/2002.)

4.43 Gift sir M la mot diem bat ky thuoc mat phang tam giac ABC. G9i M; Ia diem doi xirng voi M qua AB, M2 doi xirng voi Ml qua AC va M' doi xirng v6'i M2 qua BC.

a) 11m quy tfch nhtmg diem M trong mat phang sao cho dQ dai M M' dat gia tri nho nhat.

b) Cluing minh rang gia tri nho nhat d = min M M' khong phu thuoc van thtr nr viec lay doi xirng lien tiep qua cac canh cua tam giac ABC. (D~ thi chon dQi tuyen Toan quoc gia, VMO 5/1995).

§ 5 SlJ X3C dinh va dang chmh tac cua mot phep

dong dang phang

5.1 Chirng to rang mot phep vi nr hoan toan diroc xac dinh boi ti so (he so) vi nr k va mot cap diem nrong irng.

5.2 Chirng minh dinh ly

96

---+ ---+---+

a) Hinh vi nr cua mot vecto AB HI. mot vecto A' B' = kAB.

b) Dao Iai, moi phep bien hinh trong d6 vecto noi hai diem bat ky A va M va vecto noi cac anh A' va M' cua cluing thoa man he tlnrc

---+ ---+

A'M' = kAM HI. mot phep vi tir voi k =11, hoac mot phep tinh tien

voi k = 1.

c) Tir d6 suy ra H~ qua la: M9t phep vi nr diroc xac dinh boi viec cho

---+ ---+

hai vecto nrong irng AB va A' B' song song nhung khong bang nhau.

5.3 Ap dung dinh ly neu trong bai tap 5.2 b tren, cluing minh dinh ly 20 v~ tfch cua hai phep vi tu, phat bieu trong muc 4.2d, thuoc bai 4. (D6 Ia mot each cluing minh ntra dinh ly 20, ngoai each cluing minh da cho trong bai 4,4.2d)

5.4 Chirng minh rang tfch cua mot phep vi nr va mot phep tinh tien thea thtr nr d6 hay theo thir nr ngiroc lai Ia mot phep vi nr c6 cung ti so.

5.5 Cluing minh rang moi phep vi nr V ( 0, k) thi nrong dirong voi tfch cua mot

---+

phep vi nr Vi ( 01, k) va mot phep tinh tien song song vci vecto 001 •

5.6 Su dung phep vi tu, cluing minh dinh ly Xe-va (Ceva)

5.7 Cac true vi tir cua ba dirong tron. Gia sir (01,R1), (02,R2) va (03,R3) la ba dirong tron khong bang nhau (Ri =I R, va c6 tam khong thing hang. The thi cac duong tron nay vi nr voi nhau tirng doi mot. Cluing minh rang sau tam vi nr cua ha dtrong tron d6i mot vi nr vci nhau thi diroc phan bo tren bon dirong thing, goi la cac true vi nr cua ba dirong tron. N6i mot each khac, sau tam vi nr cua ba dirong tron la cac dinh cua mot tu giac (trr canh) toan phan rna moi canh khong chua hoac chua dung hai tam vi tir am (ty so vi nr k < 0).

5.8 Ap dung: Cac tiep diem cua mot dirong tron tiep xuc voi hai dirong tron da cho thi thing hang voi mot tam vi nr cua hai duong tron d6 khong Ia anh cua nhau trong phep vi nr nrong irng.

5.9 Dung mot dirong trong di qua mot diem cho tnroc va tiep xuc voi hai dirong tron cho tnroc.

5.10 Dung mot dirong tron di qua mot diem cho tnroc va tiep xuc voi ba dirong tron cho tnroc c6 tam khong thing hang.

97

§ 6 V *n dung phep dong dang vao viec giai mot s6 hili toan hinh hQC phang

6.1 Hai dirong tron (01) va (02) tiep xtic vci nhau a mot di~m M. Hai cat tuyen a va b di qua M cat lai (01) va (02) l~n Iuot 6 cac cap diem AI, A2 va B1, B2• Chirng minh rang

a) Duong thing A1B1 song song voi duog thing A2B2

b) Cac tiep tuyen cua (Od va (02) tai Al va A2 (cling nhu tai B1 va B2) song song voi nhau.

6.2 Chirng minh rang trong mot hinh thang ABC D cac trung diem E, F cua hai day AB, CD; giao diem I cua hai canh ben keo dai va giao diem J cua hai dmrong cheo AC, BD Ia bon diem thing hang va lam thanh mot hang di~m dieu hoa.

6.3 H'inh thang ABCD co hai day AB va CD. 09i I = (AD) n (BC) va J = AC n BD. Chirng minh rang

a) Duong tron 0: va (3 ngoai tiep cac tam giac ABI va DCI tiep xtic vci nhau.

b) Cac dirong tron r va 6 ngoai tiep cac tam giac AB J va CD J tiep xiic voi nhau.

c) Ti so cac ban kinh cua r va 6 bang ti so cac ban kinh cua 0: va (3.

6.4 09i ~ Ia mot tiep tuyen chung ngoai cua hai dirong tron (01) va (02) tiep xiic ngoai nhau 6 diem M, tiep xiic voi (01) 6 A va O2) 6 R. M9t cat tuyen di qua M cat lai (01) 6 E. Cluing miinh rang [C E] Ia mot duong kinh cua dirong tron (01).

6.5 Trong mat phang eho tam giac ABC va mot di~m 0 co dinh. M9t dirong thing ~ chuyen dong nhung Iuon Iuon song song vci AB cat AC a A'va (BC) 6 B'. TIm quy tfch cua cac diem sau day

a) Trong tam G cua tam giac 0 A' B'.

b) Dinh thir tir P cua hinh binh hanh A' 0 B' P (nhan 0 A'va OB') lam hai canh.

6.6 Cho mot goc xOy, mot diem A tren Ox va mot diem B tren Oy. Hai dong tir M1 tren Ox va M2 tren Oy l~n Iuot xuat phat tir A va B chuyen dong deu tren hai tia Ax va By voi van toe khong d6i nhung khac nhau VI va V2 = kV1 (0 < k =I- 1). TIm quy tfch cua di~m M sao eho M, va M2 l~n Iuot Ia cac hinh chieu song song cua M tren Ox va Oy thea phirong PI va P2 eho tnroc (nghia la M M, II Pi, i = 1,2; M1 E Ox, M2 E Oy) sao eho Pi khong song song voi Ox va Oy.

98

6.7 Cho ba di~m A, B, C thing hang thea thtr nr do va mot dirong tron (0, R) khong di qua diem nao trong ba diem do. M9t diem M chuyen dong tren ( 0) dirong thing A M cat lai (0) 6 diem N va cat lai dirong tron ngoai tiep tarn giac BCN 6 diem P. TIm quy tfch cua P.

6.8 Cho mot g6e xOy va mot dirong tron (I) nam trong g6e do. Hay dung mot dirong tron tiep xuc voi (I) va tiep xuc voi hai canh cua g6e da eho.

6.9 1. Duong tron (1) noi tiep tarn giac ABC tiep xiic voi canh BC 6 diem

D, dirong tron (J) bang tiep g6e LA cua tarn giac ABC tiep xuc voi BC tai diem E. Chirng minh rang mot trong hai giao diem cua AR va (1) la diem Dl, xuyen tarn doi cua D tren (1).

2. Dung mot tarn giac ABC biet ban kinh r cua dirong tron noi tiep, chieu eao AH = h (ha tir A xuong BC) va hieu b - c = £ cua hai canh CA va AB.

6.10 Dung mot duong tron di qua hai diem A, B eho tnroc va tiep xiic voi mot dirong thing d eho tnroc. Bien Iuan.

6.11 Cho tarn giac ABC. Dung bon duong tron bang nhau (Ao), (Bo), (Co), va (Do) thea thtr nr tiep xtic vci hai canh cua cac g6e LA, LB, LC, cua tarn giac da eho can (Do) thi tiep xtic vci ba dirong tron kia. Hay tfnh ban kinh chung p cua bon dtrong tron thea cac ban kinh r va R cua cac dtrong tron noi va ngoai tiep tarn giac ABC.

6.12 Trong mat phang eho hai diem A, B phan biet va mot dirong tron (0) di qua A nhung khong di qua B. V 6i m6i diem M tren dirong tron (0) ta dung dirong tron (01) di qua M va tiep xuc voi (AB) 6 A. Duong tron ( 01) cat lai tiep tuyen At 6 diem A cua duong tron (02) ngoai tiep tarn giac ABM a giao di~m thtr hai N (# A). TIm quy tfch cua di~m N khi M chuyen dong tren dirong tron (0).

6.13 GQi D, E, F nrong img la chan cac duong phan giac trong cac g6e A, B, C cua mot tarn giac khong can ABC va Al, B1, C1 Ia cac tiep di~m thtr hai cua cac tiep tuyen voi dtrong tron noi tiep tarn giac ABC ke tir D, E, F. Chirng minh cac dirong thing AoAl, Bi.B, va CoCl trong do Ao, Bo, Co liln Iuot la trung diem cac canh BC, CA va AB dong quy 6 mot diem nam tren duong tron (1) noi tiep tarn giac ABC (D~ thi olympic Toan lien bang Nga, 1998).

6.14 1. Cluing minh rang hai hinh vuong bat ky ABiCiDi, (i = 1,2) thi dong

d~ng

2. Xac dinh tarn dong dang g6e dong dang va ti so dong dang cua phep vi nr - quay bien hinh vuong AoBoCoDo thanh hinh vuong A1B1C1Dl d~ cap den trong bai toan 5.1 cua muc 5.4, bai 5.

You might also like